Paediatrics - Questions and Problems

Lakukan tugas rumah & ujian kamu dengan baik sekarang menggunakan Quizwiz!

An 18-month-old girl presents to A&E with severe stridor. On examination she has mild subcostal recession and a mild tracheal tug. The observations show SpO2 95 per cent and HR 140bpm. (Treatment of respiratory distress) A Palivizumab B Dexamethasone C Cefotaxime D Erythromycin E Amoxicillin F Nebulized salbutamol G Salbutamol via inhaler H Nebulized adrenaline I IM adrenaline J IV adrenaline

B Dexamethasone Dexamethasone for moderate croup.

Iron deficiency anaemia (Peripheral blood film) A Macrocytic RBCs B Microcytic RBCs C Normocytic RBCs D Spherocytes E Sickle cells F Neutropenia G Eosinophilia H Acanthocytes I Thrombocytosis J Thrombocytopenia

B Microcytic RBCs Microcytic RBCs. Microcytic hypochromic anaemia is seen in iron deficiency.

'Slapped cheek' appearance. (Exanthems) A Varicella zoster B Parvovirus C Measles D Epstein-Barr virus E Group A Streptoccocus F Enteroviruses G Rubella H Human herpes virus 6 I Herpes zoster J Lyme disease

B Parvovirus Parvovirus. Characteristic rash (erythema infectiosum). Incubation 4-14 days. Prodrome 3-7 days, then rash 1-4 days. The rash spreads to the limbs and can be associated with arthropathy. Treatment is supportive.

A 4-year-old girl presents to A&E with sudden onset of a limp. She has had an URTI in the last week and last night was crying out in pain while asleep. On examination she is afebrile. CRP and ESR are normal. (Limping child) A Developmental dysplasia of hip B Transient synovitis C Septic arthritis D Perthes' disease E Osteomyelitis F Slipped upper femoral epiphysis G Osgood-Schlatter's disease H Juvenile idiopathic arthritis I Non-accidental injury J Duchenne's muscular dystrophy

B Transient synovitis Transient synovitis is the most common cause of acute hip pain in children ages 3-10 years. The presentation includes painless limp and nocturnal pain.

A 6-week-old baby born at 28 weeks gestation on the neonatal unit, who still requires oxygen via nasal cannulae. (Treatment of respiratory distress) A Palivizumab B Dexamethasone C Cefotaxime D Erythromycin E Amoxicillin F Nebulized salbutamol G Salbutamol via inhaler H Nebulized adrenaline I IM adrenaline J IV adrenaline

A Palivizumab Palivizumab. RSV prophylaxis reduces risk of admission in high-risk cases, but is controversial because of high costs and need for monthly IM injections throughout RSV season.

A 1-day-old baby boy was found to have a large red birth mark that covered his cheek and orbit on one side of his face. The neonatal team were concerned and referred him for neuroimaging. (Newborn skin problems) A Port wine stain B 'Strawberry' naevus C Congenital melanocytic naevus D Sebaceous naevus E Mongolian blue spot F Stork mark G Erythema toxicum H Staphylococcal skin infection I Milia J Transient neonatal pustular melanosis

A Port wine stain Port Wine Stain. Present at birth, grow with infant and do not fade with age. Need a senior review, and if the face and orbital area involved need to consider Sturge- Weber Syndrome and investigate further with neuroimaging.

X-ray: A 4-week-old boy on the neonatal unit was born at 28 weeks' gestation and has been on nasogastric feeds for 1 week. He suddenly starts vomiting and has a prolonged apnoea. The figure shows his abdominal X-ray. a) What is the diagnosis? b) Name the pathognomonic feature of this condition seen on this X-ray?

a) Necrotizing enterocolitis. b) Pneumatosis intestinalis (gas within the bowel wall produced by bacteria seen radiologically and histologically).

Urgent investigations for the collapsed neonate include: a) Lumbar puncture b) Electroencephalogram (EEG) c) Four-limb blood pressure d) Brain MRI e) Bilirubin

c) True. This is to rule out coarctation of the aorta, which is a duct-dependentlesion and can be treated acutely with prostaglandin. a) False. This is done when stable. b) False. d) False. e) False.

Risk factors for child abuse include: a) Stable family relationships b) Both parents in full-time employment c) Supportive family network d) Children >4 years old e) Disabled child

e) True. a) False. b) False. Unemployment. c) False. d) False. Typically those <4 years old.

The gold standard diagnostic investigation in coeliac disease is: a) Antigliadin IgA antibodies b) Antigliadin IgG antibodies c) Tissue transglutaminase antibodies d) Faecal fat studies e) Small bowel biopsy

e) True. a) False. b) False. c) False. d) False. Although all these can be used for the diagnosis of coeliac disease, most clinicians consider positive mucosal histology and full clinical recovery on a gluten-free diet mandatory to confirm the diagnosis.

Typical febrile convulsions are: a) Focal seizures b) > 15 min duration c) Recurrent within 24 h d) Associated with neurological abnormalities e) Generalized tonic-clonic seizures

e) True. a) False. b) False. c) False. d) False. The incorrect answers are seen in atypical or complex febrile seizures, which are associated with a higher incidence of subsequent epilepsy.

Types of presentation of cystic fibrosis in children include: a) Bronchiolitis b) Early passage of meconium c) Female sterility d) Rectal polyps e) Rectal prolapse

e) True. a) False. Bronchiectasis - dilation of the airways - is common in cystic fibrosis b) False. Delayed passage of meconium as a result of meconium ileus. c) False. Male sterility. d) False. Nasal polyps.

Possible consequences of breast-feeding include: a) Increased risk of eczema b) Reduced maternal post-natal weight loss c) Increased risk of pre-menopausal breast cancer d) Increased risk of obesity e) Reduced risk of neonatal infection

e) True. a) False. Risk is reduced. b) False. Risk is increased. c) False. Risk is reduced. d) False. Risk is reduced.

Clinical features of Duchenne muscular dystrophy include: a) Neonatal hypotonia b) Myopathic face c) Distal weakness d) Normal creatine kinase e) Calf hypertrophy

e) True. a) False. This is feature of myotonic dystrophy. b) False. This is a feature of myotonic dystrophy. c) False. There is proximal weakness. d) False. There is grossly elevated creatine kinase.

Clinical features that increase the probability of asthma include: a) Symptoms with colds only b) Isolated cough in absence of wheeze or difficulty in breathing c) History of moist cough d) Normal peak expiratory flow e) Widespread wheeze

e) True. See British Thoracic Asthma Guidelines (2011) www.brit-thoracic.org.uk/guidelines/ asthma-guidelines.aspx. a) False. This lowers probability. b) False. This lowers probability. c) False. This lowers probability. d) False. This lowers probability.

Features of Henoch-Schönlein purpura include: a) Large vessel vasculitis b) Blanching rash c) Frank haematuria d) Small joint arthritis e) Abdominal pain

e) True. There is dipstick haematuria. There is large joint arthritis (knees, ankles or elbows). Colicky abdominal pain is commonest, but there is also intussusception, perforation and appendicitis. a) False. There is small vessel vasculitis. b) False. Purpuric rash occurs on buttocks and thighs. c) False. d) False.

Emergency treatment of status epilepticus includes: a) Lamotrigine b) Sodium valproate c) Clonazepam d) Pyridoxal e) Lorazepam

e) True. a) False. b) False. c) False. d) False.

Investigation of recurrent urinary tract infections in a 4-year-old should involve: a) Ultrasound during the acute infection b) Ultrasound within 6 weeks, DMSA, and a micturating urethrogram (MCUG) c) Ultrasound within 6 weeks and an MCUG d) Ultrasound within 6 weeks and a DMSA e) Ultrasound within 6 weeks

e) True. a) False. b) False. c) False. e) False. See National Institute for Health and Clinical Excellence (NICE) guidelines: http://www. nice.org.uk/nicemedia/live/11819/36030/36030.pdf

Innocent murmurs are: a) Diastolic b) Pansystolic c) Loud d) Continuous e) Systolic

e) True. Always. a) False. These are pathological murmurs. b) False. These are pathological murmurs. c) False. These are pathological murmurs. d) False. These are pathological murmurs.

Presentation of Kawasaki's disease include: a) Koplik spots b) Fever for 48 h c) Vesicles on hands and feet d) Purulent conjunctivitis e) Mucosal changes of mouth and lips

e) True. There is a strawberry tongue, cracked lips and red pharynx. a) False. These occur in measles. b) False. It is for >5 days. c) False. Enteroviruses. d) False. Non-purulent.

Urgent management of respiratory arrest includes: a) Endotracheal tube insertion b) Laryngeal mask airway insertion c) Nasopharyngeal airway insertion d) Oxygen mask e) Bag-valve mask ventilation with oxygen

e) True. There is no necessity to intubate; adequate ventilation can be achieved with good quality bag-mask ventilation. a) False. b) False. c) False. d) False.

Skipping, draws square, washes face, dresses on own. (Development) A Newborn B 2 months C 3 months D 6 months E 9 months F 1 year G 2 years H 3 years I 4 years J 5 years

J 5 years

Can draw a simple picture of people. (Developmental milestones) A 6weeks B 6months C 30months D 9months E 12months F 18months G 2years H 3years I atbirth J 5years

J 5years

A previously healthy 2-year-old boy collapses on the street after crying constantly for 10 minutes. (Fits) A 'Breath-holding' attacks B Absence C Infantile spasms (West's syndrome) D Benign rolandic E Juvenile myoclonic F Lennox-Gastaut syndrome G Tonic-clonic H Benign myoclonic epilepsy of infancy I Landau-Kleffner syndrome J Febrile convulsion

A 'Breath-holding' attacks 'Breath-holding' attack. A history of suddenly going limp after an episode of significant crying.

Smiles at his mother and watches faces. (Developmental milestones) A 6weeks B 6months C 30months D 9months E 12months F 18months G 2years H 3years I atbirth J 5years

A 6weeks

Cystic fibrosis. (Inheritance) A Autosomal recessive B Automsomal dominant C X-linked recessive D Trinucleotide repeat E Polygenic inheritance F Imprinting G Chromosomal H X-linked dominant I Codominant J Mitochondrial

A Autosomal recessive

Folate deficiency (Peripheral blood film) A Macrocytic RBCs B Microcytic RBCs C Normocytic RBCs D Spherocytes E Sickle cells F Neutropenia G Eosinophilia H Acanthocytes I Thrombocytosis J Thrombocytopenia

A Macrocytic RBCs Macrocytic RBCs are seen in macrocytic anaemia with reduced folate levels.

A 2-year-old girl is taken to her GP because her mother has noticed that she is passing frequent loose stools. The stools are offensive and contain undigested peas and carrots. Her mother is noted to suffer from irritable bowel syndrome. (Diarrhoea) A Toddler's diarrhoea B Hirschsprung's disease C Coeliac diasease D Inflammatory bowel disease E Intussusception F Haemolytic-uraemic syndrome G Henoch-Schönlein purpura (HSP) H Necrotizing enterocolitis (NEC) I Infective gastroenteritis J Food hypersensitivity

A Toddler's diarrhoea Toddler diarrhoea affects children from 6 months to 5 years old. It presents with colicky abdominal pain, flatus, abdominal distension and loose stools with undigested food. The child is otherwise well. Reassure parents

Red macules progress to papule, vesicle and then pustule. (Exanthems) A Varicella zoster B Parvovirus C Measles D Epstein-Barr virus E Group A Streptoccocus F Enteroviruses G Rubella H Human herpes virus 6 I Herpes zoster J Lyme disease

A Varicella zoster Varicella Zoster. The characteristic rash usually starts on head and trunk and spreads to rest of body. The lesions start as macules and progress through stages to papule, vesicle, pustule and crusting with differing stages accruing at the same time. The lesions usually completely heal by 2 weeks. Infectious period is 2 days before rash appears and after it has crusted over.

A 15-year-old girl is brought into accident and emergency from school having disclosed to a friend that she took 10 paracetamol tablets last night. Her blood level of paracetamol is below the treatment line, her liver function tests and clotting are normal. Her father died of a brain tumour 3 years ago and her mother is being treated for reactive depression. The girl tells you that she has been feeling low lately, particularly because she does not think she will do well in her up-coming exams. She regrets taking the tablets and does not think she will do it again. What is the most appropriate management? A. Admit for monitoring of liver function and Child and Adolescent Mental Health Services (CAMHS) assessment B. Refer to CAMHS as an outpatient and discharge as not currently suicidal C. Refer to Social Services D. Start antidepressant - fluoxetine E. Start IV Parvolex

A. Admit for monitoring of liver function and Child and Adolescent Mental Health Services (CAMHS) assessment She should be admitted for observation until CAMHS have assessed her (A) and created a follow-up plan. As she presented after the window of intervention for her overdose, her liver function should be monitored. While she denies feeling suicidal now, it is important that she be risk- assessed by mental health services and be discharged with a safety net that she can access if her feelings are driving her to repeat self-harm activities. She should therefore not be discharged (B). Social Services (C) may be required if it is felt that the mother's depression is leading to issues of neglect, and sometimes Social Services and CAMHS will see patients together but this is more common with illicit drug or alcohol misuse. An antidepressant would not be the first line treatment for adolescent depression and she should first be assessed by CAMHS. Parvolex (E) is used to treat and reverse the toxicity of paracetamol; however it is ineffective beyond 15 hours post-overdose.

A 14-year-old slightly overweight boy is brought into accident and emergency from a football match where he slipped and fell but was unable to get back up due to pain in his right leg, which is now looking shortened and externally rotated. X-rays show the right femur to be disconnected from the femoral head almost completely at the level of the epiphysis. What is the most appropriate management? A. Analgesia, nil by mouth until emergency internal fixation can be performed B. Antibiotics and nil by mouth while waiting for an open reduction operation C. Analgesia and bed rest with traction until healed D. Analgesia and a hip spica cast E. Reassure and mobilize with physiotherapy as tolerated

A. Analgesia, nil by mouth until emergency internal fixation can be performed This child has presented with a slipped upper femoral epiphysis, which is more common in teenage boys who are obese and often occurs with a minor injury. The management requires internal fixation (A) typically with a pin. It is rare for open reduction (B) to be used as this approach is associated with an increased incidence of avascular necrosis of the femoral head. Bed rest and traction (C) are not the best option as the risk of avascular necrosis is too high. The femur needs to be urgently pinned to the femoral head and therefore mobilization (E) is wrong. A hip spica cast (D) is used for developmental hip dysplasia.

A 12-year-old girl has been seeing her GP for the last year with heavy periods and had suffered with bleeding gums when she was younger. She is otherwise well and lives with her adoptive parents who now have parental responsibility. Her coagulation tests reveal normal prothrombin time (PT) and activated partial thromboplastin time (APTT), low factor VIII, low von Willibrand factor (vWF), abnormal platelet aggregation and increased bleeding time. What is the likely inheritance of her condition? A. Autosomal dominant B. Autosomal recessive C. X-linked D. Robertsonian translocation E. Sporadic mutation

A. Autosomal dominant This girl has von Willebrand's disease (vWD) and presents with a picture of mucosal bleeding which can be associated with platelet disorders (of actual number or function). This would suggest the mild phenotype of vWD, type 1 with partial deficiency of vWF. There are three types as follows: • Type 1 - partial deficiency of vWF • Type 2 - defective vWF • Type 3 - complete deficiency of vWF vWD type 1 and 2 are autosomal dominant (A) (type 3 is autosomal recessive) and a family history may be useful in the history. Haemophilia A and B are X-linked disorders (C). Platelet disorders are often secondary to other causes (consumption, destruction, defective production in bone marrow) but aplastic anaemias such as Fanconi's anaemia are inherited in an autosomal recessive (B) manner. Robertsonian translocations (D) (fusion of two acrocentric chromosomes near the centromere with resultant loss of the short arms) can result in Down's syndrome or recurrent pregnancy loss. Sporadic mutations (E) are not known to be associated with the development of vWD.

A 9-month-old boy presented to his GP with lethargy and a prominent forehead. He is pale on examination and has yellow sclerae. He is the first child of his non-consanguineous parents. His haemoglobin is 6.5g/dL, WCC 5.0×109/L, platelets 300× 109/L. His blood film shows evidence of haemolysis, no spherocytes, no sickle cells and a good reticulocyte count. Direct antiglobulin test (DAT) is negative. What is the most likely diagnosis? A. Beta thalassaemia B. Sickle cell disease C. ABO incompatibility D. Hereditary spherocytosis E. G6PD deficiency

A. Beta thalassaemia Beta thalassaemia (A) is a haemolytic anaemia of autosomal recessive inheritance. Thalassaemia presents with anaemia after 6 months of age, when HbA is starting to be produced. Thalassaemia is present in Mediterranean and Asian populations and is due to a defect in the production of haemoglobin beta chains or alpha chains (beta and alpha thalassaemia, respectively). Patients can be homozygous (disease) or heterozygous (carriers). They present with anaemia and jaundice and may show evidence of extramedullary haemopoeisis (frontal bossing and maxillary overgrowth). Bloods tests will show a normocytic anaemia with hyperbilirubinaemia. A reticulocyte response would be expected with normal bone marrow function. Other non-immune haemolytic anaemias include other haemoglobinopathies such as sickle cell disease (B) (HbSS on Hb electrophoresis and sickle cells on blood film), red cell membrane defects (spherocytosis (D) and elliptocytosis seen on blood film) and red cell enzyme defects (G6PD deficiency (E) and pyruvate kinase deficiency, enzyme testing required for diagnosis), which usually present with acute crises which can be haemolytic, aplastic or with chronic anaemia and jaundice, splenomegaly due to increased red cell destruction. Immune- mediated haemolytic anaemias include ABO (C) or rhesus incompatibility. Carefully examine the mother's blood group (more likely if she is O and the child is A or B, and rhesus negative) and the DAT which may be positive.

A mother brings her 2-year-old daughter to the GP on a Monday morning. Over the weekend she became very upset on being told 'no'. She was screaming and then held her breath, went blue and fainted. She woke up quickly and seemed okay afterwards. However, it has just happened again this morning when she found some scissors and her mother took them away. On this occasion she had a brief generalized convulsion lasting about 10 seconds. What is the most likely explanation? A. Breath holding attacks B. Reflex anoxic seizures C. Absence epilepsy D. Wolff-Parkinson-White syndrome E. Vasovagal syncope

A. Breath holding attacks Breath holding attacks (A) are not uncommon in toddlers at times of temper tantrums. They grow out of them and no treatment is required. Parents need to be reassured that the brief seizure is not harmful. A reflex anoxic seizure (B) typically occurs when a child is frightened or hurt, such as a bump to the head. The child goes pale and faints, hypoxia may cause a short seizure, but the child quickly recovers. Absence epilepsy (C) presents with short vacant periods where the child is unresponsive, followed sometimes by a brief period of confusion when they are aware they have missed something. Wolff-Parkinson-White syndrome is associated with a spontaneous onset re-entry tachycardia or supraventricular tachycardia which may lead to dizziness, shortness of breath and sometimes fainting. An ECG would show a delta wave. Vasovagal syncope, or a simple faint, is associated with standing for long periods of time in warm environments.

A 20-year-old man presents to the infectious diseases department with a large 7cm×8cm swollen painful lump in the left anterior triangle of his neck. He has night sweats, 10 kg weight loss and a dry cough for the last month. He was treated with surgery and radiotherapy for a high grade astrocytoma when he was 8 years old. Which of the following is not a recognized complication of his childhood condition and its treatment? A. Finger clubbing B. Haematological malignancy C. Educational difficulties D. Short stature E. Infertility

A. Finger clubbing Children with malignancies may present later in life with problems including secondary malignancy due to high levels of radiation during radiotherapy, commonly leukaemias and lymphomas (B). Educational difficulties (C) may be multifactorial in aetiology, with long periods missed from school due to ill health or poor ability to concentrate, seeing or hearing depending on the extent of the malignancy. Radiotherapy can affect the spine and growth hormone, both of which can contribute to growth problems (D). Irradiation and chemotherapy can also render gonadal tissue dysfunctional (E). Egg harvesting for females and sperm storage for males should be considered prior to therapy. Clubbing (A) is not a recognized complication of radiotherapy or chemotherapy. The causes of clubbing include cystic fibrosis, bronchiectasis, cyanotic congenital heart disease, inflammatory bowel disease and liver cirrhosis.

A 4-year-old boy with severe ezcema is brought to accident and emergency by his mother. His skin has been worse recently since the weather has become colder. He is scratching a lot more and now is very miserable and has a temperature of 38.6°C today. On examination of his skin he has multiple areas of erythematous, excoriated lesions on his elbow and knee flexures as well as his trunk and back. In addition they are hot, tender and slightly swollen with areas of broken skin. There are also some yellow fluid-filled vesicles on some of these lesions. You send some blood tests and commence him on IV flucloxacillin and aciclovir. Which are the two most likely organisms that can complicate eczema? A. Gram-positive cocci and herpes simplex virus B. Gram-negative cocci and herpes simplex virus C. Gram-positive cocci and varicella zoster D. Gram-negative bacilli and herpes zoster E. Gram-positive bacilli and herpes simplex

A. Gram-positive cocci and herpes simplex virus Cellulitis (bacterial superinfection) and eczema herpeticum (herpes simplex virus infection) are the two important complications of eczema. Gram-positive cocci (Staphylococcus spp. and Streptococcus spp.) are responsible for cellulitis. These occur due to breakage of the skin barrier in the eczematous areas allowing entry of bacterial into an already inflamed epidermis and dermis. These require aggressive treatment to prevent spread of the organisms into the blood. (B) is not correct as Gram- negative cocci (Neisseria meningitidis and N.gonorrhoeae) do not produce skin infections. Varicella zoster is the causative organism for chicken pox, and reactivation of the virus which lays dormant in the dorsal root ganglion is known as herpes zoster (shingles). They do not specifically cause infection of eczematous skin; therefore (C) and (D) are incorrect. (E) is incorrect as Gram-positive bacilli such as Clostridium spp., Bacillus spp. or Listeria spp. do not typically cause primary skin infections.

A 3 year old is brought to see the GP with multiple pearly raised papules with central umbilications. They have been there for more than a month on his torso and upper legs. His mother is worried he has warts. What is the most likely diagnosis? A. Molluscum contagiosum B. Congenital warts C. Scabies D. Melanocytic naevi E. Guttate psoriasis

A. Molluscum contagiosum This child has a common condition of molluscum contagiosum (A) caused by a pox virus. They are usually self-limiting and will self-resolve within a year. Occasionally one may become superinfected with bacteria and require antibiotics. Particularly stubborn cases may be treated with cryotherapy to hasten resolution. 'Congenital warts' (B) is a term that usually refers to genital or anal warts, and the distribution and description of the lesions in this child are not consistent with this diagnosis. There is some controversy regarding the maximum incubation period for genital warts that are acquired from contact with maternal genital warts during delivery. These may take over a year or two to evolve and when they appear, child sexual abuse may erroneously be suspected. Scabies (C) cause itchy rashes due to the burrows of the mite in the skin, usually on the hands, axillae or groin and often involving the web spaces between the fingers. A melanocytic naevus (D) would be a hyperpigmented papule which would not usually have a central punctum. Guttate psoriasis (E) often follows an upper respiratory tract infection and produces raindrop- like scaly pink patches on the torso and arms.

A 5-year-old is referred to paediatrics due to concerns initially raised by his school teacher that he is weak and clumsy. On examination he has wasting of his quadriceps and walks in a waddling gait. His blood creatine kinase is 1600 mmol/L (normal is 24-190). What is the most likely diagnosis? A. Muscular dystrophy B. Neglect with failure to thrive C. Malnutrition with failure to thrive D. Acute myositis E. Spinal muscular atrophy

A. Muscular dystrophy This child presents with a waddling gait and wasting of proximal muscles which points to a muscular dystrophy (A), which is strongly supported by the raised creatine kinase. You would expect a positive Gower's sign, in which the child is unable to get up from the floor without walking his hands up his legs for support. Spinal muscular atrophy (E) would present in infancy with decreased tone and the child would never be able to walk. There is no information in the question stem about the child's growth, and unless you are given serial weights and centiles it is impossible to say there is failure to thrive ((B) and (C)). There is no mention of pain, which makes acute myositis (D) an unlikely diagnosis.

A couple who are known to both be carriers of cystic fibrosis ask to see you. They had genetic counselling but declined antenatal diagnostic testing and their baby has now been born and is ready to be discharged home. The parents are now keen to get the baby tested so that if treatment is required it can be initiated early on. What initial test do you suggest for the baby? A. Newborn blood spot screening B. Chest x-ray C. Faecal elastase D. Genetic testing E. Sweat test

A. Newborn blood spot screening Newborn blood spot screening (NBSS) is a national screening programme to target early diagnostic testing for cystic fibrosis, hypothyroidism, phenylketonuria (and sickle cell disease and medium chain acyl dehydro- genase deficiency in some areas). The NBSS (A) is a blood spot sample and for cystic fibrosis tests for immune reactive trypsin; if positive, the sample will be sent for further genetic analysis to try to identify the gene mutation (D). The child will then require a sweat test, the gold standard test, to confirm the diagnosis (E). A chest x-ray in the newborn will not be useful (B). Faecal elastase (C) is useful to detect pancreatic insufficiency secondary to cystic fibrosis, but not all children will have pancreatic involvement so is not a diagnostic test.

A 2-year-old boy is brought to accident and emergency for the sixth time and is found to have a right-sided non-displaced transverse fracture of his tibia. His parents state that he was running in the living room and tripped landing on a toy truck. He has broken his other leg twice, several fingers and his right arm previously. He appears healthy, is well dressed and his growth is normal. His mother is very upset, she is 5 months pregnant with their second child and her anomaly scan yesterday suggested the baby has a broken leg. What is the most likely explanation for these fractures? A. Osteogenesis imperfecta B. Domestic violence and child abuse C. Osteopetrosis D. Achondroplasia E. Clumsy child

A. Osteogenesis imperfecta Osteogenesis imperfecta (A), also known as brittle bone disease, is a collagen metabolism disorder which is typically autosomal dominantly inherited and has variable penetration where the most severe forms may develop fractures in utero. While risks for child abuse (B) should always be explored in any injury and repeated injuries are a worrying sign, this does not explain the fetus with a broken bone. Osteopetrosis (C) is an autosomal recessive disorder of dense brittle bones associated with frequent fractures, failure to thrive, recurrent infections, hypocalcaemia and thrombocytopenia. This is not the correct answer, as his growth is normal. Achondroplasia (D) results in short stature due to marked shortening of the limbs but is not associated with fractures. Stating that a child is clumsy (E) is not an adequate diagnosis to explain repeated injuries; either there is an underlying diagnosis as to why the child is so unsteady and injuring themselves repeatedly, or there is a bone abnormality or there is child abuse, all of which need to be investigated.

A 4 year old is brought to accident and emergency acutely unwell and refusing to walk for the past 2 days. Her parents are not aware of any recent injuries. On examination, she is pyrexial (T = 39.2°C), capillary refill 3 seconds centrally, heart rate 150 beats per minute, respiratory rate 40 breaths per minute. Her right thigh is swollen and slightly erythematous but too tender to examine fully. An x-ray of the hip and femur shows soft tissue swelling surrounding the proximal femur but the bones look normal. An urgent MRI shows a periosteal reaction in the proximal femur with extensive inflammation in the surrounding soft tissues. What is the most likely diagnosis? A. Osteomyelitis B. Non-accidental injury C. Cellulitis D. Reactive arthritis E. Juvenile idiopathic arthritis

A. Osteomyelitis This child's presentation is worrying for osteomyelitis (A) or septic arthritis, in view of the high fever, tachycardia, tachypnoea and a swollen painful leg. This is why the MRI was arranged so quickly. X-rays do not show osteomyelitis changes until about 2-3 weeks later, but the soft tissue swelling may be noticed. An MRI is required to see the inflammatory bone changes early on, which include periosteal reactions. If her x-ray showed a fracture with no known history of trauma, that would raise concerns of non-accidental injury (B), which would need to be explored further. This is not just cellulitis (C) as there is extensive involvement of the soft tissues and bone on the MRI. A reactive arthritis (D) would not present with a high fever and a systemically unwell child. If the hip were involved this would be a septic arthritis. The history is too short for juvenile idiopathic arthritis (E), which requires a 6-week history with no infection identified, and no MRI periosteal reactions supporting osteomyelitis.

A 4-year-old girl has recently moved to the area and is registering with you, her new GP. She has had a diagnosis of primary ciliary dyskinesia (PCD) made last week and the parents wish to know more about the complications. Which of the following is not a complication of PCD? A. Pancreatic insufficiency B. Infertility C. Sinusitis D. Bronchiectasis E. Dextrocardia

A. Pancreatic insufficiency A) Pancreatic secretions are not reliant on cilia for the expulsion of enzymes and pancreatic fluids. They travel along the pancreatic duct and are joined by biliary secretions in the ampulla of Vater. The respiratory tract (C) and (D) and reproductive organs (B) are both lined by cilia to ensure the movement of particles. Cilia are also necessary for the determination of the sites of the internal organs during development. These children can have dextrocardia alone or situs inversus (E).

A 3 year old is brought to accident and emergency by his parents because he has not been walking for the past day and refuses to stand. He is normally fit and healthy but he did have antibiotics for tonsillitis 2 weeks ago. They do not think he has had any injuries but he attends daycare and something could have happened there. He is up to date with his immunizations and his parents have no concerns with his development. On examination he looks well, is apyrexial, with a heart rate of 120 and respiratory rate of 26 with no bruising. His knees are normal on examination and the hips have a full range of movement except he cries on external rotation of the right hip. There are no deformities seen on x-ray of the hips and knees. After some paracetamol he manages to stand and take a few antalgic steps with encouragement, limping on the right leg. What is the most likely diagnosis? A. Reactive arthritis B. Non-accidental injury C. Growing pains D. Osteomyelitis E. Septic arthritis

A. Reactive arthritis The most likely answer is reactive arthritis (A) which typically follows an upper respiratory tract infection 1-2 weeks later, usually affecting the large weight bearing joints. This is unlikely to be osteomyelitis (D) or septic arthritis (E) because he is well and apyrexial. Having said this, a full blood count and differential and acute phase markers should be checked before concluding this is reactive arthritis. Growing pains (C) are usually more troublesome at night and do not stop children from weight bearing. Non-accidental injury (B) should always be kept in mind; however, he has no physical evidence of injury and the x-ray shows no fractures, making this less likely.

Which of the following is not a feature of a UMN lesion? A. Slow-relaxing Achilles tendon reflex B. Brisk reflexes C. Increased tone D. Decreased power E. Up-going plantar reflex

A. Slow-relaxing Slow-relaxing reflexes may be a sign of a systemic illness such as hypothyroidism or lower motor neuron diseases such as Guillain-Barré syndrome. Increased reflexes (B) and tone (C), decreased power (D) and up-going plantar responses (E) are all features of UMN lesions.

Which of the following is not the correct side effect of anti-epileptic medicine? A. Sodium valproate - aplastic anaemia B. Carbamazepine - visual disturbance C. Lamotrigine - rash D. Vigabatrin - behavioural disturbance E. Levetiracetam - anorexia

A. Sodium valproate - aplastic anaemia Sodium valproate (A) side effects include increased appetite, weight gain, hair loss and liver failure, but not aplastic anaemia. Carbamazepine (B) is associated with lupus erythematosus syndrome, dizziness and visual disturbances. Lamotrigine (C) is associated with rash, behavioural changes and irritability. Vigabatrin (D) side effects include behavioural changes, retinopathy, sleep disturbance and weight gain. Levetiracetam (Keppra) is associated with anorexia, abdominal pain, vomiting, diarrhoea, behavioural changes and thrombocytopenia.

A 3 month old is brought into accident and emergency with a generalized tonic clonic seizure. She is apyrexial and the seizure stopped after 15 minutes with rectal diazepam given by the ambulance crew. Her heart rate is 130, respiratory rate of 36 and capillary refill is less than 2 seconds. On examination she is drowsy, has a port wine stain on her forehead but is otherwise normal on examination. What is the most likely cause of her seizure? A. Sturge-Weber syndrome B. Tuberous sclerosis C. Neurofibromatosis type I D. Meningitis E. Neurofibromatosis type II

A. Sturge-Weber syndrome A port wine stain is a flat purple haemangioma which is present from birth. Any baby born with a port wine stain in the trigeminal region or in the hair should have an MRI brain to look for intracranial haemangiomas as they are at risk of epilepsy. This is called Sturge-Weber syndrome (A). Tuberous sclerosis (B) causes infantile spasms which are brief tonic movements sometimes called 'salaam attacks' as a description of the movement and often mistaken for infant colic. Neurofibromatosis ((C) and (E)) rarely causes seizures so this would not be the most likely cause. A child with meningitis (D) which has caused a seizure would be unwell with fever and possibly evidence of shock, whereas this child has a normal capillary refill, a normal heart rate and is apyrexial.

A 7 year old with a 3-day history of upper respiratory tract infection is brought to accident and emergency by his mother because he suddenly went pale and sweaty and seems to be working hard to breath. The triage nurse calls you to see him urgently because his heart rate is 200 beats per minute. You take him round to the resuscitation area, give him oxygen and connect him to the cardiac monitor. The electrocardiogram (ECG) shows a narrow complex tachycardia with a rate of 180 beats per minute. He remains alert, with a respiratory rate of 40. What is the most appropriate initial diagnosis? A. Supraventricular tachycardia (SVT) B. Wolff-Parkinson-White syndrome C. Ventricular fibrillation D. Atrial fibrillation E. Ventricular tachycardia

A. Supraventricular tachycardia (SVT) The ECG shows a narrow complex tachycardia with a rate of 180 beats per minute. At this rate it is hard to say if P waves are present or not; however this history of a child with an intercurrent illness suddenly becoming unwell with a tachycardia is suggestive of an SVT (A). SVTs may be triggered by Wolff-Parkinson-White syndrome (B), a re-entry tachycardia due to an accessory pathway between the atria and the ventricle. Once the rhythm has slowed down there will be a delta wave visible in all QRS complexes, which is a slanting upstroke of the R wave, and this is associated with a short PR interval. All that can be said of this child is that the QRS complexes are narrow and fast, suggesting a supraventricular origin. Ventricular fibrillation (C) would be characterized by a fast wide irregular complex on ECG, with no discernible P waves. Ventricular tachycardia (E) will be characterized by a regular wide complex tachycardia with no visible P waves. Atrial fibrillation (D) would have the classical saw toothed baseline of P waves with a slower rate of QRS complexes superimposed.

A 14-year-old girl was seen in accident and emergency following her third collapse this year and referred to cardiology for review of a low rumbling murmur heard at the left upper sternal edge. Her ECG in accident and emergency was normal. Her blood sugar was 5.3mmol/L. Urea and electrolytes were normal. The most recent collapse occurred at school while waiting for exam results to be given out. Previously they occurred while watching a parade all afternoon standing in a crowded street, and at a party. On all three occasions she felt dizzy beforehand, was unconscious for less than 10 seconds and fully alert following the episode, but did feel nauseous. Her echocardiogram today is normal. What is the most likely diagnosis? A. Venous hum murmur and vasovagal syncope B. Innocent murmur and epilepsy C. Wolff-Parkinson-White syndrome D. Patent foramen ovale and sick sinus syndrome E. Neurocardiogenic syndrome

A. Venous hum murmur and vasovagal syncope A low rumbling murmur heard above the nipple line with a normal echocardiogram is a venous hum and a normal finding in children. In light of all her cardiovascular investigations being normal, the most likely diagnosis is vasovagal syncope (A), associated with prolonged standing or anxiety. You have not been given an electroencephalography report and there is no suggestion that she was convulsing or post-ictal following her collapses, making epilepsy unlikely (B). Wolff-Parkinson-White (C) syndrome would have delta waves on the ECG, whereas her ECG is normal. A patent foramen ovale would be seen on echocardiogram and sick sinus syndrome would show bradycardia on the ECG (D). Neurocardiogenic syndrome (E) could be diagnosed on a tilt table test, which has not been mentioned, and would not typically have prodromal symptoms.

Positive Ortolani's or Barlow test. (Newborn examination) A Talipes equinovarus B Developmental dysplasia of the hip C Vernix D Cephalhaematoma E Epstein's pearls F Polydactly G Syndactly H Erb's palsy I Klumpke's palsy J Subaponeurotic haematoma

B Developmental dysplasia of the hip Developmental dysplasia of the hip. Ortolani tests if hip is dislocated, and if it is, is it reducible by elevation and abduction. Barlow tests if it is dislocatable if it is not already dislocated by adduction and depression of femur. These tests are only useful in first 6-8 weeks and are performed at the newborn examination and the 6 week check.

A 4-week-old baby boy is brought to the GP with a circumscribed red patch on his back that was not present at birth. (Newborn skin problems) A Port wine stain B 'Strawberry' naevus C Congenital melanocytic naevus D Sebaceous naevus E Mongolian blue spot F Stork mark G Erythema toxicum H Staphylococcal skin infection I Milia J Transient neonatal pustular melanosis

B 'Strawberry' naevus 'Strawberry' Naevus. Very common. Uncommon at birth but usually present within first few weeks. Usually increase in size for first 3-9 months and then involute and resolve completely.

Fixes and follows, smiling, able to raise head when prone. (Development) A Newborn B 2 months C 3 months D 6 months E 9 months F 1 year G 2 years H 3 years I 4 years J 5 years

B 2 months

A 4-year-old boy is reported to have poor concentration in class. EEG reveals 3Hz spikes. (Fits) A 'Breath-holding' attacks B Absence C Infantile spasms (West's syndrome) D Benign rolandic E Juvenile myoclonic F Lennox-Gastaut syndrome G Tonic-clonic H Benign myoclonic epilepsy of infancy I Landau-Kleffner syndrome J Febrile convulsion

B Absence Absence. Characteristic EEG, usually present in first decade as absences.

Neurofibromatosis. (Inheritance) A Autosomal recessive B Automsomal dominant C X-linked recessive D Trinucleotide repeat E Polygenic inheritance F Imprinting G Chromosomal H X-linked dominant I Codominant J Mitochondrial

B Automsomal dominant

A 12-year-old girl presents with a long history of recurrent wheeze and shortness of breath. This is often precipitated by playing with the family cat. A foreign body B asthma C pulmonary haemorrhage D pneumothorax E respiratory distress syndrome F congenital heart disease G meconium aspiration syndrome H pneumonia I croup J respiratory syncitial virus

B asthma Asthma is a chronic inflammatory disorder characterized by hyper- reactive airways leading to episodic reversible bronchoconstriction. There may be a history of atopy/hypersensitivity. Treatment strategies include the use of corticosteroids, long-acting bronchodilators, theophylline, inhaled cromolyn and leukotriene modifiers.

A 7 year old is referred to neurology due to frequent episodes of day-dreaming at school where she is unresponsive. She is falling behind in her work because of this. An electroencephalograph (EEG) shows three spike waves per second activity in all leads. What is the most likely diagnosis? A. Temporal lobe epilepsy B. Absence epilepsy C. Day-dreaming D. Benign Rolandic epilepsy E. Narcolepsy

B. Absence epilepsy This is a typical presentation of absence epilepsy (B). This form of epilepsy often does not affect school performance, but frequent seizures interrupt the directions or information she receives at school and may contribute to her falling behind in her work. The EEG shows three spike waves per second in all leads, which is diagnostic for absence seizures. Temporal lobe epilepsy (A) would show an EEG with seizure activity in the temporal lobes and classically presents with a warning aura or sensation, impaired consciousness or unresponsiveness, and a focal seizure which may spread to become a generalized tonic clonic seizure. In day-dreaming (C), the person is responsive if called. Benign Rolandic epilepsy (D) causes partial seizures which usually occur in the early morning and affect young children, typically resolving in the teens. Narcolepsy (E) is a sleep disorder where affected persons spontaneously fall asleep at inappropriate times.

A 4-year-old boy is brought to accident and emergency with a limp for 1 day. He was unhappy to weight bear on his right leg. He had been with his grandparents all day and his mother brought him to hospital when she returned from work that evening. He was afebrile with a heart rate of 110bpm but had had a cold last week. Mum reports no history of trauma. What is the most important diagnosis to exclude? A. Behavioural B. Acute leukaemia C. Reactive arthritis D. Soft tissue injury E. Septic arthritis

B. Acute leukaemia Acute leukaemia (B) can present with a limp due to bone pain secondary to dissemination of the disease. It is important to do a full blood count (FBC) and blood film on these patients to look for pancytopenia and the presence of blast cells. The other diagnoses are also of importance to investigate for, but may be more or less likely depending on the history. There is no reported history of trauma so (D) is unlikely, but note that he has been with a different carer throughout the day than the one presenting with him now. He may need a plain x-ray of the hip and femur to exclude traumatic injury. The presence of a fever with limp or restricted movement of a joint must raise suspicion of septic arthritis (E), which in the hip is a medical emergency due to potential interruption of blood flow with increasing pressure in the joint. Reactive arthritis (C) can present after a viral illness and may show evidence of inflammation (raised WCC and CRP) but not to the extent as with septic arthritis.

On a newborn baby screening examination, you see a baby girl born by elective caesarean section for breech presentation. This is her mother's first child. The examination is normal except for a clunk felt on Barlow's test and a relocation click on Ortolani's manoeuvre on the right side. What is the next step in management? A. Refer to orthopaedics B. Arrange an ultrasound for 6 weeks of age C. Refer to physiotherapy D. Ask a midwife to put on a plaster hip spica E. Explain to the parents a watch and wait management is most appropriate as most self-resolve

B. Arrange an ultrasound for 6 weeks of age This child has several risk factors for developmental hip dysplasia: female, first child, breech presentation. Other risk factors include clubbed foot, family history of hip dysplasia, oligohydramnios and macrosomia. Barlow's examination of the hips involves pushing the flexed hip backwards testing for dislocation while the Ortolani test starts with the hips flexed and abducted: the examiner's fingers lift the greater trochanters forward in an attempt to relocate a dislocated hip; if a click is felt, the test is positive. The most appropriate management is to request an ultrasound as an outpatient follow-up at 6 weeks of age (B). This is a sensitive test, and waiting until 6 weeks improves the specificity as many unstable hips self-resolve. If the ultrasound is positive then a referral to orthopaedics (A) would be appropriate at that time. First line therapy would be a Pavlik harness which is put on by specially trained physiotherapists (C) and worn for 6 weeks. Midwives are not trained in administering plasters and a hip spica would be put on post-surgery when first line therapy has failed. Many services would ultrasound all babies with breech presentation even without clinical findings to support hip dysplasia. Left untreated this child may walk with a limp, never walk or develop avascular necrosis of the femoral head and therefore should be actively managed. A watch and wait management plan (E) is therefore inappropriate.

A 12-hour-old baby on the post-natal ward has just had a seizure lasting 2 minutes. It resolved spontaneously and was generalized in nature. Her mother had gestational diabetes and poor glucose control in pregnancy. The baby's birth weight was 5kg. There were no abnormalities noted on antenatal US scans or maternal serology. On examination she has no dysmorphic features and handles well. What initial blood tests would you do for the baby? A. Liver function tests B. Boehringer Mannheim (BM) glucose C. Full blood count, C-reactive protein D. Electrolytes E. Calcium, magnesium

B. Boehringer Mannheim (BM) glucose Infants of diabetic mothers are at risk of congenital malformations (if glucose control was poor around conception) and hypoglycaemia and macrosomia in the post-natal period. They become used to having high levels of circulating glucose from their mother and therefore increase their baseline insulin production. At delivery, this supply of glucose is removed and the residual insulin will decrease their glucose, predisposing to hypoglycaemia, so (B) is correct. Fits can occur in babies secondary to infection (C), or electrolyte disturbances (A), (D) and (E) but in this case (B) is more likely given the mother's history, though all should be considered after.

A couple are referred to a geneticist as they are planning on having their first child. There is a history of Wiskott-Aldrich syndrome on the woman's side. The woman's father and great grandfather have the condition (eczema, thrombocytopenia, recurrent infection) but she is unaffected. There is no history of the condition in the man's family. What is the risk of having the condition if the child is a boy or a girl respectively? A. Boy: 1/4; Girl: 1/4 B. Boy: 1/2; Girl: 0 C. Boy: 0; Girl: 0 D. Boy: 0; Girl: 1/2 E. Boy: 1; Girl: 0

B. Boy: 1/2; Girl: 0 Wiskott-Aldrich syndrome is an X-linked condition due to defects in the WASp gene, affecting cell cytosketeton protein actin and associated with the above features. It presents with this triad in approximately one-third of cases, with the other having predominantly haematological complications such as petechiae and bruising. The woman's father has the condition and therefore she is a carrier. The risk of her having a son with the condition is 1/2 and a daughter is 0 (she can only be a carrier) (B). (A) is based on an autosomal recessive condition, with both parents as carriers, and (C) occurs if the mother is a carrier and the father is not affected. (E) is based on the condition being X-linked dominant, but then the mother should have been affected.

Which of the following is not a feature of cardiac insufficiency? A. Scattered wheeze on auscultation of the chest B. Central cyanosis C. Sacral oedema D. Tachypnoea with the apex beat palpable in the 7th intercostal space just lateral to the mid-clavicular line E. Hepatomegaly

B. Central cyanosis All of these may be present in cardiovascular disease. However, while cyanosis (B) may either be due to lung pathology or a right-to-left cardiac shunt, it is not, in itself, a sign of heart failure. Both wheeze (A) or bi-basal crackles are features of heart failure due to pulmonary oedema. Dependent oedema (C) is a feature of heart failure and in an infant who mostly is lying down this will collect on the back as well as the feet. Tachypnoea and a displaced apex beat suggestive of left ventricular hypertrophy are suggestive of cardiac failure (D). Hepatomegaly (E) occurs in heart failure due to back pressure in the venous system resulting in congestion in the portal vein.

A 26 week, premature baby was born by emergency caesarean section due to maternal pre-eclampsia. He required ventilation until age 38 weeks corrected gestation and is still requiring oxygen to maintain his saturations. At 12 months of age he has poor vision and neurodevelopmental function, requires home oxygen and was admitted for a recent respiratory syncytial virus (RSV) bronchiolitis. What is the underlying diagnosis of his respiratory problems? A. Respiratory distress syndrome (hyaline membrane disease) B. Chronic lung disease (bronchopulmonary dysplasia) C. Cystic fibrosis D. Diaphragmatic hernia E. Pulmonary hypoplasia

B. Chronic lung disease Chronic lung disease is by definition an oxygen requirement at 36 weeks corrected gestation or at 28 days post-term (B). It occurs in premature babies as a consequence of barotraumas or volutrauma during the ventilation, surfactant deficiency and oxygen therapy, though the pathophysiology is complex. It can be minimized by using the lowest possible pressure and volume settings to optimize respiratory function in premature infants, but despite this there can be long-term consequences. These children are at risk of respiratory tract infections, particularly RSV for which the preventative paluvizumab monocloncal antibody can be given. (A) Respiratory distress syndrome is due to surfactant deficiency and may be present soon after birth; however, if the clinical manifestations persist, these children may also develop chronic lung disease. (C) Cystic fibrosis does not present in this way, and is more likely to cause meconium ileus or prolonged jaundice in the neonatal period. (D) Diaphragmatic hernia and renal abnormalities can both result in pulmonary hypoplasia (E) and then respiratory difficulties at birth.

A 5-year-old child was admitted overnight awaiting surgical repair of a broken right ankle and was noted to have a raised blood pressure consistently above 130/90mmHg despite adequate analgesia. On examination he has a plaster on his right foot and appears comfortable at rest. On auscultation there is a soft systolic murmur heard at the right upper sternal edge. His femoral pulse is difficult to find, but present bilaterally. When felt with the radial pulse, the impulse in the femoral pulse occurs slightly later. His abdomen is soft and there are no bruits heard. The blood pressure done in the right arm is 136/92mmHg but the left arm gives a reading of 124/80. What is the most likely diagnosis? A. Normal blood pressure in the left arm with a spurious result from the right B. Coarctation of the aorta C. Renal artery stenosis D. Phaeochromocytoma E. White coat hypertension

B. Coarctation of the aorta This child is hypertensive in both arms but there is a discrepancy between the right and left arm with a radio-femoral delay and a systolic murmur heard in the aortic area, supporting a diagnosis of coarctation of the aorta (B). This is likely to be a mild stenosis which is why it was not picked up earlier. The restricted blood flow to the abdomen and kidneys results in feedback loops producing hypertension to maintain perfusion. The blood pressure in the left arm is still high (A) but it is lower than the right arm because of the coarctation restricting blood flow to the left and lower body. There is no bruit heard on examining the abdomen and other findings point to coarctation, making renal artery stenosis (C) unlikely, but to rule it out an abdominal ultrasound with Doppler would be required. Phaeochromocytoma (D) is a rare cause of hypertension caused by an adrenal tumour producing catecholamines and imaging would be required to diagnose this; therefore it is extremely unlikely to be the correct answer. White coat hypertension (E) is raised blood pressure associated with anxiety, induced by the presence of medical professionals. There is an underlying reason found on examination for his hypertension; therefore (E) is not the correct answer.

A 3-week-old baby is brought to the 'prolonged jaundice clinic'. His mother reports he has poor feeding, is not gaining weight appropriately and is more sleepy compared to her previous child. He opens his bowel once a day and is being mix breast and bottle fed. He is floppy, jaundiced, has a large, protruding tongue and a hoarse cry. He had a newborn blood spot screening test done at birth which was normal and he has no dysmorphic features. What is the most likely diagnosis of this child? A. Beckwith-Wiedemann syndrome B. Congenital hypothyroidism C. Down's syndrome D. Normal baby E. Prader-Willi syndrome

B. Congenital hypothyroidism This child has many features of hypothyroidism (B). Children with prolonged jaundice should have their thyroid function tested. The newborn blood spot tests for high thyroid-stimulating hormone (TSH) levels in response to low thyroxine, which should detect most congenital hypothyroidism. Rarely, in children with panhypopituitarism, there is low TSH and therefore low thyroxine. Babies with Down's syndrome (C) can also have hypotonia (floppiness), poor feeding and weight gain, macroglossia and hypothyroidism but with typical facial features which are not present here. Other conditions causing macroglossia include Beckwith-Wiedemann syndrome (A) (other features would include: fetal macrosomia, hypoglycaemia secondary to hyperinsulinaemia, hemihypertrophy, abnormal ear development). Other conditions causing hypotonia and poor feeding include Prader-Willi syndrome (obesity and learning difficulties in later life) (E). This is not a normal baby (D) as jaundice beyond 2 weeks is pathological.

Which of the following is not a feature of an innocent murmur? A. Systolic murmur B. Diastolic murmur C. Asymptomatic D. Heard only at the left sternal edge E. No heaves or thrills

B. Diastolic murmur Characteristics of an innocent murmur include: a soft blowing systolic murmur (A), localized to the left sternal edge (D), with no radiation, and no diastolic component (B) or parasternal thrill (E) and normal heart sounds in an asymptomatic patient (C).

A 2-year-old child is brought to cardiology clinic due to a heart murmur heard by the GP after an examination when she was recently unwell. She was born at 40 weeks by normal vaginal delivery but was noted to have a cleft palate at birth. She was kept in hospital for establishment of feeding but during this time she had a seizure, noted later to be because her calcium was low. You hear a harsh, grade 3/6 pansystolic murmur, loudest at the left lower sternal edge, consistent with a ventral septal defect (VSD) as seen on echocardiogram. With this history and current examination finding, you wish to exclude DiGeorge's syndrome. What is the best diagnostic test? A. Karyotype B. FISH (fluorescence in situ hybridization) C. ELISA (enzyme-linked immunosorbent assay) D. Geneticist review and diagnosis E. Identification of specific mutation

B. FISH (fluorescence in situ hybridization) DiGeorge syndrome is due to a microdeletion on chromosome 22q11.2. It manifests as a collection of features including cleft palate, aortic arch and other cardiac abnormalities, thymic hypoplasia, typical facial features and hypocalcaemia. Microdeletions are best detected with FISH (B). Abnormalities in chromosome number or translocations can be seen with karyotyping (A) and known mutations can be identified (E), but since DiGeorge's syndrome is not due to a specific mutation this is not appropriate. ELISA (C) is used for the identification of known proteins, e.g. autoantibodies, bacterial/viral detection. A geneticist review (D) may be helpful, particularly for planning of further pregnancies.

A 10-month-old baby is brought to accident and emergency by ambulance having had a seizure. His mother reports that he went floppy suddenly and then his right arm and leg started shaking and he was not crying. It lasted less than 5 minutes and he was sleepy afterwards. He has had a fever and runny nose for the last 2 days and is off his food. Why is this not a febrile seizure? A. He is too young B. He has had a focal seizure C. He has recently had a viral illness D. The seizure lasted too long E. The fever was not high enough

B. He has had a focal seizure Febrile seizures are common in children aged 6 months to 6 years, hence (A) is incorrect. The aetiology is unknown, but it is thought to occur with the rapid rise in temperature at the start of an infective illness, most commonly a viral illness in children, (C) is incorrect. There is no recognized range of temperatures, though there must be a documented fever, i.e. >37.5°C; therefore (E) is incorrect. A febrile convulsion by definition must be a generalized seizure that occurs in association with a fever, in a child with no neurological abnormality, hence (B) is the correct answer. Focal seizures are not categorized as febrile fits and this child requires imaging of the brain and to commence antibiotics (to cover Streptococcus pneumoniae and Neisseria meningitidis) and antivirals (to cover HSV) as this seizure may be secondary to meningoencephalitis. Seizures are categorized into simple (<5 minutes, self-resolving) or complex (prolonged >15 minutes, requiring medical treatment) but must fulfil the other criteria above; therefore (D) is incorrect.

You are asked to examine a tall 15-year-old boy. His height is above the 98th centile for his age and he has other concerns about the development of breast tissue. He was told this was normal as he develops through puberty but his father states he has no facial or underarm hair. Jake allows a brief examination of his genitalia and you note he has a small penis and testicular volume. He has no arachnodactyly or visual problems. What is the most likely diagnosis? A. Delayed onset of puberty B. Klinefelter's syndrome C. Precocious puberty D. Marfan's syndrome E. Normal variation

B. Klinefelter's syndrome Both Klinefelter's and Marfan's syndromes (D) can result in tall stature (though associated features with Marfan's syndrome include high arched palate, myopia, lens dislocation, arachnodactly, arm span > height, hypermobility, aortic arch abnormalities, mitral valve prolapse, chest wall deformity). Precocious puberty (C) leads to tall stature initially but ultimate height is often shorter than peers as the growth spurt occurs earlier. He also has signs of delayed puberty with small testicular volume (<4mL) and no adrenarche. Delayed puberty (A) (in boys no pubertal development by the age of 15 years) alone would usually be associated with short rather than tall stature. The tall stature, hypogonadism and delayed puberty with gynaecomastia is suggestive of Klinefelter's syndrome (B) and a karyotype should be done, showing 47XXY. This is not a normal variation (E) as failure to develop signs of puberty by the age of 15 years in boys requires investigation.

A 3-year-old is brought into accident and emergency on a Monday morning because she has developed several bruises on her buttocks, left leg and right arm. She is seen with her nanny who reports finding the bruises when she was getting her dressed this morning. Recently the girl has not been herself. She has had several colds over the past 2 months and has been more lethargic lately. The nanny is worried she is losing weight. On examination she appears withdrawn, pale and has a bruise on the left buttock which is 5cm × 8cm. She has three other bruises on her left leg and right arm which are of varying colours. She also has some fine petechiae on her neck and cheeks. She has a runny nose and a cough but the chest is clear. What is the most likely diagnosis? A. Non-accidental injury B. Leukaemia C. Idiopathic thombocytopenia D. Henoch-Schönlein purpura E. Accidental injury

B. Leukaemia This child is presenting with bruising on the buttocks and limbs as well as petechiae which raise alarm bells that the child may have low platelets. The two options on the list with low platelets are idiopathic thrombocytopenia (ITP) (C) or leukaemia (B). This child has a background of being unwell with a suggestion of weight loss which points to leukaemia as the most likely diagnosis (B). Henoch Schönlein purpura (D) does not produce low platelets but could produce the bruising on the buttocks: this would normally be associated with joint and abdominal pains as well as haematuria. It is a small vessel vasculitis which typically occurs 1-2 weeks after a throat infection. ITP (C) also is often triggered after a viral infection and it would be unusual for the child to be unwell at presentation with the bruises. If the full blood count comes back normal that would raise concerns of non-accidental injury (A). The petechiae in the head and neck could be produced by strangulation and the bruises of multiple ages as well as sited on the buttocks is unusual for an accidental injury (E); combined with a withdrawn child this should raise the suspicion of non-accidental injury (A). As you have not been given the full blood count results and the child is unwell with weight loss and recurrent illnesses, the single best answer is leukaemia (B).

A 3-year-old boy is admitted to the children's ward. He has been isolated in a cubicle as he is at risk of infections. He is awaiting a bone marrow transplant and has a brother with the same condition. His mother tells you they both have SCID. What are the likely immune function test results in SCID? A. Normal B cells, normal T cells, normal immunoglobulins B. Low B cells, low T cells, low immunoglobulins C. Normal B cells, normal T cells, high immunoglobulin M subsets D. Low B cells, normal T cells, low immunoglobulins E. Normal B cells, low T cells, normal immunoglobulins

B. Low B cells, low T cells, low immunoglobulins SCID is a group of disorders caused by B and T cell dysfunction. SCID affects both B and T lymphocyte cell lines; therefore only (B) can be correct. Low B cell numbers result in reduced immunoglobulin production. (A) would be the result in an immunocompetent host. (C) may be due to hyper IgM syndrome, which is due to a defect in the CD40 ligand, resulting in defective class switching and T cell function and recurrent infections. (D) may be suggestive of Bruton's agammaglobulinaemia, where lack of B cells results in low immunoglobulins and predisposition to respiratory and central nervous system (CNS) infections. (E) may result from thymic hypoplasia or absence as the T cells alone are affected with normal B cells and immunoglobulins; this may be associated with DiGeorge's syndrome.

On a newborn baby check of an Asian, 36-hour-old baby you note a large bruise coloured area on the buttocks and lower back which seems non-tender. The mother does not know how it got there. He is handling well and the rest of the baby check is unremarkable. What is the most likely explanation? A. Non-accidental injury B. Mongolian blue spot C. Neonatal sepsis with disseminated intravascular coagulation D. Idiopathic thrombocytopenic purpura E. von Willebrand's disease

B. Mongolian blue spot This is a Mongolian blue spot (B) and is more commonly found in darker skinned races. Mongolian blue spots are classically found on the buttocks and lower back and fade as the child grows up. It is not bothering the baby so it is unlikely to be an injury (A). The child is well on examination which makes sepsis with disseminated intravascular coagulation highly unlikely (C). Idiopathic thrombocytopenic purpura (D) is not seen in newborns as it requires a preceding infection to trigger the immune response, and maternal antibodies are protecting the baby. However, maternal autoantibodies against platelets could produce thrombocytopenia and purpura but this is not an answer choice. Von Willebrand's disease (E) is a clotting defect controlling the binding of platelets to damaged endothelium and typically presents with mucosal bleeding or menorrhagia.

A 15-year-old girl comes to accident and emergency complaining of sudden right arm weakness and double vision. Last week she was incontinent of urine twice. She is normally fit and well. On examination she has a left-sided 6th nerve palsy and four out of five power in her right arm. The examination is otherwise unremarkable. An MRI head shows multiple hyperintense, inflammatory, white matter lesions. What is the most likely diagnosis? A. Brain metastasis B. Multiple sclerosis C. Tuberous sclerosis D. Tuberculous meningitis E. Neurofibromatosis

B. Multiple sclerosis She has presented with neurology consistent with multiple lesions. All of the options may cause more than one lesion in the brain but multiple sclerosis (MS) (B) classically affects the white matter with demyelinating lesions which are inflamed, as seen in this case. Brain metastasis (A) and tuberculosis (D) would typically cause space occupying lesions with surrounding oedema. In tuberous sclerosis (C), the MRI typically shows subependymal calcifications and hypointense white matter lesions or tubers. Neurofibromatosis (E) may cause gliomas or acoustic neuromas which would be seen as a space occupying lesion. The MRI taken together with a history of intermittent and sudden changes in neurological function, especially vision and urinary incontinence, point towards the diagnosis of MS.

You are in immunology clinic and the first patient is a 2-year-old boy who has a complement deficiency. You know this involves a cascade of proteins involved with innate immunity but are unsure about the manifestations in children. The professor of immunology asks you which organism is this child at risk of being infected with. He gives you a clue by telling you the child has a late complement deficiency, meaning C5-C9. What is the most likely causative organism that infects these children? A. Streptococcus pneumoniae B. Neisseria meningitidis C. Haemophilus influenzae D. Mycobacterium tuberculosis E. Pneumocystis jiroveci

B. Neisseria meningitidis The complement pathway is important in detecting and eradicating these organisms early on and, without effective complement systems, overwhelming infection may develop. The complement cascade involves a series of proteins which can opsonize and create and insert a membrane attack complex into the organism, which aims to eradicate it. Deficiencies along the pathway have been noted: protein C3 resulting in recurrent pyogenic infection, protein C1, C2 and C4 associated with autoimmunity and proteins C5-9 with a risk of getting infections with N. meningitidis (B). Children with sickle cell disease (functional asplenia) or other children without a functioning spleen are at risk of infection with encapsulated organisms such as (A) or (C). Patients who are immunocompromised (secondary to HIV or immunosuppressive therapy, e.g. steroids or chemotherapy), are at risk of (D) or (E).

A 6-year-old boy is registering with a new GP, having just moved to the area. He is in a wheelchair but is able to mobilize with a fast scissoring gait over short distances. He has increased tone in his legs and has scars from previous tendon release surgeries. His upper limbs are normal. His mother says that his school performance is good and he is writing well. She thinks he was going to have a Statement of Special Educational Needs assessment before they moved. As the GP, what is the most appropriate next step in management? A. Reassure his mother that as he is doing well at school he does not need a statement B. Refer to a community paediatrician C. Refer to the physiotherapists and occupational therapists D. Liaise with his new school teacher to make sure the school is able to support his physical needs E. Refer to an educational psychologist

B. Refer to a community paediatrician This child has diplegic cerebral palsy and needs multidisciplinary support to manage his care. He should have a formal statement to create a plan of what physical support he will need for school, so (A) is wrong. The community paediatrician (B) is best placed to coordinate this process, although assessments by physiotherapists and occupational therapists (C), liaison with the school (D), and educational psychology (E) reports will be needed. In addition he will need to be referred to orthopaedic specialists and possibly Social Services to assess eligibility for grants to adapt the home.

A 3-year-old boy was brought to accident and emergency with his mother. She says he has been limping for a day now and refusing to walk for the last 2 hours. He has had a fever to 39°C which can be brought down with paracetamol. He has had no vomiting, diarrhoea, rash, cough, coryza or injury. He lives with his mother and is her only child. She is currently unemployed and has a background of depression. On examination of the right leg he has a swollen thigh and cries inconsolably when it is touched. It is red and tender. He refuses to allow movement of the hip either passive or active. The left leg is unremarkable on examination. What is the most likely diagnosis? A. Perthes' disease B. Septic arthritis C. Fractured femur due to accidental injury D. Juvenile idiopathic arthritis E. Fractured femur due to non-accidental injury

B. Septic arthritis This child has a fever and a limp, with limited movement and signs of acute inflammation in his right lower limb. The most likely diagnosis is septic arthritis of the right hip (B). He is too young for (A) which is due to avascular necrosis of the femoral head which usually occurs in children over 5 years of age. There is no history of trauma and therefore this makes (C) unlikely. (E) must remain a consideration, in particular in view of the social background. This history does not fit the definition of juvenile idiopathic arthritis (D), which is 6 weeks of joint pain and swelling which persists after other diagnoses have been excluded.

On the day 1 baby check a mother is very concerned about a rash on her baby's face. Over the right eye, forehead and temple there is a pink-red, flat area of erythema. He is opening the eye, and his eye movements seem intact. The child's observations and rest of the examination are normal. What should you tell the mother? A. This is a strawberry naevus and it may get bigger before it goes away by about 5 years of age B. This is a port wine stain and the baby needs an MRI scan to check for intracranial involvement C. This is a capillary haemangioma (stork mark) and is normal; it will fade over the first year or so of life D. This is orbital cellulitis and he needs intravenous antibiotics E. This is erythema toxicum which is a normal baby rash and will go away within the first few weeks

B. This is a port wine stain and the baby needs an MRI scan to check for intracranial involvement This is likely to be a port wine stain or naevus flammeus (B) which is a capillary vascular malformation in the dermis, present from birth and will persist for life. When present in the trigeminal distribution, a small proportion of children will have underlying brain involvement (Sturge-Weber syndrome) and should have an MRI brain to look for this as the child will be at risk of seizures and developmental problems. A strawberry naevus (A) or cavernous haemangioma is not usually present at birth and typically appears in the first month. Strawberry naevi will grow larger before shrinking and disappearing, typically before the age of 5 years. They may compress neighbouring structures as they grow and sometimes ulcerate with troublesome bleeding. They are not flat. A capillary haemangioma or 'stork bite' (C) is a pink macule found on the eyelids, central forehead or nape of the neck which is due to distension of the dermal capillaries; they mostly fade over the first year. Those on the neck may persist but will be covered by hair. They are not found on the lateral face; therefore this is not the right answer. The child is well, with normal observations, making cellulitis (D) very unlikely. In addition you have been given no information on whether the lesion feels warm, which you would expect in infection. Erythema toxicum (E) is a common innocent rash of the newborn which has an erythematous base with small pustules. It comes and goes all over the body for the first few weeks of life.

A 13-year-old Somali girl presents to accident and emergency with a 1-month history of headaches, weight loss and night sweats. Her father is concerned that she seems confused and is more unwell with her headache despite paracetamol. She was born in the UK and has had all her immunizations. She travelled to Somalia 6 months ago. The rest of the family is well although dad has a cough. On examination she is thin and looks unwell but is neurologically intact with no abnormal findings on clinical examination. Which diagnosis needs to be ruled out first? A. Brain tumour B. Tuberculous meningitis C. Pulmonary tuberculosis D. Migraines E. HIV infection

B. Tuberculous meningitis A brain tumour (A) is unlikely with a normal neurological examination. She has weight loss and night sweats which should raise a suspicion of tuberculosis (TB), which is particularly common in the Somali community. There is no specific history of cough and more concerning is her headache and confusion, making the most likely diagnosis tuberculous meningitis (B). She should have a lumbar puncture, provided there are no signs of raised ICP. A chest x-ray would be performed routinely in this case to look for pulmonary TB (C). While migraines (D) may be very troublesome and frequent, they do not cause weight loss and night sweats and investigations for lymphoma or tuberculosis should always be carried out when these symptoms are present. Patients diagnosed with TB should be screened for HIV (E) co-infection, but this does not need to be done immediately as it will not change the initial management.

What is the most common congenital heart defect? A. Coarctation of the aorta B. Ventricular septal defect (VSD) C. Atrial septal defect D. Patent ductus arteriosus E. Transposition of the great arteries

B. Ventricular septal defect (VSD) One-third of all congenital heart defects are VSDs (B). All others are much less common. The second most commonly seen is the patent ductus arteriosis (D) which makes up just over 10 per cent. Atrial septal defects (C), coarctation of the aorta (A) and transposition of the great arteries (E) each make up approximately 5 per cent.

A 2-year-old boy presents to A&E with a sudden onset of fever and sore throat. On examination he is drooling, sitting upright and has a fever of 40°C. (Treatment of respiratory distress) A Palivizumab B Dexamethasone C Cefotaxime D Erythromycin E Amoxicillin F Nebulized salbutamol G Salbutamol via inhaler H Nebulized adrenaline I IM adrenaline J IV adrenaline

C Cefotaxime Cefotaxime. Epiglottitis: this is a medical emergency.

A 7-month-old baby boy presents to the GP with 1-month history of diarrhoea. The mother has noticed that his stools have become pale. He has a reduced appetite and his weight has fallen off his growth centile. (Diarrhoea) A Toddler's diarrhoea B Hirschsprung's disease C Coeliac disease D Inflammatory bowel disease E Intussusception F Haemolytic-uraemic syndrome G Henoch-Schönlein purpura (HSP) H Necrotizing enterocolitis (NEC) I Infective gastroenteritis J Food hypersensitivity

C Coeliac disease Coeliac disease. Enteropathy due to lifelong intolerance of gluten protein. Presents at any age after weaning onto solids with diarrhoea, pale and bulky stools, failure to thrive, anorexia and irritability.

A 5-month-old baby with developmental delay presents with sudden repetitive jerking movements while awake. EEG shows hypsarrhythmia. (Fits) A 'Breath-holding' attacks B Absence C Infantile spasms (West's syndrome) D Benign rolandic E Juvenile myoclonic F Lennox-Gastaut syndrome G Tonic-clonic H Benign myoclonic epilepsy of infancy I Landau-Kleffner syndrome J Febrile convulsion

C Infantile spasms (West's syndrome) Infantile Spasms describe short tonic contractions of the trunk and upward elevation of the arms. These can often be confused with gastro-oesophageal reflux or colic.

Sweating, pallor, hypertension. (Tumours) A Acute lymphoblastic leukaemia B Lymphoma C Neuroblastoma D Wilm's tumour E Retinoblastoma F Rhabdomyosarcoma G Osteosarcoma H Ewing's sarcoma I Acute myeloid leukaemia J Glioma

C Neuroblastoma Neuroblastoma. A malignant embryonal tumour derived from neural crest tissue. Presentation is non-specific and variable, and depends on site, spread and metabolic effects. If it involves the sympathetic chain then it is associated with these symptoms.

A 6-day-old baby girl, born by normal vaginal delivery, presents with a 24-hour history of poor feeding and lethargy. On examination she has a CRT of 4 seconds, RR 80, and HR 180bpm. Blood tests show CRP 100 and blood glucose 1.6mmol/L. (Collapsed neonate) A Coarctation of the aorta B Non-accidental injury C Sepsis D Inborn error of metabolism E Neonatal SVT F Neonatal abstinence syndrome G Pulmonary stenosis H TGA I Persistent pulmonary hypertension J AVSD

C Sepsis Sepsis. With this presentation it is important to consider congenital heart disease and sepsis. In this case the most likely diagnosis is sepsis with a CRP of 100 and blood sugar of 1.6. If this is Group B Streptococcus then this is late onset and therefore would be horizontally acquired and therefore not affected by intrapartum antibiotics.

An 8-year-old boy presents to A&E with a sudden onset of hot and swollen right knee. On examination he is unable to bear weight, has reduced range of movement and has pain on passive motion in this knee. He is febrile and has raised ESR, WCC and CRP. (Limping child) A Developmental dysplasia of hip B Transient synovitis C Septic arthritis D Perthes' disease E Osteomyelitis F Slipped upper femoral epiphysis G Osgood-Schlatter's disease H Juvenile idiopathic arthritis I Non-accidental injury J Duchenne's muscular dystrophy

C Septic arthritis Septic arthritis presents with an acute onset, decreased range of movement, pain on passive movement, a hot, swollen joint and systemic symptoms of infection.

White substance on skin at birth. (Newborn examination) A Talipes equinovarus B Developmental dysplasia of the hip C Vernix D Cephalhaematoma E Epstein's pearls F Polydactly G Syndactly H Erb's palsy I Klumpke's palsy J Subaponeurotic haematoma

C Vernix Vernix is the benign slough of sebum and skin.

Duchenne muscular dystrophy. (Inheritance) A Autosomal recessive B Automsomal dominant C X-linked recessive D Trinucleotide repeat E Polygenic inheritance F Imprinting G Chromosomal H X-linked dominant I Codominant J Mitochondrial

C X-linked recessive

A 2 year old is brought in by ambulance after pulling a pot of boiling water off the stove down on top of himself. He has significant burns to the whole of his face, torso and right arm. Estimate the percentage body surface area affected. A. 20 per cent B. 30 per cent C. 40 per cent D. 50 per cent E. 60 per cent

C. 40 per cent In children, the head represents a larger proportion of the body and the Lund-Browder chart is used to assess the burn percentage. The whole head would account for 18 per cent. This child has burned the anterior head = 9 per cent, the anterior torso = 18 per cent and the whole right arm = 9 per cent, which is not quite 40 per cent (C). In an adult, the whole head represents only 9 per cent of the body and therefore this same burn would be closer to 30 per cent (B). Just the torso would be about 20 per cent (A). Burns of 50 (D) or 60 per cent (E) are extremely extensive.

Ninety-nine per cent of healthy term infants will pass meconium within the first 24 hours of life and all should do so within 48 hours. Which of the following is not a cause of delayed meconium passage? A. Hirschsprung's disease B. Cystic fibrosis with a meconium ileus C. Choanal atresia D. Imperforate anus E. Meconium plug syndrome

C. Choanal atresia Choanal atresia (C) is a congenital blockage of the nasal airway which presents with newborn cyanosis and respiratory distress, as infants are obligate nose breathers. The most common cause of delayed meconium passage is (E), which is a transient immaturity of the gut resulting in failure to move a plug of meconium along. The management is with anal stimulation with a glycerine chip. Approximately 20 per cent of cystic fibrosis cases will present with a meconium ileus (B) due to inspissated meconium. Approximately one in 4000 babies will have Hirschprung's disease (A) which is caused by the absence of ganglion cells from the myenteric and submucosal plexi in part of the rectum or colon which results in narrowed bowel in the denervated region. The lack of bowel wall movement fails to move the meconium along. The management is surgical resection of the denervated portion of bowel. An imperforate anus (D) is slightly less common than Hirschprung's disease, and will require surgical intervention.

A 3-week-old baby attends accident and emergency with bloody diarrhoea. Mum says he has been having diarrhoea for the past 2 days since she started using formula milk. He was previously breastfed and mum was not having any dairy products due to lactose intolerance. He also has eczema on his cheeks and a strong family history or asthma and eczema. Mum is concerned that he may be allergic to milk too. What is the most likely diagnosis? A. Lactose intolerance B. Gastroenteritis C. Cow's milk protein intolerance D. Fructose intolerance E. Galactosaemia

C. Cow's milk protein intolerance Cow's milk protein intolerance (C) is an allergic reaction secondary to allergens in cow's milk. Babies may present with diarrhoea on introduction of cow's milk based formula or breast milk if the mother takes dairy products. Cow's milk protein intolerance may also be associated with eczema, commonly on the face. Lactose intolerance (A) can present similarly, but the congenital form is extremely rare (due to absolute lactase deficiency at the brush border) though lactase expression starts to reduce from 2 years of age in most people. Fructose intolerance (D) is a rare metabolic condition presenting with vomiting, hypoglycaemia, failure to thrive, hepatomegaly, jaundice, renal complications and severe metabolic acidosis. Gastroenteritis (B) should be considered, but in the presence of vomiting and fever which are absent here. Galactosaemia (E) is due to galactose-1-phosphate uridyl transferase deficiency, which can present with metabolic disturbances, sepsis, vomiting and collapse.

A 2-year-old girl was brought by her mother to accident and emergency after 4 days of vomiting and abdominal pain. She had brought her in 2 days ago after developing a cold and was discharged home and diagnosed with a 'tummy bug'. On examination, she was drowsy, had dry mucous membranes, deep heavy breathing, cool peripheries and tachycardia. Her mother reports a 1-month history of weight loss, excessive drinking and passing large volumes of urine prior to this episode. Her urine dipstick has ketones and glucose. Her blood gas shows the following: pH 7.10, PCO2 kPa 2.1, PO2 kPa 10.0, BE −12, HCO-3 mmol/L 18. What is the most likely diagnosis? A. Severe dehydration secondary to gastroenteritis B. Sepsis secondary to gastroenteritis C. Diabetic ketoacidosis (DKA) D. Chronic kidney disease E. Hyperosmolar hyperglycaemic non-ketotic state

C. Diabetic ketoacidosis (DKA) This girl has the classical history of developing polyuria and polydipsia with weight loss, now with a viral illness precipitating DKA (C). Abdominal pain and vomiting are common in DKA and the disease process is triggered by insulin deficiency leading to lipolysis and ketone production. This results in the acidosis along with glycosuria osmotically increasing water losses in urine and leads to dehydration, worsening the acidosis. She will likely have high blood sugar which should also be measured in the laboratory as the bedside tests are inaccurate at very high levels. Gastroenteritis ((A) and (B)) can cause both profound dehydration and sepsis (with bacteraemia or viraemia), with ketonuria but there should not be glycosuria. Children with polyuria should have renal function checked (D), but in this setting with acute illness and ketonuria and glycosuria suggestive of DKA, diabetes is a better explanation of polyuria. Hyperosmolar hyperglycaemic non-ketotic state (E) occurs as a complication of type 2 diabetes, which is rare in childhood and does not present with ketonuria, though the incidence is increasing with childhood obesity.

An 11-year-old girl was brought to accident and emergency in December with pain in her left leg. She is known to have sickle cell disease and her baseline haemoglobin is 7.0 g/dL. She has been admitted in the past with painful leg and chest crises. She has a cough and coryza. Today her blood results show: haemoglobin 6.8, white cell count (WCC) 12×109/L, platelets 209×109/L, C-reactive protein (CRP) 20 mg/L. What is not part of the appropriate initial management? A. IV fluids B. 15L oxygen through a non-rebreather mask C. Exchange transfusion D. IV antibiotics E. Oramorph

C. Exchange transfusion Sickle cell anaemia. This child has a painful crisis caused by vaso-occlusion of small blood vessels supplying the bones in her leg. This leads to pain, which can require analgesia with morphine (Oramorph or patient controlled pumps). Painful crises may be triggered by cold weather, infection, hypoxia or dehydration. Therefore, the initial management of painful crises includes rectifying these triggers with analgesia (E) (opiates may be required), rewarming, antibiotics (D), oxygen (B) and IV fluids (A). If there is a marked drop in haemoglobin, top up transfusion may be required. Exchange transfusion is indicated for sickle chest crisis, stroke and priapism.

A 3 year old is brought into accident and emergency by ambulance following a generalized tonic clonic seizure that lasted 2 minutes. She did not require any treatment to stop the seizure but on arrival the ambulance crew measured her temperature as 39.2°C and gave paracetamol. She is now apyrexial with a heart rate of 140, respiratory rate of 30 and capillary refill less than 2 seconds. On examination she has red enlarged tonsils with no pus, no neck stiffness or rash. What is the most appropriate management? A. Oral penicillin B. Lumbar puncture and IV ceftriaxone C. Explain that this was a febrile convulsion and discharge home D. Start phenytoin E. Discharge home with rescue buccal midazolam for future seizures

C. Explain that this was a febrile convulsion and discharge home This is a typical febrile convulsion which are associated with rapid rises in temperature. The child is now stable and likely has a viral tonsillitis as the focus of infection. Parents need reassurance and explanation that febrile convulsions are not usually associated with later epilepsy but may recur in future febrile illnesses. Typically, children do not continue to have febrile convulsions beyond the age of 5 years. Management should be regular anti- pyretics and dress the child lightly during febrile illnesses. Oral penicillin (A) should be prescribed for bacterial tonsillitis, either with pus or if the child is unwell clinically. IV ceftriaxone and lumbar puncture (B) would be for suspected meningitis but this 3-year-old child has a clear focus for the fever, no neck stiffness or rash making this unlikely. In children having their first febrile convulsion under the age of 12 months, it is mandatory to rule out meningitis. Phenytoin (D) would be used for status epilepticus (seizure lasting longer than 30 minutes). Buccal midazolam (E) is used in the community for children who have seizures lasting longer than 5 minutes; febrile convulsions are usually short and unless a child has had a prolonged seizure they would not be sent home with rescue medication.

A 2-month-old baby is brought in by the babysitter because he has been crying since she arrived to look after him and his right leg looks swollen. He is the only child living in the household. She does not think he is moving it and is worried it is injured. On examination he is miserable, his heart rate is 160, respiratory rate of 56, and capillary refill is less than 2 seconds. He has a swollen right thigh. He cries more when that leg is examined. You note a yellow bruise on his left thigh and two purple bruises on either arm. X-rays show a fracture of the right femur but the arms appear intact. A chest x-ray shows three healing posterior rib fractures. You are highly suspicious of non-accidental injury. What is the most appropriate management? A. Give analgesia and plaster the leg fracture. Ask the babysitter to bring him back with the parents because he needs to be admitted B. Give analgesia. Call the duty social worker on-call to get permission to discharge him once his leg has been plastered C. Give analgesia and plaster the leg fracture. Contact the parents and inform them that he needs to be admitted. Ask them to come to the hospital and inform social services once they have arrived and been updated D. Give analgesia and plaster the leg fracture. Call the police to bring the parents to hospital E. Give analgesia and plaster the leg fracture. Call the police to arrest the babysitter for child abuse

C. Give analgesia and plaster the leg fracture. Contact the parents and inform them that he needs to be admitted. Ask them to come to the hospital and inform social services once they have arrived and been updated The most appropriate action is to bring the parents to the hospital to take a full history (C). The child's safety is paramount and he should not be discharged until a full safety assessment is made and all of his injuries have been assessed and treated. Once the parents are there and have been updated it is then important to inform them of the social services referral, but they cannot refuse it. The child must be kept in a place of safety so discharge is not an option (A). The parents should be informed prior to social services referral (B) unless there are other children in the community that may also need protection, in which case, social services need to be informed and will liaise with the police to bring those children in to a place of safety. It would be inflammatory and counter-productive to bring the parents into hospital by police, although the police will be involved as part of child-safeguarding enquiries (D). There are injuries of several different ages on this child, different colour bruises and healing rib fractures making it unlikely that the babysitter is at fault (E), although it is important to keep an open mind regarding potential perpetrators. An accurate, well-documented history from each person who has cared for the child is crucial.

You are doing a baby check on the post-natal ward on a baby who is 23 hours old. His mother tells you that he is not feeding well. On examination he is unsettled with a respiratory rate of 76 and a heart rate of 182. You think his hands and feet look blue and there is a soft systolic murmur heard at the left upper sternal border. You ask the midwives to check his saturations which are 85 per cent in air and start some oxygen. You explain to the mother that he needs to be managed on the neonatal unit. What is the next step in your management? A. Stop the oxygen as this may drive the closure of the ductus arterioles B. Give prostaglandin intravenously to open the duct while organizing an echocardiogram C. Give antibiotics and prostaglandin intravenously while organizing an echocardiogram D. Give indomethacin intravenously to open the duct while organizing an echocardiogram E. Give indomethacin and antibiotics intravenously while organizing an echocardiogram

C. Give antibiotics and prostaglandin intravenously while organizing an echocardiogram The correct answer is to cover the baby for sepsis with antibiotics and to maintain the patency of the ductus arteriosus while organizing an echocardiogram to elucidate the underlying cardiac anatomy (C). A blue baby with a murmur should be treated as a duct-dependent anomaly until proven otherwise. A septic baby could be mottled and present with a flow murmur and until the echocardiogram has been performed it is difficult to rule out sepsis; thus not covering with antibiotics would be wrong (B). You should always administer oxygen to a cyanotic child (A) unless specifically told not to by the cardiology team, and in this situation there would be clear guidelines to maintain the saturations between an upper and lower limit, typically in the 80s. Indomethacin is wrong ((D) and (E)); this is used to close the duct in children who have a persistent ductus arteriosus resulting in significant respiratory distress or impaired systemic oxygen delivery.

A 1-day-old baby is on the post-natal ward. You are asked to review her as she is febrile and lethargic. On examination she is tachycardic, has a capillary refill time of 3 seconds centrally and reduced urine output. Her blood culture 24 hours later grows Gram-positive cocci. Which is the most likely causative organism? A. Streptococcus pneumoniae B. Staphylococcus aureus C. Group B Streptococcus D. Streptococcus viridans E. Group A Streptococcus

C. Group B Streptococcus Group B Streptococcus (GBS) (C) can colonize the reproductive tract of women. When babies are delivered through this tract, they can become infected with GBS. This may manifest as sepsis, pneumonia, meningitis, urinary tract infection and septic arthritis among others. All others are possible but in this demographic, the three most common pathogens are group B Streptococcus, E. coli and Listeria monocytogenes.

An 18-month-old boy presented to the GP with a history of eating soil. He had been in the garden this afternoon as his mother put the washing out. She found him eating the soil and took him straight inside. On examination, he is well and alert but has pale conjunctivae. He is not tachycardic or tachypnoeic. His diet consists of predominantly of breast milk. What is the most likely result of his haemoglobin and haematinics? A. Hb 10 g/dL, MCV 80 fl, ferritin normal, iron normal, vitamin B12 and folate normal B. Hb 6.5 g/dL, MCV 100 fl, ferritin normal, iron normal, vitamin B12 and folate low C. Hb 5.5 g/dL, MCV 55 fl, ferritin low, iron low, vitamin B12 and folate normal D. Hb 7 g/dL, MCV 70 fl, ferritin normal, iron normal, vitamin B12 and folate normal E. Hb 6.8 g/dL, MCV 65 fl, ferritin normal, iron low, vitamin B12 and folate normal

C. Hb 5.5 g/dL, MCV 55 fl, ferritin low, iron low, vitamin B12 and folate normal This child is exhibiting pica, a phenomenon of eating substances that are not food, secondary to severe iron deficiency anaemia consistent with microcytic anaemia with low iron stores (ferritin) as well as low circulating iron (C). The most likely cause is due to prolonged breastfeeding and delayed weaning. Children have sufficient iron stores for 4 months after birth. After this time they require supplementation which usually comes from the introduction of pureed and solid foods. Delay in weaning can lead to iron deficiency anaemia, hypocalcaemia and poor weight gain. (A) shows a normal study, with no evidence of anaemia and normal haematinics. (B) shows a macrocytic anaemia with vitamin B12 and folate deficiency (megaloblastic anaemia). (D) shows a microcytic anaemia with normal iron, suggesting another cause such as thalassaemia or sideroblastic anaemia. (E) is a macrocytic anaemia with normal vitamin B12 and folate, suggesting another cause such as hypothyroidism, liver disease, myeloproliferative disorders.

Which of the following is not a cause of erythema nodosum? A. Oral contraception B. Tuberculosis infection C. Hepatitis B infection D. Streptococcal infection E. Sarcoidosis

C. Hepatitis B infection The oral contraceptive pill (A), tuberculosis (B), streptococcal infections (D) and sarcoidosis (E) are all known causes of erythema nodosum, which produces tender red nodules on the shins. Hepatitis B (C) does not cause erythema nodosum. It may produce non-specific rashes and jaundice.

A 6-year-old girl is taken to see her GP because she is complaining of knee and elbow pains frequently. Her mother thinks it is worst after her ballet classes and when she gets home from school. She denies stiffness or pain in the mornings. Her mother has been administrating paracetamol several times a week and is worried that this is too much to be giving a child. On examination, the child looks well and has full range of movement of her joints with evidence of hyperextension. There are no swollen joints or effusions present and she is non-tender on examination. What is the most likely diagnosis? A. Repetitive strain injury B. Marfan's syndrome C. Hypermobile joints D. Osteoarthritis E. Juvenile idiopathic arthritis (JIA)

C. Hypermobile joints Her joint pain does not stop her from activities and she has normal joint assessment except for hyperextension of the joints, making the most likely cause of her joint pain being simply hypermobile joints (C). Sometimes physiotherapy may be helpful to strengthen the muscles supporting joints. While Marfan's syndrome (B) may feature hypermobile joints, it is not the most prominent feature and tall stature, high arched palate and long limbs should be present to suggest this as a possible diagnosis. There is no history to support repetitive strain injury (A) and a child's ballet class should not cause this. There are no swollen joints on examination to support a diagnosis of osteoarthritis (D) or JIA (E).

A 6-year-old girl presents to accident and emergency with a fever. She has no history of cough, cold, vomiting, diarrhoea, rash, headache or joint pain. On examination, she is tachycardic at 150bpm and there are two petechial spots on her right ankle. Her capillary refill time is 4 seconds and she has cold feet. All her other observations are normal. What is the most appropriate course of action? A. Inform the consultant about child protection concerns B. IV ceftriaxone C. IV fluid bolus and IV ceftriaxone D. Admit to the ward for observation E. Discharge home and advise to return if the rash spreads

C. IV fluid bolus and IV ceftriaxone This girl is tachycardic and therefore should be investigated to find the cause. With the petechial spots in the presence of a fever and clinical signs of shock, one must consider meningococcal sepsis and commence antibiotics immediately. Since she has features of early shock, namely tachycardia and prolonged capillary refill time, she therefore requires a fluid bolus so (C) is correct, not (B). Although bruising must always raise suspicion and appropriate measures taken, in this case an alternative diagnosis is more likely; therefore (A) is not applicable on this occasion. She should not be sent home (E) as she is tachycardic. She needs prompt treatment after assessment to avoid progression to septic shock and therefore (D) is not appropriate.

A 2 year old was seen in accident and emergency by the senior house officer with a short history of fever, malaise and now vomiting. She had a blanching rash on her arms and abdomen. She looked unwell but had no clear focus for her fever. She was tachypnoeic but her chest was clear. A urine sample was requested which showed a trace of leukocytes and two plus of ketones. Forty-five minutes later the paediatric registrar came to review the child who appears lethargic with a capillary refill centrally of 6 seconds and the rash on her abdomen is now non-blanching. What is the most likely diagnosis? A. Urinary tract infection (UTI) B. Idiopathic thrombocytopenia C. Meningococcal sepsis D. Human herpes virus 6 infection E. Diabetic ketoacidosis

C. Meningococcal sepsis This child is rapidly becoming unwell with impending shock (capillary refill 6 seconds). The expanding rash quickly becomes non-blanching and suggests that she has disseminated intravascular coagulation, often seen with meningococcal sepsis (C). Idiopathic thrombocytopenia (B) is usually seen in well children following a recent upper respiratory tract infection. While the urine dipstick has a trace of leukocytes, this is most likely non-specific due to her being so unwell and in the absence of nitrites is not strongly suggestive of a UTI (A). Human herpes virus 6 infection is a very common cause of fever and rash in this age group, not infrequently associated with febrile convulsions, but it does not cause shock and the rash would remain blanching (D). Tachypnoea without chest signs is an indicator of acidosis but the urine dipstick did not have any glucose in it, so this presentation is not due to diabetic ketoacidosis (E).

A mother brings her 6-month-old, formula-fed baby to see the GP complaining that the olive oil she is using is not helping his persistent cradle cap and worsening rash on his face and arms. On examination he has extensive cradle cap and eczematous changes on his cheeks, neck, chest and arms. The neck skin creases are red and oozing with yellow crusts. He is miserable and feels warm to touch. What is the most appropriate management? A. Advise using emollients and a soap substitute B. Start emollients with a topical antibiotic C. Refer to hospital for intravenous antibiotics D. Recommend a trial of switching to soya based formula as he may be cow's milk protein allergic E. Start topical steroids on the inflamed areas, and intensive emollient treatment

C. Refer to hospital for intravenous antibiotics This child has infected eczema, characterized by red inflamed skin with yellow crusts which suggests a staphylococcal infection (but eczema herpeticum also needs to ruled out). He is systemically unwell, with misery and most probably a fever. He therefore needs intravenous antibiotics and must be referred to hospital (C). If he was not unwell, starting emollients with oral antibiotics would be appropriate, but topical antibiotics are to be avoided since they may rapidly select for resistant organisms and they may themselves be sensitizing (B). This child's eczema is severe and exploring other contributory factors such as cow's milk protein sensitivity is part of good management, but a trial of soya based formula without any clear history or investigations is inappropriate (D). He should be started on emollients and soap substitute (A) but this is not the best answer as he needs systemic antibiotics as well. Steroids (E) should not be started until the infection has been treated but he will require steroid creams to help treat his eczema once the infection has resolved.

A 9-year-old girl presents to accident and emergency with fever, vomiting and dysuria. She is wearing a steroid bracelet and has a steroid card stating she is on daily prednisolone for severe asthma and eczema and is therefore at risk of adrenal suppression. She is tachycardic at 140bpm and you are concerned that her blood pressure is low. Her capillary glucose is 3.0 mmol/L. What is the single most important investigation? A. Cortisol B. Full blood count C. Renal function tests D. Urine culture E. Blood culture

C. Renal function tests This girl is having an Addisonian crisis. She has been on long-term corticosteroids and is therefore at high risk of having developed adrenal suppression. She has subsequently developed a urinary tract infection and the physiological response would be to increase endogenous steroid production. However, with her hypothalamic-pituitary-adrenal axis down-regulated due to her exogenous steroid use, this is not possible. She therefore is steroid depleted. She has hypotension, hypoglycaemia and is likely to develop hyperkalaemia and hyponatraemia. Cortisol (A) is likely to be low, but this will not change your management of her condition. Full blood count (B), urine culture (D) and blood culture (E) may help to elucidate the cause of the deterioration and treat it, but will not provide information about the Addisonian crisis. Renal function (C) is important to identify hyponatraemia and hyperkalaemia which may require treatment. Hydrocortisone and IV dextrose would be the initial treatment of choice, with antibiotics.

John is a 2-year-old boy whose mother has been concerned about a cough for the last 2 weeks which started out of the blue. He has been previously fit and well with no respiratory or cardiac problems from birth. There is no family history of illness. He is thriving and eating as normal, but has a persistent cough, recently productive of yellow and slight blood stained sputum. You suspect that John may have a pneumonia and lung collapse secondary to an inhaled foreign body. Which is the most likely location of this boy's foreign body? A. Left lower lobe B. Right upper lobe C. Right middle lobe D. Left upper lobe E. Right lower lobe

C. Right middle lobe The right middle lobe is the terminal one of the three branches of the right main bronchus (the other being the right upper lobe and right lower lobe). The right main bronchus is the widest, shortest and most vertical of the bronchi and hence the most likely for a foreign body descent (path of least resistance). The right middle lobe is the most direct anatomically likely location for the foreign body; therefore (A), (B), (D) and (E) are incorrect.

A 6-year-old boy is taken to see the GP by his mother because he has been getting severe abdominal pains, sometimes with vomiting and yesterday with a headache as well. He has no diarrhoea or constipation. His growth and examination are normal. He has no significant past medical history. In his family history, his maternal grandfather recently died of gastric cancer and mum's migraines have been worse since his death. She is worried her son is getting gastric cancer too. What is the most likely diagnosis? A. Crohn's disease B. Brain tumour C. Somatization disorder D. Gastric cancer E. Coeliac disease

C. Somatization disorder This child presents with abdominal pain, sometimes with vomiting or associated headache, with a family history of migraine and distinct psychological triggers, making the most likely diagnosis a somatization disorder (C). This term encompasses various different descriptive terms such as abdominal migraine, recurrent abdominal pain, non-organic pain, functional pain and irritable bowel syndrome. Children may experience migraines as abdominal pain and as they get older they develop more classical migraine headaches. Crohn's disease (A) would usually present in an older child (but can occur in this age group) with changes in bowel habit and failure to thrive. There is often a family history, which is not present in this case. A brain tumour (B) would be more likely to present with headaches rather than abdominal pain as well as having neurological signs and symptoms. Gastric cancer (D) is very rare in children but would have features of poor appetite and weight loss as well. Coeliac disease (E) usually presents with abdominal distension, diarrhoea and failure to thrive.

A 13-year-old girl is on the ward having a bone marrow transplant for acute leukaemia. She is noted to be profoundly anaemic with haemoglobin 5.9 g/dL and she is due to receive a transfusion of one unit of red blood cells. You are called to see her 5 minutes after starting the transfusion. She has come out in a rash, is looking frightened, with a heart rate of 120bpm and respiratory rate of 30. As you arrive, you can see she has swollen lips and tongue and her blood pressure is measured as 90/45 mmHg. What is the best initial management step? A. Repeat a full set of observations as it is likely to be anxiety resulting in the abnormal heart rate, respiratory rate and blood pressure. If still abnormal, stop transfusion B. Stop the transfusion and return the unit of blood to blood bank C. Stop the transfusion, take down the giving set, give IM adrenaline immediately D. Give IM adrenaline, stop the transfusion, take down the giving set E. Stop the transfusion, give IM adrenaline immediately and restart if the reaction settles

C. Stop the transfusion, take down the giving set, give IM adrenaline immediately There are clear transfusion guidelines which you should be familiar with - see Handbook of Transfusion Medicine from the UK Blood Transfusion Services. There are multiple transfusion reactions, but this is an allergic/ anaphylactic reaction. The blood should be stopped and taken down first (removal of the allergen); therefore (A) and (D) are incorrect. The patient is in anaphylactic shock and therefore should be given IM adrenaline (C), not (B). This unit of blood should most certainly not be restarted (E) and should be returned to the blood bank for further analysis.

A 3-year-old boy has been admitted to hospital with a right-sided pneumonia and pleural effusion. The pleural fluid grew Gram-positive cocci. He is on IV ceftriaxone, oral azithromycin and has a chest drain in situ. On further questioning of Richard's mother, you establish that he has had multiple chest infections since he was born (in the UK). He has been admitted three times before and also had a sinus wash out following an episode of sinusitis. He has no cardiac anomalies or dysmorphism. His mother also tells you about his older brother, who sadly died of meningitis aged 6 years old. He too had 'more than his fair share of infections'. The two brothers had different fathers but his mother is HIV negative. What is the most likely underlying immunodeficiency in this family? A. DiGeorge's syndrome B. Complement deficiency C. X-linked agammaglobulinaemia D. Subacute combined immunodeficiency disorder (SCID) E. HIV

C. X-linked agammaglobulinaemia This boy presents with recurrent respiratory tract infections and although these are common in childhood, the frequency, severity and causative organisms are important to consider. It is difficult to clearly differentiate immunodeficiencies, but it is a balance of probabilities as to the most likely cause. He is unlikely to have SCID (D) as this usually presents earlier in life and may have been fatal by 3 years of age if undiagnosed. Their mother is HIV negative which means vertical transmission of HIV is excluded and he is unlikely to have acquired HIV (E). He is lacking in other features which are suggestive of DiGeorge's syndrome (A) (cleft palate, midfacial hypoplasia, cardiac anomalies (including Tetralogy of Fallot), VSD) and hypocalcaemia. Complement deficiency (B) can present in a spectrum of phenotypic characteristics, but commonly in meningococcal infections. The fact that two brothers both have the propensity to infections should raise the suspicion of an X-linked condition (irrespective of the paternity of the brothers). The lack of B cells, and therefore immunoglobulins, predisposes to bacterial infection, particularly of the respiratory tract but also the CNS, making (C) the most likely.

A 5-week-old baby was admitted today to the children's ward with bronchiolitis. The nasopharyngeal aspirate identified respiratory syncitial virus. He was saturating to 96 per cent in air this morning and was feeding two-thirds of his usual amount of formula milk. You are asked to review him as his work of breathing is worsening now it is night time. He has nasal flaring, intercostal and subcostal recession, tachypnoea and crepitations and wheeze heard bilaterally. What do you expect his capillary blood gas to show? A. pH 7.16 PCO2 kPa 3.1 PO2 10.0 kPa BE -8 HCO3- 18 mmol/L B. pH 7.38 PCO2 kPa 5.5 PO2 12.0 kPa BE +1 HCO3- 25 mmol/L C. pH 7.20 PCO2 kPa 8.2 PO2 8.3 kPa BE +2 HCO3- 26 mmol/L D. pH 7.40 PCO2 kPa 1.2 PO2 7.5 kPa BE +5 HCO3- 28 mmol/L E. pH 7.47 PCO2 kPa 6.3 PO2 11.0 kPa BE +10 HCO3- 35 mmol/L

C. pH 7.20 PCO2 kPa 8.2 PO2 8.3 kPa BE +2 HCO3- 26 mmol/L He has increased work of breathing causing him to tire. He has an infective process in his lungs resulting in inflammation and an inability to oxygenate effectively. Therefore he will develop type 2 respiratory failure as seen in (C) with hypoxia and hypercapnia producing respiratory acidosis. The blood gas in (D) may represent his situation before he becomes exhausted, with hyperventilation resulting in low PCO2 and hypoxia, but with metabolic compensation. (A) is a metabolic acidosis which may be secondary to sepsis. (B) is a normal blood gas. (E) is a metabolic alkalosis.

A mother is awoken by her 5-year-old son gurgling and then notices that the right-hand side of his face is twitching. EEG shows centrotemporal spikes when asleep. (Fits) A 'Breath-holding' attacks B Absence C Infantile spasms (West's syndrome) D Benign rolandic E Juvenile myoclonic F Lennox-Gastaut syndrome G Tonic-clonic H Benign myoclonic epilepsy of infancy I Landau-Kleffner syndrome J Febrile convulsion

D Benign rolandic Benign Rolandic. Predominantly nocturnal sensorimotor seizures. Characteristic EEG during slow wave sleep or drowsiness.

Common, fluctuant swelling due to subperiosteal bleed, limited by suture lines. (Newborn examination) A Talipes equinovarus B Developmental dysplasia of the hip C Vernix D Cephalhaematoma E Epstein's pearls F Polydactly G Syndactly H Erb's palsy I Klumpke's palsy J Subaponeurotic haematoma

D Cephalhaematoma Cephalhaematoma. Most commonly occurs over parietal bones and resolves over weeks.

A 2-month-old baby girl is admitted via A&E following an apnoeic episode. She had a 2-day history of coryza followed by episodes of spasmodic coughing. FBC revealed a lymphocytosis. (Treatment of respiratory distress) A Palivizumab B Dexamethasone C Cefotaxime D Erythromycin E Amoxicillin F Nebulized salbutamol G Salbutamol via inhaler H Nebulized adrenaline I IM adrenaline J IV adrenaline

D Erythromycin Erythromycin. Whooping cough (Bordetella pertussis).

An 8-year-old boy presents to A&E with a history of right knee pain and a limp. On examination his right thigh has atrophied and he has reduced range of movement in the right hip. X-ray of the hip shows lateral subluxation of the femoral head and a bone scan shows reduced uptake in the femoral epiphysis. (Limping child) A Developmental dysplasia of hip B Transient synovitis C Septic arthritis D Perthes' disease E Osteomyelitis F Slipped upper femoral epiphysis G Osgood-Schlatter's disease H Juvenile idiopathic arthritis I Non-accidental injury J Duchenne's muscular dystrophy

D Perthes' disease Perthes' disease is due to an idiopathic avascular necrosis of the femoral head of unknown aetiology and causes a classical 'painless limp'.

Palpable abdominal mass, haematuria. (Tumours) A Acute lymphoblastic leukaemia B Lymphoma C Neuroblastoma D Wilm's tumour E Retinoblastoma F Rhabdomyosarcoma G Osteosarcoma H Ewing's sarcoma I Acute myeloid leukaemia J Glioma

D Wilm's tumour Wilm's tumour. An embryonal tumour of the kidney (nephroblastoma). Mostly sporadic.

What is a greenstick fracture? A. The classic pattern of vertebral column fractures associated with abuse by being hit with a cane or 'green stick' B. A fracture of the distal radius and ulna with dorsal displacement associated with a fall on the outstretched hand C. A fracture of the distal radius and ulna with ventral displacement D. A fracture of the long bones in young children where only one cortex is broken and the other is buckled E. A fracture of the long bones in young children where the cortex is buckled on one side of the bone with no cortex separation on the opposite side

D. A fracture of the long bones in young children where only one cortex is broken and the other is buckled In young children the bones are soft and flexible so a forceful impact may bend the bone rather than break it. This results in a buckle in the cortex. In a torus fracture (E) only buckling of the cortex is seen just on one side of the long bone whereas in a greenstick fracture (D) a buckle is seen on one side with the opposite cortex interrupted. The greenstick fracture takes longer to heal than the torus fracture. A Colles fracture (B) results in the distal end of the ulna and radius sliding backwards and shortening; it is unusual in children and would more commonly occur through the growth plate, known as a Salter-Harris fracture. The reverse of a Colles fracture occurs in a Smith's fracture (C) with ventral displacement of the radial fragment. There is no classic fracture pattern of the spine associated with child abuse (A).

A 14-year-old girl was diagnosed with idiopathic thrombocytopenic purpura (ITP) last week after she attended the children's assessment unit with recurrent epistaxis. She had a platelet count of 16×109/L last week and now re-presents to accident and emergency with further episodes of epistaxis, haematemesis and petechiae. She had a heart rate of 110bpm and her blood pressure is 100/70 mmHg. What is the next best management step? A. Give a platelet transfusion and red cell transfusion B. Arrange an urgent upper gastrointestinal endoscopy C. Give IV immunoglobulin and steriods D. Admit and monitor the haemodynamic status and administer a fluid bolus E. Discharge home with advice to return if the symptoms continue for more than 24 hours

D. Admit and monitor the haemodynamic status and administer a fluid bolus This girl has ITP, characterized by low platelets and mucosal bleeding. It usually occurs a few weeks after a viral illness. It is caused by immune- mediated destruction of circulating platelets within the reticuloendothelial system. It typically occurs in children between 2 and 10 years of age. The management depends on the severity of the presenting symptoms. The course of the illness is self-limiting, and if there is no continued bleeding, they may be monitored as an outpatient. However, this girl has continued episodes of bleeding and is tachycardic and hypotensive. She cannot be discharged (E) as she is medically unstable and needs close monitoring and a fluid bolus (D). Platelet transfusion (A) is not usually required as these platelets will be destroyed too, but in life-threatening bleeding with low platelets they may be required. An urgent endoscopy (B) would help to identify a gastric or oesophageal lesion, but given the previous history and low platelets this is not required and may cause harm if it were to cause further bleeding. IV immunoglobulins and steroids (C) can be used for children with continued bleeding, to try to reduce the underlying immune destruction, but a bone marrow aspirate should be carried out prior to commencing steroids to exclude malignancy.

A 10-year-old girl was diagnosed with diabetes 1 year ago. She has been compliant with her insulin regimen and her HBA1c is 6 per cent. She is attending her annual diabetic review and has been asking about why she has diabetes. What is the aetiology of type 1 diabetes? A. Inflammation of the pancreas causing exocrine and endocrine dysfunction B. Impaired glucose tolerance C. Secretory dysfunction of the pancreatic duct D. Autoimmune destruction of pancreatic islet cells E. Peripheral insulin resistance

D. Autoimmune destruction of pancreatic islet cells Type 1 diabetes is due to autoimmune destruction of the B cells of the islets of Langerhans in the pancreas. This leads to absolute insulin deficiency and a state of catabolism, particularly lipids, in the absence of insulin. These children are dependent on exogenous insulin for life. Type 2 diabetes is due to reduced production of insulin but also peripheral resistance in tissues (E). Diabetes may be acquired with other conditions such as chronic pancreatitis (A), corticosteroid use (B) or cystic fibrosis (C).

A 5-year-old boy was diagnosed with asthma aged 3 years. He presented to accident and emergency with shortness of breath, increased work of breathing and a 1-week history of coryzal illness and fever. On examination he is tachypnoeic 60/ min, tachycardia 160 bpm and has minimal air entry bilaterally. He has intercostal recession, tracheal tug and is too breathless to complete a sentence. Oxygen saturation is 90 per cent in air. What is the initial management of this boy? A. Immediate intubation and ventilation B. High flow oxygen through non-rebreather mask C. IV salbutamol and magnesium sulphate infusion D. Back to back salbutamol and Atrovent though oxygen driven nebuliser E. Trial of continuous positive airway pressure support

D. Back to back salbutamol and Atrovent though oxygen driven nebuliser Asthma management follows the British Thoracic Society guidelines which have a step-wise approach to the management of both chronic and acute asthma. According to his symptoms he has life-threatening asthma, but initial management still requires the use of back to back salbutamol nebulizers (D). He is hypoxic in room air, so giving oxygen driven nebulizers will be of benefit compared to inhalers or oxygen alone (B). If after three doses of salbutamol and one of Atrovent nebulizer he is not improving, he may require IV salbutamol or aminophylline and magnesium sulphate, but not first line (C). Although he does require high flow oxygen, this alone will not help the underlying problem. If he is not responding to intravenous therapy then intubation and ventilation will be required with intensive care support (A). Continuous positive airway pressure (E) is not used in the management of asthma, as it does not provide ventilatory support.

A 6-year-old girl has presented to her GP with a rapid increase in growth. Her mother is also concerned that she seems to have developed pubic and axillary hair and breast development prior to this but thought it would go away. She has no history of trauma and has reported problems with her vision. Her levels of gonadotrophin-releasing hormone (GnRH), follicle-stimulating hormone (FSH), luteinizing hormone (LH) and oestrogen are high. You are concerned that she may have a pituitary tumour. What is the likely visual field defect? A. Monocular blindness B. Central scotoma C. Homonymous hemianopia D. Bitemporal hemianopia E. Myopia

D. Bitemporal hemianopia This girl has features consistent with precocious puberty, which is the onset of puberty at <8 years in girls and <9 years in boys. The causes may be central or gonadal. Central causes will have raised GnRH, FSH and LH levels which stimulate the gonads to produce oestrogen or testosterone. Gonadal causes have low GnRH, FSH and LH due to negative feedback from excessive endogenous production of oestrogen or testosterone. Central causes may include a pituitary tumour which can lead to excessive production of the pituitary hormones (FSH, LH, melanocyte-stimulating hormone, adrenocorticotrophic hormone, prolactin, growth hormone, oxytocin, antidiuretic hormone). The pituitary is in close proximity to the optic chiasm, where the optic tracts decussate from the nasal visual fields. As the chiasm is compressed by the tumour, these tracts are affected leading to a bitemporal hemianopia (D). The other visual disturbances would not be directly related to a pituitary tumour. Monocular blindness (A) may be due to unilateral disease in front of the optic nerve or involving the optic nerve itself, e.g. cataracts, vitreous haemorrhage, retinoblastoma. Central scotoma (B) can result from multiple sclerosis or optic nerve gliomas. Homonymous hemianopia is due to lesions of the optic radiation posterior to the lateral geniculate body, such as stroke or cerebral palsy. Myopia (E) or short-sightedness is common and may be related to syndromes such as Marfan's syndrome.

Which for the following is not a feature of raised ICP? A. Headache B. Morning vomiting C. Sun setting eyes D. Bulging anterior fontanelle E. Papilloedema

D. Bulging anterior fontanelle All of these options are associated with intracranial pathology but a bulging anterior fontanelle occurs when there is increased fluid in the brain in a young child before the fontanelle closes; thus there will be no raised ICP as the open fontanelle provides a space for the extra fluid. In an older child with the same underlying pathology and a closed fontanelle, the result is raised ICP. Symptoms and signs may include: headache (A), morning vomiting (B), sun setting eyes (C) (a late sign of raised ICP caused by pressure on cranial nerves III, IV and VI) and papilloedema (again a late sign of raised ICP).

A 10-year-old girl with sickle cell disease presents to her GP on Monday morning complaining of weakness in her right leg. She says she collapsed on Saturday afternoon and has not felt right since. What is the most likely diagnosis? A. Sickle cell painful crisis B. Parvovirus B19 infection C. Aplastic crisis D. Cerebral infarction E. Osteomyelitis of the right femur

D. Cerebral infarction Here the worrying and important diagnosis to rule out is a stroke (D). Sickle cell patients are at risk of stroke, especially if their sickle cell percentage is above 30 per cent of the total red blood cell population. She is not complaining of pain in her leg, just weakness; therefore this is not a painful crisis (A) or osteomyelitis (E). Sickle cell patients with parvovirus B19 infection (B) are at risk of an aplastic crisis (C) with associated secondary infections, bleeding and severe anaemia, but these would not explain her symptoms.

A 14-year-old refugee from Afghanistan who has lived in the UK for 2 years comes to see you complaining of increasing fatigue and breathlessness on exertion. On examination she appears cyanosed and has bilateral basal fine crepitations and a soft pansystolic murmur with a displaced apex beat. She has never been in hospital and has no surgical scars. You urgently refer her for a cardiology review. What is the most likely diagnosis? A. Bacterial endocarditis B. Tetralogy of Fallot C. VSD producing a left-to-right shunt D. Eisenmenger's syndrome E. Ebstein's anomaly

D. Eisenmenger's syndrome This child grew up in an area without resources for congenital cardiac corrective surgery. She likely has a large VSD which has led to left ventricular hypertrophy and failure, to the extent that she now has pulmonary hypertension and a right-to-left shunt resulting in cyanosis, i.e. Eisenmenger's syndrome (D). There are no infective features to suggest bacterial endocarditis (A). Without surgical correction, a child with Tetralogy of Fallot (B) would not survive to their teens. Young children with a VSD have a left-to-right shunt (C) with blood flowing from the high pressure left ventricle to the right ventricle and this does not produce cyanosis. Ebstein's anomaly (E) is another congenital heart defect that would not survive without surgery and is a combination of an abnormal tricuspid valve, hypoplastic right ventricle and pulmonary stenosis.

A 2-year-old boy is admitted to the paediatric ward with a swollen, painful left knee. He has been afebrile and has a history of minor trauma to his knee earlier today. His mother is a haemophilia carrier and his father is not affected. You are keen to rule out haemophilia in this child. Which two clotting factors should you test for? A. Factor VII and IX B. Factor VII and VIII C. Factor V and VI D. Factor VIII and IX E. Factor X and XI

D. Factor VIII and IX Haemophilia A and B are X-linked inherited conditions. Haemophilia A is due to a deficiency in clotting factor VIII. Haemophilia B is due to a deficiency in clotting factor IX. There is a spectrum in severity of the disease based on the extent of the deficiency and proportion of functional factor: Mild 10-50 per cent, moderate 2-10 per cent, severe <2 per cent Haemophilia can be treated with replacement of factors VIII or IX for haemophilia A and B, respectively. Desmopressin can be useful to stimulate production of clotting factors. Consideration of the clotting function is needed during times of elective and emergency surgery. Physiotherapy is important to minimize the complication of arthritis in the affected joint.

A 4-year-old boy has been brought into accident and emergency with breathing problems. He is assessed by the paediatric team and found to have inspiratory and expiratory stridor, audible wheeze, lip and tongue swelling, and an urticarial rash on his trunk and abdomen. His heart rate is 167 bpm and his respiratory rate is 40, BP 90/45 mmHg. What is the single most important management step? A. Do not examine his throat as this may distress him B. Give a normal saline fluid bolus C. Give IV adrenaline 0.1mg/kg of 1:10000 D. Give IM adrenaline 0.01mg/kg of 1:1000 E. Mobilize the paediatric anaesthetist as his airway is compromised

D. Give IM adrenaline 0.01mg/kg of 1:1000 He has clear signs of anaphylaxis, with involvement of cardiovascular and respiratory systems. He is hypotensive and tachycardic and although a fluid bolus is required (B), adrenaline IM (D) is the most important initial step. Adrenaline is a catecholamine which in anaphylaxis should be given IM and not IV (C) (this is in contrast to a cardiac arrest protocol where it is given IV or through an endotracheal tube). IM adrenaline will cause vasoconstriction, bronchodilation and temporarily slow the anaphylactic process, which is histamine driven. A paediatric anaesthetist will also be required to assess and manage airway obstruction due to the enlarged tongue and upper airway soft tissues (E); however adrenaline must be given first to buy time for them to arrive. The effects of adrenaline are short lived and continued assessment is necessary. Once the reaction has settled there is a biphasic response, meaning the child may have a second reaction despite no contact with the allergen 6-12 hours later. Not examining the throat (A) refers to upper airway obstruction caused by epiglottitis or bacterial tracheitis and is not appropriate here.

A 6-year-old boy with a history of asthma and eczema is brought in to accident and emergency from a local restaurant. He is on high flow facial oxygen with significant facial oedema and generalized erythema. On auscultation there is widespread wheeze for which the ambulance crew gave a salbutamol nebulizer. What is the next step in management? A. Insert an IV line and give 10mg slow intravenous antihistamine B. Insert an IV line and give 100mg slow intravenous hydrocortisone C. Insert an IV line and give 200μg of 1:10000 intravenous adrenaline D. Give intramuscular 1:1000 adrenaline, 250μg E. Repeat the salbutamol nebulizer and call for an anaesthetist for intubation

D. Give intramuscular 1:1000 adrenaline, 250μg All of the answers are correct except for (C) which is the dose of adrenaline for cardiac arrest. (D) is the best answer as it is the most important life-saving treatment; IM adrenaline will work to reduce the capillary leak which is producing airway oedema. Once the fluid has shifted out of the vascular space, there is no way to rapidly move it out of the tissues; therefore the key first management of anaphylaxis is to administer IM adrenaline. In reality you would put out an emergency call for an anaesthetist (E) to manage the rapidly closing airway, give further salbutamol nebulizers and gain intravenous access to give hydrocortisone and antihistamine. Give (A) to block the histamine release which is driving the capillary leak and (B) to reduce general inflammation and aim to prevent the late type IV hypersensitivity. It is important not to forget about delayed type IV hypersensitivity; these children should be observed and sent home with two further doses of prednisolone for the next 2 days to cover this.

A 13-year-old girl has presented to her GP with her mother with concerns that she is the shortest in her class at school. She has always been 'on the small side' according to her mother, despite eating well. When you examine her you find she is hypertensive but has no cardiac murmur. Respiratory and abdominal systems are normal. She has no signs of pubertal development and you notice she has widely spaced nipples and a low hair line. You are considering the diagnosis of Turner's syndrome. What is the most appropriate diagnostic investigation? A. Mid-parental height B. Echocardiogram C. Four limb blood pressures D. Karyotype E. Fluorescence in situ hybridization (FISH)

D. Karyotype Turner's syndrome results from the genotype 45X0. Therefore a karyotype would be diagnostic (D). FISH (E) is used to diagnose conditions caused by microdeletions in conditions such as DiGeorge' syndrome, cri du chat syndrome and Williams' syndrome. The four limb blood pressure (C) and an echocardiogram (B) may be useful in coarctation of the aorta and aortic stenosis which can occur as part of Turner's syndrome but are not important in the diagnosis itself. Mid-parental height (A) is important to note in children who are short, as one of the differentials to consider is familial short stature. It is calculated by the sum of the parents' height in centimetres, divided by 2, then +7cm for a boy or -7cm for a girl.

A 15-year-old boy attends his GP with a week of cough productive of yellow sputum, fever to 39°C and chest pain on the right side of the chest on coughing. There is no history of foreign travel or unwell contacts. On examination there is reduced air entry in the right lower zone with crepitations and bronchial breathing. You diagnose a right-sided chest infection. What is the most likely causative organism? A. Staphylococcus aureus B. Mycobacterium tuberculosis C. Streptococcus pneumoniae D. Mycoplasma pneumoniae E. Chlamydophila pneumoniae (Chlamydia pneumoniae)

D. Mycoplasma pneumoniae The causative organisms of pneumonia can be categorized into typical (Streptococcus pneumoniae, Staphylococcus aureus, Haemophilus influenzae, Moraxella catarrhalis) or atypical (Mycoplasma pneumoniae, Legionella spp., Chlamydia spp.). Streptococcus pneumoniae (C) is the most common typical cause and would be more likely in under 4 year olds. Mycoplasma pneumoniae (B) is more common in older children presenting with pneumonia. The presentation of a lobar pneumonia and a short history is not consistent with tuberculosis (B) which may present with systemic features of weight loss, loss of appetite and a more indolent course. Staphylococcus aureus (A) and Chlamydia pneumonia (E) are possible but are less likely causes epidemiologically.

A 10-year-old girl with Graves' disease attends her GP with worsening of her symptoms. She was well controlled on carbimazole and has had relatively few symptoms for the past 6 months. She now has sweats, weight loss, diarrhoea and tremors which are affecting her school performance. What is the next management step? A. Review in 3 months B. Radioisotope therapy C. Stop carbimazole and start propranolol D. Optimize carbimazole dose and add propanolol E. Referral for thyroidectomy

D. Optimize carbimazole dose and add propanolol Hyperthyroidism is due to raised thyroxine which may be due to high pituitary stimulation (high TSH) or endogenous thyroid production (low TSH). The symptoms are multisystemic and include tachycardia, heat intolerance, diarrhoea, weight loss, amenorrhoea, tremor and psychosis. Carbimazole acts to suppress thyroxine production and therefore reduce symptoms. This girl has got worsening symptoms and therefore the first course of action would be to optimize the dose of carbimazole to reduce the production of thyroxine, and then add propranolol which can be given for symptomatic relief, especially for tremors (D). Propranolol is not a replacement for carbimazole as it does not directly affect thyroxine production (C). Radioisotope treatment (B) and surgery (E) would be considered as further management strategies. (A) is not correct as she is symptomatic and therefore requires treatment to become clinically euthyroid.

Which of the following is not a correct match? A. Systemic JIA - acute illness with daily fevers, malaise, failure to thrive, rash, muscle and joint aches for greater than 6 weeks associated with raised inflammatory markers B. Extended oligoarthritis - an arthritis originally affecting one or two joints for the first 6 weeks and over time has spread to multiple joints C. Psoriatic arthritis - presents with interphalangeal joint swelling, scaly skin rash, nail pitting and dactylitis D. Polyarticular arthritis - more common in boys, affecting multiple small joints for more than 6 weeks E. Enthesitis-related arthritis - associated with HLA-B27 tissue type, and presents in older boys with large joint arthritis, swollen tender tendons, sacro-iliitis and bamboo spine on x-ray. It is associated with anterior uveitis which if left untreated may cause blindness

D. Polyarticular arthritis - more common in boys, affecting multiple small joints for more than 6 weeks All of the above definitions are true except for polyarticular arthritis (D) which is more common in girls and presents with symmetrical arthritis of the wrists, hands, ankles and knees. Occasionally the spine and jaw may be affected as well. By definition, polyarticular arthritis affects more than four joints at presentation for more than 6 weeks. Oligoarthritis (B) presents with less than five affected joints in the first 6 weeks and even if other joints become involved later it is still referred to as oligoarthritis but becomes extended. If the oligoarthritis persists beyond 6 months without extension to five or more joints it is called persistent. Option (A) is a classic description of systemic juvenile idiopathic arthritis, which should be considered in the differential diagnosis of prolonged fever in children, even when minimal or no joint symptoms or signs are evident. Option (C) is a classic description of the findings in psoriatic arthritis. Enthesitis-related arthritis (E) is also called juvenile spondylitis, and refers to inflammation of the entheses, which are the areas where tendon and other connecting tissues join to bone.

A 5-year-old girl was admitted to the ward after she presented to her local accident and emergency with diarrhoea. She was passing 7-8 loose, watery stools per day for the last 4 days and had been vomiting for 1 day prior to this. There was blood in the stools and this had worried her mother. You ask about foreign travel and her mother reveals they had been in India until 2 weeks ago, staying with family and drinking tap water. She had no vaccines prior to travelling. On examination, she now has abdominal pain, swinging pyrexias, right upper quadrant tenderness but no rebound or guarding. You notice a pale pink (rose) spot on her trunk. What is the most likely infecting organism? A. Rotavirus B. Shigella spp. C. Vibrio cholerae D. Salmonella typhi E. Escherichia coli 0157

D. Salmonella typhi Although the most common cause of infective gastroenteritis worldwide is rotavirus, causing over 50 per cent of all cases, this case has some unusual features, excluding (A). Cholera (C) presents with dysentery not just diarrhoea. Blood in the stool (although possible with severe rotavirus) is more commonly caused by bacteria. With the history of travel to India and the added risk factors of staying in the local community, a more tropical cause should be considered. The presence of rose spots indicates this is (D), excluding (B) and (E), though stool microscopy and culture will provide the definitive diagnosis with sensitivities for effective treatment.

A 3-day-old baby is brought to accident and emergency with acute respiratory distress. She is tachypnoeic, tachycardic, cyanosed and her capillary refill is 5 seconds centrally. You note she has a flat nasal bridge, down sloping palpebral fissures and epicanthic folds. On auscultation there is a loud ejection systolic murmur at the left sternal edge. What is the most likely diagnosis? A. Coartation of the aorta B. VSD C. Transposition of the great arteries D. Tetralogy of Fallot E. Patent ductus arteriosus

D. Tetralogy of Fallot This is a presentation of a duct-dependent type of congenital heart defect. The ductus arteriosus is a short vessel connecting the pulmonary artery to the aortic arch. It allows most of the blood from the right ventricle to bypass the fetal lungs which are filled with fluid. Within hours or at the most a few days following birth, the duct closes to form the ligamentum arteriosum. However, any congenital heart defect that results in decreased blood flow from the heart to the lungs, or decreased oxygenated blood on reaching the systemic circulation, will depend on blood flowing through the ductus arteriosus to compensate for the abnormal anatomy. When the duct closes, that circulation stops and infants with Tetralogy of Fallot (D), pulmonary atresia, tricuspid atresia, transposition of the great arteries (C) and total anomalous pulmonary venous drainage will go blue. These duct-dependent lesions require urgent re-opening of the duct with prostaglandin. This situation is the opposite of a patent ductus arteriosus (E). The other information you are given describes dysmorphic features of Down's syndrome (trisomy 21). Down's syndrome is associated with atrioventricular septal defects, VSDs and Tetralogy of Fallot (D). Coarctation of the aorta (A) does not produce central cyanosis, although the left arm and lower limbs may have reduced blood pressure and saturations depending on the severity of the stenosis. VSDs (B) produce a left-to-right shunt (thus the infant is not cyanosed) unless they are left untreated, when Eisenmenger's syndrome develops, typically in the teenage years where pulmonary hypertension results in a right-to-left shunt and cyanosis. This process develops slowly and does not present in infancy.

A 4-month-old baby being investigated for infantile spasms is noted to have an ash leaf macule on his back under Wood's light. His EEG shows hypsarrythmia. The report of his MRI brain states there are subependymal nodules. What is the diagnosis? A. Neurofibromatosis type I B. Neurofibromatosis type II C. West's syndrome D. Tuberous sclerosis E. Tay-Sachs disease

D. Tuberous sclerosis Ash leaf macules are a skin manifestation of tuberous sclerosis (D) and the typical MRI findings of this are subependymal nodules. Tuberous sclerosis is a cause of infantile spasms, associated with hypsarrythmia on EEG, which are treated with vigabatrin. Seizures do occur in neurofibromatosis ((A) and (B)) but are much less common and not usually associated with any imaging changes. West's syndrome (C) is a syndrome of infantile spasms and developmental regression, one cause of which is tuberous sclerosis. As you are given no information on this child's development, West's syndrome is not the best answer when there is clear evidence to support the cause as tuberous sclerosis. Tay-Sachs disease (E) is an autosomal recessive deficiency of hexosaminidase A and presents with seizures, developmental regression, deafness, progressive loss of motor function and increased tone, which is most common in Ashkenazi Jewish populations.

What is the first step in management? A. Non-synchronized shock B. Adenosine C. Adrenaline D. Vagal manoeuvres E. Synchronized shock

D. Vagal manoeuvres The first step in management for a child who is still alert would be vagal manoeuvres (D) such as asking a child to blow a syringe like a balloon, unilateral carotid massage or putting the head in ice. Ideally, these should be carried out while the child is on recorded cardiac monitoring to assess the response. If these do not work, adenosine (B) would be the next step. Failing these actions, sedated synchronized cardioversion (E) is indicated while the patient remains alert. If a patient becomes cardiovascularly compromised, for example is losing consciousness, then the synchronized cardioversion should be carried out urgently without waiting for sedation. Adrenaline (C) would not be used unless the patient loses output. Non-synchronized cardioversion (A) is only indicated for ventricular fibrillation (VF) or ventricular tachycardia with no cardiac output; this is because in all other situations if the shock lands on the QRS complex it may cause VF.

Which of the following is not a presenting symptom or sign associated with congenital heart disease? A. Respiratory distress with feeds B. Cyanosis C. Hepatosplenomegaly D. Vomiting with feeds E. Sweating with feeds

D. Vomiting with feeds Congenital heart disease may be diagnosed antenatally on the anomaly ultrasound or at the baby check if a murmur is heard. However, if the signs are not present or seen at these time points, it is important to know the possible symptoms and signs that may evolve over the next few weeks or months that might be associated with congenital heart disease. Classically, children become breathless (A) and sweaty (E) with feeds. The effort of feeding burns calories and tires the child so that growth becomes restricted and the child may fail to thrive. Infants with heart failure may develop hepatomegaly and potentially splenomegaly, as a result of back-pressure secondary to right-sided heart failure (C). This sign is more helpful than looking for jugular venous distension in an infant, since the neck is short and the jugular veins are difficult to observe. Any cardiac defect with a right-to-left shunt may present with cyanosis (B). It would be unusual for congenital heart disease to be associated with vomiting (D) unless there were other illnesses such as gastro-oesophageal reflux or pyloric stenosis as well.

A 9-year-old boy is brought in by ambulance having been hit by a car while playing football in the street. You have been assigned to do the primary survey in resus when the ambulance arrives. The patient is receiving oxygen, crying for his mummy and holding his right arm, but able to move over from the stretcher to the bed when asked. Which is the correct examination procedure? A. The trachea is deviated to the right. On auscultation you hear decreased air entry on the left. Percussion note is hyper-resonant on the left. He is tachycardic and his heart sounds are muffled, heard loudest at the right lower sternal edge. You ask for a left-sided thoracocentesis. B. You introduce yourself and tell him that you will be gentle but need to check that he is okay. You see his left wrist is deformed and swollen and check the fingers which are cool and note the capillary refill is 4 seconds. He is able to feel you touching him and moans when you examine the wrist. You call for an x-ray to assess the probable fracture in the wrist. C. You introduce yourself and tell him that you will be gentle but need to check that he is okay. You listen for equal air entry and think there is decreased air entry on the left but there is air entry on the right. He is tachypnoeic and has a pulse which is tachycardic. His capillary refill is 4 seconds. You expose his abdomen and notice bruising and grazes to the left side. He moans as you palpate in the left upper quandrant and has guarding. You ask for an IV canula or intraosseous needle and a 20mL/kg fluid bolus while organizing an urgent CT chest and abdomen. D. You introduce yourself and tell him that you will be gentle but need to check that he is okay. He is tachypnoeic. The trachea is deviated to the right. On auscultation you hear decreased air entry on the left. Percussion note is hyper-resonant on the left. He is tachycardic and his heart sounds are muffled, heard loudest at the right lower sternal edge. You ask for a left-sided thoracocentesis. E. You listen for equal air entry and think there is decreased air entry on the left but there is air entry on the right. He is tachypnoeic and has a pulse which is tachycardic. His capillary refill is 4 seconds. You expose his abdomen and notice bruising and grazes to the left side. He moans as you palpate in the left upper quandrant and has guarding. You ask for an IV canula or intraosseous needle and a 20mL/kg fluid bolus while organizing an urgent CT chest and abdomen.

D. You introduce yourself and tell him that you will be gentle but need to check that he is okay. He is tachypnoeic. The trachea is deviated to the right. On auscultation you hear decreased air entry on the left. Percussion note is hyper-resonant on the left. He is tachycardic and his heart sounds are muffled, heard loudest at the right lower sternal edge. You ask for a left-sided thoracocentesis. Answer (D) is correct, you introduce yourself to the patient and follow the ABCD approach: A is for Airway, he is crying therefore it is patent. B is for Breathing; you find evidence of a pneumothorax therefore you do not proceed further in your assessment until B is addressed with a needle thoracocentesis decompression. (A) is incorrect as you have not introduced yourself which is very important in gaining patient trust and examination cooperation. (B) is examination of D = disability, you have skipped the ABC and missed several life-threatening conditions. (C) is incorrect as you failed to respond to a problem in (B) despite noticing signs of a tension pneumothorax. (E) is incorrect as you failed to introduce yourself and to address the pneumothorax noted in assessing breathing.

Is shy with strangers and may understand certain phrases. Will hold out arms and feet for sleeves and shoes. (Developmental milestones) A 6weeks B 6months C 30months D 9months E 12months F 18months G 2years H 3years I atbirth J 5years

E 12months

Crawling, pincer grip, stranger anxiety. (Development) A Newborn B 2 months C 3 months D 6 months E 9 months F 1 year G 2 years H 3 years I 4 years J 5 years

E 9 months

'Sandpaper-like' diffuse rash in neck and chest area. (Exanthems) A Varicella zoster B Parvovirus C Measles D Epstein-Barr virus E Group A Streptoccocus F Enteroviruses G Rubella H Human herpes virus 6 I Herpes zoster J Lyme disease

E Group A Streptoccocus Group A Streptococcus. Also known as scarlet fever, this is a fine popular rash on flushed skin with a sandpaper texture that may occur with streptococcal pharyngitis. Incubation 2-5 days, duration 7 days. Other features include sore throat, strawberry tongue and lymphadenopathy. Treatment is with 10 days of Penicillin V to prevent the development of rheumatic fever.

A 7-month-old baby boy presents to A&E with diarrhoea and vomiting. His mother describes episodes of colic associated with pallor, screaming and drawing up of his legs. In between these episodes he is lethargic and falls asleep. On examination 'redcurrant jelly stools' are found in the nappy. (Diarrhoea) A Toddler's diarrhoea B Hirschsprung's disease C Coeliac diasease D Inflammatory bowel disease E Intussusception F Haemolytic-uraemic syndrome G Henoch-Schönlein purpura (HSP) H Necrotizing enterocolitis (NEC) I Infective gastroenteritis J Food hypersensitivity

E Intussusception Intussusception typically affects infants from 6 months to 18 months, the majority associated with viral gastroenteritis.

A 10-year-old girl experiences jerking movements of her right arm while eating her breakfast. (Fits) A 'Breath-holding' attacks B Absence C Infantile spasms (West's syndrome) D Benign rolandic E Juvenile myoclonic F Lennox-Gastaut syndrome G Tonic-clonic H Benign myoclonic epilepsy of infancy I Landau-Kleffner syndrome J Febrile convulsion

E Juvenile myoclonic Juvenile myoclonic. Onset in second decade. Typically myoclonic jerks within first hour of wakening.

A 1-day-old Afro-Caribbean boy was found to have a large, blue, irregular patch on his buttocks during the newborn examination. (Newborn skin problems) A Port wine stain B 'Strawberry' naevus C Congenital melanocytic naevus D Sebaceous naevus E Mongolian blue spot F Stork mark G Erythema toxicum H Staphylococcal skin infection I Milia J Transient neonatal pustular melanosis

E Mongolian blue spot Mongolian Blue Spot. These are found in 90 per cent of Afro-Caribbean and Asian babies and in 5 per cent of Caucasians. They are benign, very common and usually fade.

A 1-month-old baby girl presents to A&E with a history of poor feeding, irritability and then sudden collapse. On examination she is collapsed with a heart rate of 240 bpm. (Collapsed neonate) A Coarctation of the aorta B Non-accidental injury C Sepsis D Inborn error of metabolism E Neonatal SVT F Neonatal abstinence syndrome G Pulmonary stenosis H TGA I Persistent pulmonary hypertension J AVSD

E Neonatal SVT Neonatal SVT. HR >220 in infants. Can present with poor feeding, irritability and pallor.

Leucocoria. (Tumours) A Acute lymphoblastic leukaemia B Lymphoma C Neuroblastoma D Wilm's tumour E Retinoblastoma F Rhabdomyosarcoma G Osteosarcoma H Ewing's sarcoma I Acute myeloid leukaemia J Glioma

E Retinoblastoma Retinoblastoma presents with abnormal or absent light reflex (leucocoria), squint or visual deterioration.

A 3-year-old child presents to the GP with a chronic cough for the last month. He had previously been fit and well since he suffered a severe pertussis infection when he was 1 month of age. He has subsequently been fully immunized but was noted to be on the 0.4th centile for height. What is the most likely cause for his cough? A. Cystic fibrosis B. Recurrent pertussis infection C. Habit D. Asthma E. Bronchiectasis

E. Bronchiectasis A severe respiratory infection in early childhood can present later in life with bronchiectasis (E) caused by dilatation and poor mucociliary clearance, predisposing to further infection. Any chronic illness may impact on the growth and development of a child. Other important causes of chronic cough include (D) asthma (though usually associated with nocturnal cough, atopy and eczema) and (A) cystic fibrosis (can present with respiratory and gastrointestinal involvement due to pancreatic insufficiency), and a thorough history is needed, but bronchiectasis (E) is still more likely given the history. Once the initial pertussis infection is treated it is unlikely that he will have further infections (B), and having been immunized this provides some protection, though immunity does wane with time and infected adults are likely the carriers of this disease. Coughs may become habitual (C) but his must be a diagnosis of exclusion.

A 4-year-old is brought into accident and emergency by very anxious parents. She has had a bad cough which makes her vomit and a fever for 2 days. She has now developed a rash on her face which does not pass the 'glass test', in that the spots are still visible when a glass is pressed against the skin. On examination she is alert and comfortable at rest, with fine petechiae on her cheeks and neck which are non-blanching. She has red, enlarged tonsils without pus and the chest is clear. What is the most likely cause of her rash? A. Meningococcal sepsis B. Idiopathic thrombocytopenia C. Henoch-Schönlein purpura D. Non-accidental injury E. Capillary rupture secondary to raised pressure in the superior vena cava distribution

E. Capillary rupture secondary to raised pressure in the superior vena cava distribution This child's parents have done 'the glass test' by pressing a tumbler against the skin, to check if the rash is blanching or not. They are very anxious because they have been taught that a positive glass tests means meningitis, usually caused by meningococcal sepsis (A). However, she is well in herself making this less likely. It would be sensible to admit her for observation even if antibiotics are not started. She has petechiae in the superior vena caval distribution (E) which often occur in situations of raised pressure such as coughing or vomiting but can also be seen in shaken babies or strangulation injuries. In young children coughing bouts often trigger vomiting, but vomiting is a warning sign of meningitis, so she should be monitored. ITP (B) is often triggered about 1 week after a throat infection and it would be unusual for the child to still be unwell and only have petechiae on the face. A strangulation injury by an angry caregiver could produce this pattern of injury; however her coughing and vomiting give a plausible explanation of the lesion, making non-accidental injury (D) unlikely. Henoch-Schönlein purpura (C) is a vasculitis of the capillaries and typically causes a macular papular purpuric rash over the buttocks and extensor surfaces of the limbs, so the distribution of the rash in this case is wrong.

A pregnant woman attends her booking appointment at the antenatal clinic and has her routine blood tests done. She is now 13 weeks pregnant with her first child and you have a positive result for cytomegalovirus (CMV) IgM. You need to discuss the implications of CMV infection on her unborn child. Which of the following are not features of congenital CMV infection? A. Deafness B. Intrauterine growth retardation C. Hydrocephalus D. Thrombocytopenia E. Congenital cardiac defects

E. Congenital cardiac defects CMV is the most common congenital infection acquired in pregnancy. It results in all the features in (A), (B), (C) and (D) after infection in the first trimester, but not (E). Cardiac defects are associated with other congenital infections such as rubella. Mothers are routinely screened for rubella, hepatitis B, HIV and syphilis in early pregnancy to ascertain risk to the fetus and treatment is given during pregnancy and may be required for the baby after birth.

A 4-year-old boy was diagnosed with nephrotic syndrome 6 months ago and has required a long course of oral corticosteroids to maintain remission of the condition. He has developed truncal obesity and you are concerned he may be developing Cushing's syndrome. Which of the following is not a complication of Cushing's syndrome? A. Osteoporosis B. Short stature C. Gastric irritation D. Hypertension E. Hypoglycaemia

E. Hypoglycaemia Oral corticosteroids are required for many inflammatory or autoimmune conditions to suppress the immune response and provide symptomatic relief for patients. We can minimize the effects of steroids by ensuring we give the minimum required dosage, alternate day prescribing and weaning at the end of the treatment course. Steroids have many side effects including: hypertension (D), glucose intolerance and tendency to hyperglycaemia, not hypoglycaemia (E), weight gain causing truncal obesity and a subscapular fat pad, short stature (B), osteoporosis (A), cataracts, gastric irritation (C), striae, thinning of skin and increased bruising, proximal myopathy, increased susceptibility to infection and adrenal suppression. The truncal obesity and thin myopathic limbs is characteristic of patients on long-term steroid therapy.

A 16-year-old boy attends your GP clinic for the first time with his father. He has recently moved to the area. His father is concerned that he is shorter than his peers at school and he frequently complains about being bullied. On further questioning there is no evidence of chronic illness or familial illness and he eats a balanced diet. His weight is on the 25th centile and his height is on the 10th centile. On examination he has no evidence of facial, axillary or pubic hair, his testes are both descended and are <4mL volume. What is the most likely cause of his delayed puberty? A. Anorexia nervosa B. Hypothalamo-pituitary dysfunction C. Kallmann' syndrome D. Cryptorchidism E. Constitutional delay

E. Constitutional delay The most common cause of delayed puberty is constitutional delay (E), although this is a diagnosis of exclusion. Delayed puberty is more common in boys. He does not have cryptorchidism (undescended testes) (D) on examination. Although there is a history of bullying which may in turn lead to psychological problems including anorexia nervosa (A) (rare in boys, but should be considered), his weight centile is greater than his height, making this unlikely though a body mass index should be calculated too (weight/height squared). Kallmann's syndrome (C) is characterized by low luteinizing hormone-releasing hormone and anosmia, which is not mentioned but this disease is rare (1 in 10000- 86000). It is important to exclude hypothalamo-pituitary dysfunction (B) by measuring GnRH, LH and FSH but it is not the most likely cause.

A 12-year-old girl with a history of discitis in her lumbar spine was admitted following investigation at her tertiary centre. She was started on IV benzylpenicillin and clindamycin. She received 24 hours of medication and a rash appeared on her trunk and arms. There were discrete red lesions which outlined a central target lesion. They were non-blanching and itchy. What is the most likely diagnosis? A. Erythema migrans B. Erythema toxicum C. Erythema marginatum D. Erythema nodosum E. Erythema multiforme

E. Erythema multiforme Erythema multiforme (E) is the rash described with target lesions with a surrounding red ring. The causes are numerous including: drugs (e.g. penicillin), infection (e.g. atypical pneumonia) or idiopathic. Erythema nodosum (D) can have similar categories of causes including drugs (e.g. sulphonamides), infection (e.g. streptococcal spp.), autoimmune and malignancy, but inflammatory bowel disease is also a well-known cause. Erythema migrans (A) is a rash due to Lyme's disease. Erythema toxicum (B) is a benign rash (characteristically seen as small pustules or vescicles surrounded by an erythematous area) seen in newborns in the first 2 weeks of life. Erythema marginatum (C) in seen in rheumatic fever along with the other cardinal signs which include carditis, arthralgia, subcutaneous nodules and Sydenham's chorea.

A 5-day-old baby is brought to see the GP because she has had a rash for the past 3 days which started on her chest, is spreading to her face and getting worse. On examination she handles well and is alert. There is an erythematous rash on her face, torso and right arm with little pustules. What is the most likely diagnosis? A. Infected eczema B. Neonatal sepsis C. Neonatal acne D. Molluscum contagiosum E. Erythema toxicum

E. Erythema toxicum This is a typical presentation of erythema toxicum (E), a common innocent rash of the newborn which has an erythematous base with small pustules. It comes and goes all over the body for the first few weeks of life. The baby is well so an infection ((A) or (B)) is unlikely. Neonatal acne (C) is typically confined to the face and peaks at about 2 months. Molluscum contagiosum (D) does not have erythema or pustules, it produces pearly growths with a central punctum.

The major criteria for rheumatic fever include all of the following features except? A. A new murmur B. Swollen right knee for the past 8 days C. A geographic shaped rash with central pallor on the abdomen D. Involuntary movements of the arms E. Fever

E. Fever Rheumatic fever is one of the post-infectious complications of group A streptococcal infection and classically occurs after an episode of tonsillitis. Diagnosis is made on the presence of two major, or one major with two minor, criteria, plus evidence of streptococcal infection such as a throat swab or positive antistreptolysin titre. Major criteria include: pancarditis such as endocarditis - murmur (A) or valvular dysfunction, myocarditits - heart failure, pericarditis - rub, effusion or tamponade; polyarthritis common in the knees (B), wrists, and ankles and may flit from joint to joint, lasting for more than 1 week in total (to distinguish this from reactive arthritis); Sydenham's chorea results in involuntary movements (D) starting 2-6 months after the infection; erythema marginatum (C) is an early feature which is a flat erythematous rash on the trunk or limbs that expands with a topographical map-like border and fades from the centre; subcutaneous nodules are painless pea sized nodules on the extensor surfaces. Minor criteria include fever (E); arthralgia; family or personal history of rheumatic fever; raised inflammatory markers; and ECG changes such as a prolonged PR interval.

You are asked to see a 2-year-old child with difficulty in breathing, a runny nose and a barking cough. His mother tells you he had a heart defect repaired as a baby and he still has a murmur. On examination he has noisy breathing with mild subcostal recession. He is apyrexial with a respiratory rate of 44 breaths per minute and heart rate of 152 beats per minute; capillary refill is 1-2 seconds. His throat is red and the tonsils are enlarged with no exudate. On his chest you see a midline sternotomy scar with a drain scar and a right thoracotomy scar. On auscultation the lung fields are clear, but he has an ejection systolic murmur in the left upper sternal edge which radiates to the back. He does not have a gallop rhythm. There are transmitted upper airway sounds only on the lung fields and the abdomen is soft with no organomegaly. What is the most appropriate management? A. Admit for IV antibiotics B. Give IV furosemide and admit C. Admit for observation D. Send home on oral antibiotics E. Give oral dexamethasone and observe

E. Give oral dexamethasone and observe This child has had corrective cardiac surgery and has a residual murmur of pulmonary stenosis. The most likely explanation for this acute presentation is croup, which should be treated with dexamethasone as for any other child. His breathing and observations should have improved before he goes home (E). This child does not have a fever and there is no pus on the tonsils to suggest a bacterial infection. Croup is largely due to viral upper respiratory infections, and dexamethasone should be all that is required. He does not need antibiotics ((A) and (D)). Admission for observation (C) would not be unreasonable as a tachycardic child should not be sent home without careful evaluation, but it is not the best answer because you should treat his respiratory distress due to the croup with dexamethasone (E) and admit if no improvement. He is not in heart failure (no gallop rhythm, no crackles in his lung fields, and no hepatomegaly) and he therefore does not need furosemide (B).

A 4-year-old girl has just returned from holiday in France where she visited a petting farm. She has had diarrhoea for 2 days, and her mother noticed fresh red blood mixed with the stools. She has also been vomiting. On admission to hospital her blood tests showed: Hb 5 g/dL, WCC 15×109/L, platelets 55×109/L, urea 19 mmol/L, creatinine 110 μmol/L. Her stool culture is pending. What is the most likely diagnosis? A. Platelet disorder B. Inflammatory bowel disease (IBD) C. Severe dehydration D. Henoch-Schönlein purpura (HSP) E. Haemolytic uraemic syndrome (HUS)

E. Haemolytic uraemic syndrome (HUS) HUS (E) is a microangiopathic haemolytic anaemia consisting of acute kidney injury, haemolytic anaemia and thrombocytopenia. It occurs secondary to Escherichia coli or Shigella gastroenteritis due to an endotoxin initiating an inflammatory response. Treatment is largely supportive fluid management but may require renal replacement therapy. Renal impairment may resolve but a few can develop chronic kidney disease. IBD (B) is unlikely in this age group but can present with bloody diarrhoea, anaemia, abdominal pain and weight loss. Severe dehydration (C) may result in a pre-renal cause of acute kidney injury leading to raised urea and creatinine (though the urea may be disproportionately high). However, this would not explain the abnormalities of the full blood count. Platelet disorders (A) can lead to mucosal bleeding including haematemesis, haematuria, per rectum (PR) or per vaginam bleeding, and epistaxis. However, it does not explain the renal impairment. HSP (D) is a syndrome consisting of purpuric rash, arthralgia, abdominal pain and renal involvement. It may present with PR bleeding and diarrhoea due to gut vasculitis and renal failure, but there is no evidence of rash or joint involvement.

A 2-year-old child is referred to hospital by the GP after his third visit that week; he now has a rash and the GP is worried he has meningitis. He has had a fever for 5 days up to 39.5°C or above every day and is not eating or drinking well. On examination, he has a temperature of 38.5°C, heart rate of 150, respiratory rate of 30 and is miserable. He has a blanching macular rash on his torso, swollen hands and feet, red eyes, red cracked lips, large tonsils with no pus, and a left-sided 2cm×3cm cervical lymph node which is mobile. There is no photophobia or neck stiffness. His chest is clear with normal heart sounds and his abdomen is soft with a palpable liver edge. You note his BCG scar is inflamed. What is the most likely diagnosis? A. Viral tonsillitis B. Bacterial tonsillitis C. Meningitis D. Hand, foot and mouth disease E. Kawasaki's disease

E. Kawasaki's disease The most likely diagnosis is Kawasaki's disease (E) as he fulfils the diagnostic criteria (5 out of 6): 1) fever for more than 5 days, 2) bilateral non-purulent conjunctivitis, 3) rash, 4) oral changes such as dry, cracked, red lips, strawberry tongue, 5) erythema and oedema of the hands or feet and 6) cervical lymphadenopathy. Inflammation at the BCG scar is common but is not a diagnostic criterion. Kawasaki's disease is a vasculitis of unknown aetiology that typically affects young children and is complicated by coronary aneurysm formation. Treatment is with intravenous immunoglobulin and high dose aspirin. By 5 days the fever caused by a viral tonsillitis (A) should be settling, even though other symptoms such as cough may persist for some time. This child could have a bacterial tonsillitis (B) and covering with antibiotics would be prudent, but it would be unusual to have conjunctivitis and swollen hands and feet with bacterial tonsillitis. You might expect to see pus on the tonsils if it is due to a bacterial infection. Hand, foot and mouth disease (D) produces a mild coryzal illness, mouth ulcers and painful papules on the hands and feet in young children and is caused by a coxsackie virus infection. He has no signs of meningism - neck stiffness or photophobia and his rash is blanching - making meningitis (C) or meningococcal septicaemia less likely.

A mother with known placenta praevia with heavy vaginal bleeding was rushed into the labour ward and delivered by emergency caesarean section at 35 weeks' gestation. Pre-delivery the fetus was bradycardic and after birth APGARs were three at 1 minute, five at 5 minutes and nine at 10 minutes. Thirty-six hours later on the special care baby unit the baby is irritable and requiring nasogastric tube feeds as he is not sucking well. The tone in his upper limbs is reduced and an EEG showed seizure activity which has been controlled by intravenous phenobarbitone. His cranial ultrasound is normal. His blood sugar monitoring is between 3.5 and 5mmol/L, C-reactive protein (CRP) was less than 5 mg/L and is 7 mg/L today. He is apyrexial. What is the most likely diagnosis? A. Intraventricular haemorrhage B. Group B streptococcal meningitis C. Hypoglycaemia D. Mild hypoxic ischaemic encephalopathy (HIE) E. Moderate HIE

E. Moderate HIE This child has experienced an hypoxic insult perinatally during his mother's haemorrhage and is now showing signs of HIE: poor feeding, altered tone and seizure activity on EEG. Mild HIE (D) presents with irritability, startle responses, poor feeding and hyperventilation. Moderate HIE (E) also has altered tone or reduced movement and seizure activity. While ischaemic changes early on may be difficult to see on cranial ultrasound, intraventricular haemorrhage (A) is easily seen and is associated with both hypoxic insults and a cause of seizures in the neonate. This child's cranial ultrasound was normal. Group B streptococcal meningitis (B) is a serious and life-threatening illness for the neonate and may cause seizures. It is therefore routine practice to treat unwell neonates with antibiotics, but the exposure for this infant is reduced as the delivery was a caesarean section and the CRP has been normal twice, making this an unlikely explanation for this illness. Hypoglycaemia (C) is also a cause of seizures in neonates but the definition is lower in this age group (below 2.5mmol/L) so this child is not hypoglycaemic.

A 3-year-old boy is brought in by ambulance fitting. You are assigned to get the history from the father. Harry is normally fit and well with no significant past medical history or allergies. He is up to date with his immunizations and has been growing and developing normally. His behaviour has been difficult for the past 2 weeks since the birth of his little sister. Mum has been unwell as she developed HELLP syndrome and was in hospital for a week following the delivery. Yesterday, he was quite unwell with a tummy bug, vomiting and had black diarrhoea. That evening they found a mess he had made in the bathroom with all of his mum's things strewn over the floor including her tablets from the hospital. By that time, Harry was getting better so they did not think anything of it. Today he has been acting strangely and has been difficult to understand, he then became lethargic at about 4 pm and started fitting 15 minutes ago. What is the most likely diagnosis? A. Paracetamol overdose B. Aspirin overdose C. Tricyclic antidepressant overdose D. Bleach intoxication E. Iron overdose

E. Iron overdose The answer is iron overdose (E) which classically is a two phase illness with early vomiting and diarrhoea due to gastric irritation and may present with haematemesis or malaena (the black diarrhoea). There is a period up to 24 hours of improvement and then deterioration with liver failure, drowsiness and coma. The liver failure can produce hypoglycaemia and seizures. The mother has recently had HELLP syndrome, liver failure, thrombocytopenia and hypertension in pregnancy. These patients often have significant haemorrhage and go home on iron supplements. Paracetamol overdose (A) in young children is often not in large doses due to the tablets being difficult to swallow; it may present with gastric irritation or a history that the child took some tablets, liver failure develops on day 3-5, our case presented on day 2 and you would not expect malaena from paracetamol intoxication. Tricyclic antidepressant (C) overdose would present much earlier than 2 days, with tachycardia, anticholinergic symptoms (dry mouth, blurred vision, agitation). Patients become shocked with seizures or coma and develop severe metabolic acidosis and this is one of the few indications for giving intravenous bicarbonate. Bleach intoxication (D) should be thought of as he was in the bathroom playing on his own; however problems are rare as the bleach does not taste nice and typically there will only be localized lesions where the bleach contacted the mucous membranes.

A 10-month-old baby boy is brought to accident and emergency with inconsolable crying. His mother says he is a miserable baby and even after feeding he does not settle. He has recently started to cruise around furniture, but is not yet walking. His crying has been worse today and both his parents had been awake all night due to his incessant crying. On examining the baby, you note that he is more upset when being handled and is a bit better when lying on his front. You do a chest x-ray which shows three posterior rib fractures; his mother states he fell down some steps yesterday. What is the likely diagnosis and appropriate management strategy? A. Birth trauma; no intervention necessary as they will heal spontaneously B. Accidental injury; ensure no pneumothorax present, reassure and discharge home C. Accidental injury; ensure no pneumothorax present and admit for observation D. Non-accidental injury (NAI); advise the parents you will refer to social services and discharge home E. NAI; discuss with social services and paediatric consultant and admit the child to a place of safety

E. NAI; discuss with social services and paediatric consultant and admit the child to a place of safety Posterior rib fractures are highly suspicious of NAI, hence (B) and (C) are less likely to be correct. This child is 10 months old and the injuries he has sustained are not consistent with his development or proposed mechanism. The baby may be inconsolable, particularly after feeding, if he has symptoms of gastro-oesophageal reflux. It may be inferred that the parents in their fatigue and frustration may be responsible for the injuries. If you suspect NAI, then a detailed history and examination are needed, discussion with a senior paediatrician and social services to fully investigate the situation to safeguard the child (E) while (D) is not a safe protocol to follow. Birth should not result in rib fractures (A), though clavicular fractures can be seen after shoulder dystocia or forceps deliveries.

Clara is a 14-year-old girl who was diagnosed with muscular dystrophy when she was younger. She now mobilizes in a wheelchair and other co-morbidities include a scoliosis and cardiomyopathy. She is being seen for her annual review in clinic. Which of these would best represent the respiratory complications of muscular dystrophy? A. Normal FVC, low FEV1/FVC ratio B. Flattened diaphragms on chest x-ray C. Morning dips in peak expiratory flow rate D. Extrathoracic obstruction on flow-volume loops E. Reduced FVC, normal FEV1/FVC ratio

E. Reduced FVC, normal FEV1/FVC ratio Muscular dystrophy causes a restrictive pattern of respiratory disease (E). This may be due to weak intercostals and diaphragmatic muscles and scoliosis, resulting in a reduced capacity for lung and chest wall expansion. Spirometry can be used in a clinic setting to assess lung function. (A) is an obstructive pattern as may be seen with asthma, and morning dips (C) are characteristic of asthma. Flattening of the diaphragms is typical of hyperinflation and air trapping, for example bronchiolitis (B). There is no indication from the history of an extrathoracic obstruction (D), for example chest pain, cough, or lymphadenopathy.

Forty per cent of children with trisomy 21 have congenital heart defects. Which of the following is not associated with Down's syndrome? A. Tetralogy of Fallot B. Atrioseptal defect (ASD) C. VSD D. Atrioventricular septal defect (AVSD) E. Transposition of the great arteries

E. Transposition of the great arteries Forty per cent of cardiac defects in children with Down's syndrome are AVSDs (D). Thirty per cent have a VSD (C), 10 per cent an ASD (B) and 6 per cent have Tetralogy of Fallot (A). While transposition of the great arteries (E) is often associated with ASDs or VSDs, it is not classically associated with trisomy 21 and is therefore the correct answer.

A 14-year-old girl went to her GP with a sore throat and cervical lymphadenopathy. She had a blood test done and you are called later that day with results. Haemoglobin 6.0g/dL, WCC 230×109/L, neutrophils 0.9×109/L, platelets 77×109/L; blood film showed blasts and Auer rods. What is the most important management priority for this child in the first 24 hours from diagnosis? A. Overwhelming sepsis B. Febrile neutropenia C. Heart failure D. Uncontrollable bleeding E. Tumour lysis syndrome

E. Tumour lysis syndrome She has a diagnosis of AML, as suggested by Auer rods on blood film. There is a very high WCC, predominantly due to blast cells in the bone marrow. This results in a reduction in the other cell line production producing a pancytopenia. Although anaemia can result in heart failure (C), low platelets can result in bleeding (D) and low neutrophils can result in increased susceptibility to infection (B) and overwhelming sepsis (A). The most important initial management step is to prevent tumour lysis syndrome. As the treatment for AML begins, a large number of cells break down which can cause complications such as hyperkalaemia, hyperphosphataemia, gout, fluid overload or dehydration, raised urea and creatinine, known as tumour lysis syndrome (E). Prophylactic hyperhydration, allopurinol or rasburicase and monitoring of electrolytes is required from the time of diagnosis.

Cruising, first word with meaning, drinks from cup. (Development) A Newborn B 2 months C 3 months D 6 months E 9 months F 1 year G 2 years H 3 years I 4 years J 5 years

F 1 year

Feeds using a spoon. May be dry during the day. (Developmental milestones) A 6weeks B 6months C 30months D 9months E 12months F 18months G 2years H 3years I atbirth J 5years

F 18months

Prader-Willi syndrome. (Inheritance) A Autosomal recessive B Automsomal dominant C X-linked recessive D Trinucleotide repeat E Polygenic inheritance F Imprinting G Chromosomal H X-linked dominant I Codominant J Mitochondrial

F Imprinting Imprinting. This is the phenomenon whereby mutations in the same genes cause different conditions based on whether the genes came from the mother or father.

A 1-day-old baby boy, born to a mother who had not booked in pregnancy, presents with seizures, a fever and poor feeding. On examination he is sweating and has a HR of 180bpm. (Collapsed neonate) A Coarctation of the aorta B Non-accidental injury C Sepsis D Inborn error of metabolism E Neonatal SVT F Neonatal abstinence syndrome G Pulmonary stenosis H TGA I Persistent pulmonary hypertension J AVSD

F Neonatal abstinence syndrome Neonatal Abstinence Syndrome. Withdrawal symptoms may occur from day 1 up to day 7. Symptoms include irritability, high pitched cry, sleep disturbance, seizures, yawning, sucking, sneezing, sweating, hiccups.

Extra digit. (Newborn examination) A Talipes equinovarus B Developmental dysplasia of the hip C Vernix D Cephalhaematoma E Epstein's pearls F Polydactly G Syndactly H Erb's palsy I Klumpke's palsy J Subaponeurotic haematoma

F Polydactly Polydactly can be isolated or associated with other abnormalities.

Palpable mass on thigh. (Tumours) A Acute lymphoblastic leukaemia B Lymphoma C Neuroblastoma D Wilm's tumour E Retinoblastoma F Rhabdomyosarcoma G Osteosarcoma H Ewing's sarcoma I Acute myeloid leukaemia J Glioma

F Rhabdomyosarcoma Rhabdomyosarcoma. Most common soft tissue tumour in childhood. Presentation is site dependent with palpable mass, local pain, and obstruction.

A 13-year-old African boy presents to A&E with a 2-month history of worsening right knee pain and walking with a limp. On examination he is obese and has limited hip flexion and abduction on the right side, with right thigh atrophy. (Limping child) A Developmental dysplasia of hip B Transient synovitis C Septic arthritis D Perthes' disease E Osteomyelitis F Slipped upper femoral epiphysis G Osgood-Schlatter's disease H Juvenile idiopathic arthritis I Non-accidental injury J Duchenne's muscular dystrophy

F Slipped upper femoral epiphysis Slipped upper femoral epiphysis is the displacement of the upper femoral epiphysis on the metaphysis. This is the most common adolescent hip disorder. Presentation consists of 2 broad types of children: 1) obesehypogonadal(delayedskeletalmaturationofboneagewithlowcirculating sex hormones) 2) tall, thin, post growth spurt (over abundance of growth hormones).

A newborn 2-day-old girl is found to have a well-circumscribed erythematous patch at the back of her neck. (Newborn skin problems) A Port wine stain B 'Strawberry' naevus C Congenital melanocytic naevus D Sebaceous naevus E Mongolian blue spot F Stork mark G Erythema toxicum H Staphylococcal skin infection I Milia J Transient neonatal pustular melanosis

F Stork mark Stork Mark. Also known as salmon patches. Very common (up to 50 per cent in newborns). Circumscribed lesions, usually at back of neck, nuchal fold, eye-lids, forehead and above upper lip. They fade over time and therefore parents need reassurance.

Down's syndrome. (Inheritance) A Autosomal recessive B Automsomal dominant C X-linked recessive D Trinucleotide repeat E Polygenic inheritance F Imprinting G Chromosomal H X-linked dominant I Codominant J Mitochondrial

G Chromosomal Chromosomal. Trisomy 21.

Asthma (Peripheral blood film) A Macrocytic RBCs B Microcytic RBCs C Normocytic RBCs D Spherocytes E Sickle cells F Neutropenia G Eosinophilia H Acanthocytes I Thrombocytosis J Thrombocytopenia

G Eosinophilia Eosinophilia is associated with allergic syndromes e.g. asthma, drugs, parasitic infection.

A 3-day-old Caucasian girl, who was born 10 days late, was brought into the GP with erythematous patches with small pustules in the centre. The patches varied in size and were found all over her body. (Newborn skin problems) A Port wine stain B 'Strawberry' naevus C Congenital melanocytic naevus D Sebaceous naevus E Mongolian blue spot F Stork mark G Erythema toxicum H Staphylococcal skin infection I Milia J Transient neonatal pustular melanosis

G Erythema toxicum Erythema toxicum. Present in >50 per cent of newborns in first 2 weeks. More common if term or post-term. Completely benign and parents just need reassurance.

A 10-year-old girl presents to A&E with colicky abdominal pain associated with diarrhoea. She is found to have dipstick haematuria and a rash on her buttocks. (Diarrhoea) A Toddler's diarrhoea B Hirschsprung's disease C Coeliac diasease D Inflammatory bowel disease E Intussusception F Haemolytic-uraemic syndrome G Henoch-Schönlein purpura (HSP) H Necrotizing enterocolitis (NEC) I Infective gastroenteritis J Food hypersensitivity

G Henoch-Schönlein purpura (HSP) Henoch-Schönlein purpura is a small vessel vasculitis associated with IgA immune complexes. The classic triad is of arthritis, colicky abdominal pain, and a palpable, papular, pupuric rash on the buttocks and lower legs. This condition commonly affects prepubertal boys.

A 4-year-old boy presents to A&E with difficulty in breathing. On examination he has a widespread wheeze, with moderate subcostal recession. He is able to talk in sentences and shows SpO2 96 per cent. (Treatment of respiratory distress) A Palivizumab B Dexamethasone C Cefotaxime D Erythromycin E Amoxicillin F Nebulized salbutamol G Salbutamol via inhaler H Nebulized adrenaline I IM adrenaline J IV adrenaline

G Salbutamol via inhaler Salbutamol via inhaler for moderate asthma.

Draws a circle, rides a tricycle, gives full name, asks lots of questions. (Development) A Newborn B 2 months C 3 months D 6 months E 9 months F 1 year G 2 years H 3 years I 4 years J 5 years

H 3 years

Flaccid arm with pronated forearm and flexed wrist. (Newborn examination) A Talipes equinovarus B Developmental dysplasia of the hip C Vernix D Cephalhaematoma E Epstein's pearls F Polydactly G Syndactly H Erb's palsy I Klumpke's palsy J Subaponeurotic haematoma

H Erb's palsy Erb's palsy. Commonest brachial plexus injury after birth. Caused by damage to C5-6 nerve routes. It may result from difficult assisted delivery (e.g. shoulder dystocia).

Fever stops as rash appears. Rose-coloured maculopapular rash initially on trunk and spreading peripherally. (Exanthems) A Varicella zoster B Parvovirus C Measles D Epstein-Barr virus E Group A Streptoccocus F Enteroviruses G Rubella H Human herpes virus 6 I Herpes zoster J Lyme disease

H Human herpes virus 6 Human herpes virus 6. Also known as roseola infantum it affects children usually before 2 years old. High spiking fever up to 41°C for up to four days that stops once rash appears. Supportive management. Common complication is febrile convulsion due to high spiking fever.

A 10-year-old girl presents to A&E with a 7-week history of left knee swelling associated with a limp. She is not suffering from any pain in the joint. She is ANA positive. (Limping child) A Developmental dysplasia of hip B Transient synovitis C Septic arthritis D Perthes' disease E Osteomyelitis F Slipped upper femoral epiphysis G Osgood-Schlatter's disease H Juvenile idiopathic arthritis I Non-accidental injury J Duchenne's muscular dystrophy

H Juvenile idiopathic arthritis Juvenile idiopathic arthritis. Common chronic childhood disorder. Diagnosis of exclusion in children with a history of at least six weeks of arthritis.

A 2-week-old baby boy born at 28 weeks gestation becomes unwell on the neonatal unit. He is no longer tolerating his feeds and is found to have blood and mucus in his stools. On examination he has a distended tender abdomen. (Diarrhoea) A Toddler's diarrhoea B Hirschsprung's disease C Coeliac diasease D Inflammatory bowel disease E Intussusception F Haemolytic-uraemic syndrome G Henoch-Schönlein purpura (HSP) H Necrotizing enterocolitis (NEC) I Infective gastroenteritis J Food hypersensitivity

H Necrotizing enterocolitis (NEC) Necrotising enterocolitis (NEC). The most common neonatal surgical emergency, mostly affects preterm infants (~90 per cent of cases) and is most common in second week after birth and rarely occurs before milk feeding. Early signs include vomiting, blood and mucus in stools and abdominal tenderness.

A term baby boy who is born following meconium-stained liquor develops respiratory distress. CXR shows patchy changes and overinflation. The SpO2 is 60 per cent on air. (Collapsed neonate) A Coarctation of the aorta B Non-accidental injury C Sepsis D Inborn error of metabolism E Neonatal SVT F Neonatal abstinence syndrome G Pulmonary stenosis H TGA I Persistent pulmonary hypertension J AVSD

I Persistent pulmonary hypertension Persistent Pulmonary Hypertension of the Newborn (PPHN). In this case meconium aspiration results in failure of the normal postnatal fall in pulmonary vessel resistance leading to ongoing right-to-left shunting across fetal channels (PFO and PDA) and resultant hypoxia.

Kawasaki's disease (Peripheral blood film) A Macrocytic RBCs B Microcytic RBCs C Normocytic RBCs D Spherocytes E Sickle cells F Neutropenia G Eosinophilia H Acanthocytes I Thrombocytosis J Thrombocytopenia

I Thrombocytosis Thrombocytosis. Leucocytosis in acute phase, thrombocytosis peaks in 3rd to 4th week with normocytic, normochromic anaemia and a low reticulocyte count.

An antenatally diagnosed Down's syndrome baby is grossly cyanotic after birth. A soft pansystolic murmur is heard at the lower left sternal edge. ECG shows right ventricular hypertrophy and right atrial enlargement. (Collapsed neonate) A Coarctation of the aorta B Non-accidental injury C Sepsis D Inborn error of metabolism E Neonatal SVT F Neonatal abstinence syndrome G Pulmonary stenosis H TGA I Persistent pulmonary hypertension J AVSD

J AVSD AVSD. Neonatal cardiac conditions can present with murmur, shock, heart failure and cyanosis. As in this case AVSD is commonly associated with Down's syndrome.

Raised intracranial pressure. (Tumours) A Acute lymphoblastic leukaemia B Lymphoma C Neuroblastoma D Wilm's tumour E Retinoblastoma F Rhabdomyosarcoma G Osteosarcoma H Ewing's sarcoma I Acute myeloid leukaemia J Glioma

J Glioma Glioma. Brain tumours are the most common solid tumours, accounting for 25 per cent of childhood malignancies.

Erythema migrans. (Exanthems) A Varicella zoster B Parvovirus C Measles D Epstein-Barr virus E Group A Streptoccocus F Enteroviruses G Rubella H Human herpes virus 6 I Herpes zoster J Lyme disease

J Lyme disease Lyme Disease. Caused by Spirochete Borrelia burgdorferi transmitted by a tick bite. The rash is an erythematous macule at the site of the tick bite that increases in size and forms a painless red lesion called erythema migrans.

Wiskott-Aldrich syndrome (Peripheral blood film) A Macrocytic RBCs B Microcytic RBCs C Normocytic RBCs D Spherocytes E Sickle cells F Neutropenia G Eosinophilia H Acanthocytes I Thrombocytosis J Thrombocytopenia

J Thrombocytopenia Thrombocytopenia. Eczema, thrombocytopenia, and immunodeficiency.

X-ray: A term Caucasian baby boy has a cyanotic episode during his first attempt at breast-feeding. He is admitted to the neonatal unit for investigation. The figure shows his chest X-ray. What is the diagnosis?

Oesophageal atresia.

X-ray: An African Caribbean baby boy was born at term after an uncomplicated pregnancy. Soon after delivery he developed respiratory distress and was admitted to the neonatal unit. When the junior doctor performed cannulation she noticed his arm break and therefore an X-ray of all four limbs was performed. a) What is the most likely diagnosis? b) What is the characteristic colour of sclerae in this condition?

a) Osteogenesis imperfecta. Note the multiple fractures in the lower limbs. b) Blue.

Diagnostic criteria of attention-deficit hyperactivity disorder include: a) Symptoms for minimum 6 months b) Symptoms for minimum 3 months c) Symptoms for minimum 1 month d) Onset before 3 years old e) Impairment in three or more functional domains or settings

a) True. b) False. c) False. d) False. 7 years old. e) False. Two or more.

Down's syndrome is commonly associated with: a) Atrioventricular septal defect b) Lymphoma c) Congenital hypothyroidism d) Anal atresia e) Inflammatory bowel disease

a) True. b) False. It is associated with leukaemia. c) False. It is associated with acquired hypothyroidism. d) False. It is associated with duodenal atresia. e) False. It is associated with coeliac disease.

Primary enuresis may be caused by: a) Drinking too close to bedtime b) Constipation c) Urinary tract infection d) Spina bifida e) Abuse

a) True. b) False. Secondary. c) False. Secondary. d) False. Secondary. e) False. Secondary.

Signs of a severe acute asthma attack include: a) Silent chest b) Talking in sentences c) Pre-nebulizer SpO2 92-94 per cent d) Low arterial carbon dioxide e) Thirst

a) True. b) False. Talking in words only. c) False. Pre-nebulizer SpO2 <90 per cent. d) False. A normal or high carbon dioxide is typical in severe asthma. e) False. In severe asthma, the child is too breathless to feed.

Features of autism include: a) Language delay b) Sociable personality c) Attentive d) Good eye contact e) Early pointing as a baby

a) True. b) False. There are poor social skills. c) False. The child is easily distracted. d) False. There is poor eye contact. e) False. There is poor interaction with parents.

Causes of non-communicating hydrocephalus include: a) Aqueduct stenosis b) Meningitis c) Subarachnoid haemorrhage d) Intraventricular haemorrhage e) Arnold-Chiari malformation

a) True. b) False. This is a feature of communicating hydrocephalus. c) False. This is a feature of communicating hydrocephalus. d) False. This is a feature of communicating hydrocephalus. e) False. This is a feature of communicating hydrocephalus.

Long-term treatment of sickle cell disease includes: a) Hydroxycarbamide b) Vitamin C c) Amoxicillin d) Avoiding over-hydration e) Hepatitis B vaccination

a) True. Increases HbF production. b) False. Folic acid (vitamin B) c) False. Penicillin V due to hyposplenism. d) False. Avoid dehydration. e) False. Pneumococcal, Haemophilus influenzae B, meningococcus A and C due to hyposplenism

Which of the following statements about child development are true? Select one or more: a. An infant of 3 months tends to hold their hands in the midline and demonstrate "hand regard" b. The Moro reflex is an example of a Primitive reflex c. At 12 months of age normal social development would include feeding with a spoon and mimicking adults d. At 9-months of age an infant will localise sound consistently, babble for self-amusement and initiate sounds e. Assessment of vision and fine motor development are closely related f. Gross motor development at 4-months can be assessed by "pulling to sit"

a. An infant of 3 months tends to hold their hands in the midline and demonstrate "hand regard" b. The Moro reflex is an example of a Primitive reflex d. At 9-months of age an infant will localise sound consistently, babble for self-amusement and initiate sounds e. Assessment of vision and fine motor development are closely related f. Gross motor development at 4-months can be assessed by "pulling to sit" The normal development to infant and young child can be usefully considered in different spheres ie gross motor, fine motor and vision, language and hearing, social and play. Whilst it is important to know the developmental milestones achieved by certain ages it must be appreciated that there is a wide normal range. The primitive reflexes are important as persistence suggests neurodevelopmental delay. It can be helpful to think of development in each of the developmental spheres in a sequential way - eg for gross motor development - initially lies prone or supine, pushes up on arms in prone, rolls over, can sit, pulls to stand, 'cruises with furniture' etc.

The current routine immunisation schedule given at 2 months of age includes immunisation against: Select one or more: a. Pertussis (Whooping cough) b. Meningitis C c. Haemophilus influenzae B d. Diptheria e. Polio f. Tetanus g. Measles h. Pneumococcus i. Rotavirus

a. Pertussis (Whooping cough) c. Haemophilus influenzae B d. Diptheria e. Polio f. Tetanus h. Pneumococcus i. Rotavirus Please visit http://www.immunisationscotland.org.uk/when-to-immunise/immunisation-schedule.aspx to get full details on the immunisation programme.

Associations of gastro-oesophageal reflux include: a) Upright positioning b) Cerebral palsy c) Early weaning d) High resting lower oesophageal sphincter pressure e) Rapid gastric emptying

b) True. a) False. It is associated with horizontal positioning. c) False. A liquid diet is associated with more reflux. d) False. It is associated with low resting lower oesophageal sphincter pressure. e) False. It is associated with delayed gastric emptying.

Prematurity is commonly associated with: a) Necrotizing fasciitis b) Patent ductus arteriosus c) Coarctation of the aorta d) Cataracts e) Diaphragmatic hernia

b) True. a) False. It is associated with necrotizing enterocolitis. c) False. d) False. Retinopathy of prematurity. e) False.

Urgent dialysis in acute renal failure is indicated if there is: a) Severe dehydration b) Severe hyperkalaemia c) Hypotension d) Hypocalcaemia e) Hyponatraemia

b) True. a) False. Symptomatic fluid overload. c) False. Uncontrollable hypertension. d) False. Only if symptomatic. e) False. Only if symptomatic.

Major criteria for rheumatic fever include: a) Raised C-reactive protein/erythrocyte sedimentation rate b) Fever c) Increased or rising antistreptolysin O titre d) Prolonged PR interval e) Previous history of rheumatic fever

b) True. a) False. These are minor criterion. b) False. These are minor criterion. c) False. These are minor criterion. d) False. These are minor criterion.

The acute management of diabetic ketoacidosis involves: a) Rapid rehydration b) Immediate treatment with insulin c) Nil by mouth d) Bicarbonate e) Phosphate

c) True. Patient is kept nil by mouth until no longer acidotic. a) False. Slow rehydration over 48 h. b) False. Insulin starts at least an hour after fluids started. d) False. There is no evidence for this. e) False. There is no evidence for this.

Management of nephrotic syndrome includes: a) Hyperhydration b) High protein diet c) Oral steroids d) Immunization with varicella zoster vaccine e) Prophylactic aciclovir

c) True. Prednisolone 60mg/m2/day for 4 weeks, then 40mg/m2/alternate days for 4 weeks, then stop. a) False. Fluid restriction. b) False. Low protein diet. d) False. Pneumococcal vaccine. e) False. Oral penicillin V.

The commonest cause of community-acquired septicaemia in children is: a) Escherichia coli b) Group B Streptococcus c) Neisseria meningitidis d) Streptococcus pneumoniae e) Haemophilus influenzae type B

c) True. a) False. b) False. d) False. e) False.

Bronchiolitis is most commonly caused by: a) Mycoplasma pneumoniae b) Parainfluenza c) Respiratory syncytial virus d) Chlamydia e) Adenovirus

c) True. a) False. b) False. d) False. e) False. These are, however, all possible agents responsible

Normal fluid requirement for a 25 kg child would be: a) 2500mL/day b) 1750mL/day c) 1600mL/day d) 1250mL/day e) 500mL/day

c) True. a) False. b) False. d) False. e) False. First 10 kg 100 mL/kg/day; second 10 kg 50 mL/kg/day; subsequent kilograms 20 mL/kg/day (i.e. 1000 + 500 + 100 = 1600 mL/day).

Management of hypospadias includes: a) Circumcision b) Urgent surgery in first week of life c) Surgical referral d) Antibiotic prophylaxis e) Catheterization

c) True. a) False. Advise parents not to circumcise child. b) False. Surgical referral is made at about 6 weeks of age. d) False. e) False.

Clinical features of congenital adrenal hyperplasia include: a) Metabolic alkalosis b) Hypokalaemia c) Hyponatraemia d) Reduced plasma 21-deoxycortisol levels e) Reduced plasma 17-hydroxyprogesterone levels

c) True. a) False. There is metabolic acidosis. b) False. There is hyperkalaemia. d) False. These are elevated. e) False. These are elevated.

Conjugated hyperbilirubinaemia is caused by: a) ABO incompatibility b) Breastfeeding c) Biliary atresia d) Hereditary spherocytosis e) Hyperthyroidism

c) True. a) False. Unconjugated. b) False. Unconjugated. d) False. Unconjugated. e) False. Congenital hypothyroidism.

Early management of hypoxic ischaemic encephalopathy includes: a) Hyperthermia b) Hyperhydration c) Fluid restriction d) Magnetic resonance imaging (MRI) scan e) Enteral feeds

c) True. a) False. Hypothermia. b) False. Fluid restriction. d) False. Optimum time for scan day 5-7. e) False. Enteral feeds are withheld for at least 48 h.

Causes of delayed puberty include: a) McCune-Albright syndrome b) Congenital adrenal hyperplasia c) Exogenous steroids d) Prader-Willi syndrome e) Craniopharyngioma

d) True. a) False. This is precocious puberty. b) False. This is precocious puberty. c) False. This is precocious puberty. e) False. This is precocious puberty.

Social work and police ask for your opinion after you have examined an eighteen months old boy with unexplained bruising. He has a non tender reddish blue bruise to his lower back. They want to know how old it is. What is your response? Select one: a. 3-5 days b. More than 10 days c. Unable to comment d. 6-10 days e. 0-2 days

c. Unable to comment Estimating the age of a bruise is notoriously difficult. There are few studies which have looked into this and systematic review on CORE gives a summary of findings. http://www.core-info.cardiff.ac.uk/reviews/bruising/ageing/results

The definition of cerebral palsy is: a) An acute disorder of movement and/or posture that presents early b) An acute disorder of movement and/or posture that presents late c) A chronic disorder of movement and/or posture that presents late d) A chronic disorder of movement and/or posture that presents early e) A chronic disorder of movement that presents early

d) True. a) False. b) False. c) False. e) False.

Croup presents with: a) No prodrome b) A fever >38.5 °C c) Drooling saliva d) A 'barking cough' e) An acute presentation over hours

d) True. a) False. Coryza occurs for 2-3 preceding days. b) False. Usually fever <38.5 °C is seen. c) False. The mouth is usually closed. e) False. Presents over a few days. All the others are presenting features of acute epiglottis.

Treatment of anaphylaxis includes: a) IV 1:1000 adrenaline b) IM 1:10 000 adrenaline c) IO 1:1000 adrenaline d) IM 1:1000 adrenaline e) Nebulized 1:10 000 adrenaline

d) True. a) False. IV 1:10 000 adrenaline is used only for cardiopulmonary arrest. b) False. c) False. IO 1:1000 adrenaline is used only for cardiopulmonary arrest. e) False. Nebulized 1:1000 adrenaline is used for the treatment of stridor.

Risk factors for neonatal respiratory distress syndrome include: a) Advancing gestational age b) Female sex c) Vaginal deliveries d) Sepsis e) Maternal drug misuse and smoking

d) True. a) False. Risk decreases with advancing gestational age, and is therefore rare in term babies. b) False. Risk is decreased in female babies. c) False. Risk is increased in caesarean section deliveries. e) False. Risk reduced with maternal drug misuse and smoking; but the risks associated with these far outweigh the benefits.

The UK infant immunization schedule includes: a) BCG b) Varicella zoster c) Hepatitis B d) Haemophilus influenzae B e) Meningococcus A

d) True. a) False. This is only given to high-risk groups. b) False. c) False. This is only given to high-risk groups. e) False. Meningococcal C.

Gross motor skills

• head lag evident when pulled to sitting - 0-6 weeks • rolls over - 8-18 weeks • holds head up (momentarily) when in sitting position - 4-6 weeks • can sit alone - 18-30 weeks • pulls self up to stand - 6-10 months • can walk by holding furniture - 7-13 months • walks alone - 11-14 months • kicks ball - 14-24 months • pedals tricycle - 18-38 months • hops, skips - 34-60 months

Which of the following statements about basic life support in a child of 5 yrs are true? Tick all that apply. Select one or more: a. The correct position for maximising the airway is the 'sniffing' position b. The correct position for chest compressions is over the lower half of the sternum c. The ratio of breaths to chest compressions is 2 breaths to 15 chest compressions d. Before giving rescue breaths you should check for signs of life e. Jaw thrust is the preferred method of opening the airway in a child with a suspected neck injury

a. The correct position for maximising the airway is the 'sniffing' position b. The correct position for chest compressions is over the lower half of the sternum c. The ratio of breaths to chest compressions is 2 breaths to 15 chest compressions e. Jaw thrust is the preferred method of opening the airway in a child with a suspected neck injury The correct sequence for basic life support in a child is: Safe approach, stimulate, shout for help, open airway, look/listen/feel for breathing, give 5 rescue breaths, check for signs of life, give 15 chest compressions, continue CPR at 2 breaths to 15 compressions for 1 minute and then call emergency services.

At what age (in years) would a boy be considered to have delayed puberty if he has no signs of secondary sexual development Select one: a. 9 b. 15 c. 14 d. 4 e. 7

c. 14 Pubertal delay is more common in boys and is defined as the absence of secondary sexual development in those aged 14 years or older. There are a number of potential causes for this, but the most common cause is an inherited tendency. The age at which girls are defined as having delayed puberty is 13 years or older.

A one-and-a-half-year-old Caucasian child is referred to paediatrics for failure to thrive. On examination he is a clean, well-dressed child who is quite quiet and withdrawn. He is pale and looks thin with wasted buttocks. His examination is otherwise unremarkable. What is the most likely cause of this child's growth failure? His growth chart shows good growth along the 50th centile until 6 months followed by weight down to the 9th, height down to 25th and head circumference now starting to falter at 1.5 years. (failure to thrive) A. Coeliac disease B. Neglect C. Constitutional delay D. Normal child E. Beta thalassaemia

A Coeliac disease The answer here is coeliac disease (A). The clue to this is that his growth was normal until the age of weaning, 6 months. With the introduction of gluten into the diet his growth began to falter. You also note he is pale, likely anaemic, suggestive of malnutrition. One of the classic signs of coeliac disease is the wasted buttocks. Neglect (B) should always be considered in any case of failure to thrive, especially in a child who seems withdrawn, but it is important to remember that chronic illness may make children listless and withdrawn and it is always important to rule out physical illness. You would need multidisciplinary input prior to making the diagnosis of neglect. Constitutional delay (C) is the isolated finding of delay in skeletal growth, i.e. height, and is typically seen around the time of puberty; as his growth failure started with weight this answer is incorrect. He is not a normal child (D) as he was born on the 50th centile and his weight is now on the 2nd centile with other parameters following in the traditional pattern of growth failure: weight followed by height, followed by head circumference. Beta thalassaemia (E) is highly unlikely in a Caucasian child. You would expect them to be pale and develop symptoms including growth failure around 6 months of age, but without transfusions he would be unlikely to survive to a year-and- a-half old.

A 12-year-old boy presents to A&E with right sided iliac fossa pain, nausea and vomiting. History from the GP shows that initially the pain was periumbilical. Blood tests show a raised white cell count and Rovsing's sign is positive. A appendicitis B coeliac disease C ulcerativecolitis D intussusception E Peutz-Jegherssyndrome F Crohn's disease G anorexia nervosa H constipation I pepticulcer J Wilms' tumour

A appendicitis In appendicitis, pain moves from the periumbilical area to the right iliac fossa as the peritoneum become inflamed. Other symptoms include a mild temperature and anorexia. Rovsing's sign is elicited (in cases of appendicitis) by palpation of the left iliac fossa which precipitates right iliac fossa pain.

A 6-year-old boy is brought to your clinic. His mother complains to you that he is scratching his skin all the time. Examination reveals patches of dry skin in the cubital fossae and popliteal fossae with scratch marks and lichenification. Further history reveals that the sister suffers from asthma and the mother suffers from hay fever. A atopic eczema B meningococcal septicaemia C alopecia D systemic lupus erythematosus E Henoch-Schönlein purpura F contact dermatitis G erythema multiforme H pompholyx I psoriasis J incontinentia pigmenti

A atopic eczema Atopic eczema. There is a strong family history of atopic disease and the patient is suffering from skin rash in flexural creases such as cubital fossae and popliteal fossae. Application of emollient and steroid cream can help control this troublesome condition.

A 15 year old with well-controlled type 1 diabetes presents with frank haematemesis. Her blood tests in accident and emergency show: pH 7.37, glucose 18.3 mmol/L, haemoglobin 12.3 g/dL, white cell count 5.3×10^9/L, neutrophils 2.1×10^9/L, platelets 165×10^9/L, Na+ 135 mmol/L, K+ 3.5 mmol/L, urea 5.0 mmol/L, creatinine 83 μmol/L, alanine transaminase 740 IU/l, bilisubin 96 μmol/L, alkaline phosphatase 102 IU/l, and albumin 25 g/L. Further investigations once she is stable on the ward show hepatitis B surface antigen negative, anti-hepatitis C virus negative, anti-nuclear antibody (ANA) 1:320 and anti-smooth muscle antibodies are positive. What is the most likely diagnosis? A. Autoimmune hepatitis with varices B. Metabolic ketoacidosis C. Gastroenteritis with a Mallory-Weiss tear D. Pregnancy with hyperemesis and a Mallory-Weiss tear E. Systemic lupus erythematosus (SLE)

A. Autoimmune hepatitis with varices It is important to know some of the associated autoimmune diseases that type 1 diabetics may suffer from. The most common are coeliac and Graves' disease which are screened for yearly. Autoimmune hepatitis (A) is more common in females and typically presents between the ages of 10 and 30 with either chronic liver failure or acute hepatitis. The ANA will be positive in approximately 80 per cent and the anti-smooth muscle antibody with be positive in approximately 70 per cent of cases. Her pH is normal, so she is not in ketoacidosis (B), and her glucose is raised as a stress response. While she would be isolated in a cubicle on admission in case of gastroenteritis (C), there is no history of diarrhoea and the investigations do not support an infectious cause with a normal white cell count and autoantibodies positive. There is nothing in the history to suggest she is pregnant (D); however it would be good practice to test the urine of any woman of child-bearing age with vomiting or abdominal pain. ANA would normally be positive in SLE (E) but anti-smooth muscle antibodies are associated with autoimmune hepatitis.

A 3-month-old baby is brought to accident and emergency because he has been vomiting and having diarrhoea for the past month. His mother breastfed him until he was 8 weeks old and he is now taking formula milk, 4-5oz every 4 hours. On examination he is alert but fussy and looks thin. He has eczema on his face, neck and torso and the mother says this is new. The abdomen is soft, the genitalia are normal with a significant nappy rash and the anal margin is erythematous. You plot his growth in his red book and find that he was born on the 50th centile and was following that but now he is on the 25th centile for weight. What is the most likely diagnosis? A. Cow's milk protein intolerance B. Lactose intolerance C. Gastroenteritis D. Hyper IgE syndrome E. Wiskott-Aldrich syndrome

A. Cow's milk protein intolerance In light of the eczema that has developed since breastfeeding cessation, plus the nappy rash and anal inflammation, the most likely explanation is cow's milk protein intolerance (A), an allergic reaction to the cow's milk proteins, which is different from lactose intolerance (B). Cow's milk protein intolerance typically presents with exposure to cow's milk in infancy, although in severe cases maternal ingestion of dairy products may affect a breastfeeding infant. The infant has worsening skin inflammation (eczema), vomiting, diarrhoea (which may be bloody), failure to thrive, irritability and colic. Lactose intolerance (B) is typically acquired following acute gastroenteritis. It is unusual in infancy, although rarely it can be congenital. It is a deficiency in the enzyme, lactose dehydrogenase, which breaks down lactose (the disaccharide sugar in milk). The build up of sugars in the gut results in an osmotic diarrhoea and dehydration. This history is too long for gastroenteritis (C) which could be associated with the introduction of bottle feeds if the equipment and water has not been sterilized. Hyper IgE syndrome (D), sometimes referred to as Job's syndrome, is an autosomal dominant immunodeficiency which is associated with severe eczema and skin boils but does not have significant gastrointestinal presentations. Wiskott-Aldrich syndrome (E) is also associated with eczema. This is an X-linked recessive disorder associated with thrombocytopenia, eczema and lymphopenia, but no significant gastrointestinal symptoms.

A pregnant woman is admitted to the labour ward for an elective caesarean section at 38 weeks for her baby who had an antenatal diagnosis of gastroschisis. The paediatric team are called to attend the delivery. The baby is born in good condition with no resuscitation required. He is taken to the neonatal unit for further care. Which of these is a complication of gastroschisis? A. Dehydration B. Hyperthermia C. Necrotizing enterocolitis (NEC) D. Fluid overload E. Hypernatraemia

A. Dehydration Gastroschisis refers to herniation of the bowel through a defect in the anterior abdominal wall of the developing fetus. It is not covered with a membrane as in exomphalos and is rarely associated with other anomalies. Gastroschisis can lead to fluid, electrolyte and heat losses. The baby will require fluid replacement and strict fluid balance assessment, placing in a heated incubator and regular electrolyte monitoring. Dehydration (A) is a serious complication, not overload (D), along with hypothermia and not hyperthermia (B). Hyponatraemia is more likely than hypernatraemia (E). NEC (C) is not more likely in children with gastroschisis.

An 8-year-old girl is brought to see you, having not opened her bowels in 8 days. She complains of hard painful stools and recurrent abdominal pain for the past 6 months but no vomiting. Her mother thinks that she is avoiding going to the toilet and reports that she has always been a bit irregular opening her bowels, averaging about twice a week. In her past medical history, she passed meconium on day 1 of life and has had no significant medical problems. On examination she is a well-looking, normally grown child. Her abdomen is soft with a palpable indentable mass in the left iliac fossa. The anus is normal, as are her lower limbs. What is the first step in management? A. Encourage her to increase her fluid intake, dietary fibre and exercise B. Introduce scheduled toileting with a positive reward scheme such as a star chart C. Refer for bowel disimpaction under anaesthesia D. Start polyethylene glycol with electrolytes such as Movicol E. Start a stimulant laxative such as senna

A. Encourage her to increase her fluid intake, dietary fibre and exercise This is a common presentation of constipation and on examination she has faecal impaction. There are no red flags present in the history such as growth failure, a history of delayed passage of meconium, distended abdomen with vomiting, anal pathology or neurological complications affecting the lower limbs. The correct first line management would be advice on increased fluid/fibre intake and exercise (A). Once her stools are softer, behavioural change management around toileting using star charts will help her overcome the toilet avoidance (B). If lifestyle changes do not produce softer stools or she develops vomiting, she may need medical bowel disimpaction with non-stimulant laxatives (D) but this would not be first line management. Stimulant laxatives (E) should not be used as they may produce more abdominal pain, dependence and have an associated risk of bowel perforation. Referral to the surgeons for bowel disimpaction under anaesthesia would be a last resort after failed medical management and underlying causes of constipation have been ruled out or treated. It is worth reviewing the NICE guidelines on paediatric constipation (www.nice.org.uk/nicemedia/live/12993/48754/48754.pdf).

A 3-year-old boy is brought into A&E with wheeze and increasing shortness of breath. A few days ago his mother said that he had an episode of choking but following this was 'fine'. Chest x-ray reveals unilateral hyperinflation. A foreign body B asthma C pulmonary haemorrhage D pneumothorax E respiratory distress syndrome F congenital heart disease G meconium aspiration syndrome H pneumonia I croup J respiratory syncitial virus

A. Foreign body The x-ray suggests collapse of a lung secondary to obstruction by a foreign body. Clinical examination may show decreased air entry or unilateral wheeze/added sounds. Early bronchoscopy is indicated for retrieval of the foreign body.

The accident and emergency triage nurse asks you to look at a 3-year-old child with a short history of waking up this morning unwell with a cough and fever. She looks unwell, heart rate is 165, respiratory rate 56, saturations of 96 per cent in air, temperature of 39.3°C and central capillary refill of 4 seconds. She has a mild headache but no photophobia or neck stiffness and you notice a faint macular rash on her torso and wonder if one spot is non-blanching. You ask the triage nurse to move her to the resuscitation area and call your senior to review her. Fifteen minutes later your senior arrives and the spot you saw on the abdomen is now non-blanching and there is another spot on her knee. What are the three most important things to give her immediately? A. High flow oxygen, IV fluid bolus, IV ceftriaxone B. IV fluid bolus, IV ceftriaxone, IV methylprednisolone C. High flow oxygen, IV ceftriaxone, IV fresh frozen plasma D. IV fluid bolus, IV ceftriaxone, IV fresh frozen plasma E. High flow oxygen, IV ceftriaxone, IV methylprednisolone

A. High flow oxygen, IV fluid bolus, IV ceftriaxone The correct answer is (A): she has presented with a classic presentation of meningococcal sepsis. Do not forget to always give sick children oxygen; if the mask makes them more distressed a minimum of wafted high flow O2 may suffice. The child has a prolonged capillary refill and is tachycardic; she should have a fluid bolus (20mL/kg) and be reassessed. As soon as intravenous or intraosseous access is obtained she should be given IV ceftriaxone, but only after the blood culture has been taken. (B) and (E) are incorrect as there is no mention of the child having meningitis. Currently there is no formal consensus as to whether steroids should be routinely used but administration before or with the first dose of antibiotics has been shown to be beneficial for some types of bacterial meningitis. (C) and (D) are incorrect as you do not yet have clotting results. She will likely need fresh frozen plasma to correct her disseminated intravascular coagulation which is evidenced by the forming non-blanching rash.

A 14-year-old girl presented to the GP with an enlarged lymph node in her neck. She first noticed it 3 weeks ago and it is increasing in size. She has also had a dry cough, fevers, night sweats and weight loss. She has had a poor appetite over the last 2 weeks, which her mother blames for her weight loss. There is no history of foreign travel or tuberculosis (TB) contacts. A chest x-ray shows a mediastinal mass. What is the most likely diagnosis? A. Lymphoma B. Pneumonia C. TB D. Lung tumour E. Leukaemia

A. Lymphoma There is a history of prolonged fever, which must raise the possibility of diagnoses other than infectious causes. There are no TB contacts and the x-ray does not show a focal collection, hence (C) and (B) are unlikely. The presence of a rapidly enlarging lymph node should prompt investigation for infection and malignancy. Primary lung cancer is rare in children, hence (D) is unlikely. Both (A) and (E) can present with lymphadenopathy and should be excluded with a lymph node biopsy and blood film/bone marrow aspirate. However, with the history of dry cough caused by a mediastinal mass and 'B' symptoms of fevers and night sweats, this should point towards (A) as the most likely diagnosis.

A 15-year-old Asian girl with Down's syndrome came to accident and emergency with a prolonged fever. She has severe learning difficulties and was difficult to assess. Her parents think she is more unsettled than usual and not eating and drinking properly for the last 3 weeks. She is admitted as you cannot confidently find the source of the infection, but she has no cough, rash, vomiting, diarrhoea or meningism. The next day she complains of a headache and starts to vomit. She has a CT scan which is normal and then a lumbar puncture (LP). White cell count (WCC) 150×109/L (20 per cent neutrophils), red blood count 0, protein 2 g/L, glucose 1.2 mmol/L (serum glucose 6.0 mmol/L). What is the most likely cause of this meningitis? A. Mycobacterium tuberculosis B. Herpes simplex virus (HSV) C. Streptococcus pneumoniae D. Cryptococcus neoformans E. Neisseria meningitidis

A. Mycobacterium tuberculosis The insidious onset of this case must raise the possibility of tuberculous meningitis (A). Although (C) and (E) are the most common causes of meningitis in the UK, the LP result and insidious nature make them less likely. An LP in these causes of meningitis may show raised white cells, normal protein and a glucose less than two-thirds of the serum glucose. Cryptococcus neoformans (D) is more likely in immunosuppressed patients, for example with coexisting HIV infection, which is not noted in this case. HSV results in encephalitis, which clinically is difficult to distinguish from bacterial meningitis so should be treated until you have confirmation that HSV is not present in the cerebrospinal fluid, which can now be done by laboratory test HSV polymerase chain reaction.

A mother brings her 2 year old to see you. She is very worried that he always has diarrhoea or loose stools. He eats a normal diet, and no particular foods seem to upset him but he often still has bits of vegetables or food he has eaten visible in the stool. She thinks he is losing weight and he is starting to potty train, so she is concerned this will affect his ability to anticipate needing the toilet. On examination he is an alert and well-looking child with a normal capillary refill, heart and respiratory rate. His abdomen is soft with no masses, there is no evidence of wasting and his weight and height are following the 50th centile. What is the most appropriate management? A. Reassure the mother, explaining this is toddler's diarrhoea and he will grow out of it B. Start loperamide as toddler's diarrhoea is affecting his toilet training C. Refer for endoscopy and biopsy to rule out coeliac disease D. Refer for a colonoscopy and biopsy for inflammatory bowel disease E. Order a blood test for thyroid function to rule out hyperthyroidism

A. Reassure the mother, explaining this is toddler's diarrhoea and he will grow out of it This is toddler's diarrhoea which is a common cause of loose stool in pre-school aged children and almost always requires no treatment (A). It is likely to be related to immature development of intestinal motility and it resolves in most children by the age of 5. Rarely, if the diarrhoea is socially disruptive to the child, it can be treated with loperamide (B), but he is only just starting to potty train and should be given a chance to do so. 'Significant social disruption' would be considered if he has failed to potty train and was unable to start school. He is a well-grown child with no evidence of wasting, so coeliac disease (C) is unlikely and the first investigation for this would be a blood test for anti-endomesial antibodies, not endoscopy. Inflammatory bowel disease (D) would be rare in a child this young, and there is no mention of abdominal pain, passage of blood and mucus or any suggestion of growth failure. While diarrhoea is a common symptom of hyperthyroidism (E), it is more commonly seen in female teenagers or in the neonatal period in infants born to mothers with Graves' disease. He is growing normally and has no other features of hyperthyroidism such as restlessness, increased appetite, sweating or tachycardia.

3. Blood pressure management A 12-year-old girl presents to her GP with a UTI. She has no past medical history of note and is not taking any medication. On testing her routine observations, her blood pressure was 140/90 mmHg with a manual sphygmomanometer. You are concerned this may be high for her age. She has no headaches, visual disturbance, vomiting, chest pain, dyspnoea or neurological signs. What is your next course of action? (BP management) A. Repeat the blood pressure on three different occasions B. Discuss the blood pressure reading with a paediatric nephrologist C. Commence sodium nitroprusside D. Repeat the blood pressure measurements with an automated machine E. Discharge home with reassurance

A. Repeat the blood pressure on three different occasions Blood pressure measurements in children need the same considerations as for adult patients. There may be external factors influencing the reading such as pain, anxiety, incorrect cuff size and poor technique. Manual measurements are preferable to automated ones (D). The blood pressure should be repeated on at least three occasions (A) and if still high investigations and management should be initiated with guidance from paediatric nephrologists (B). Sodium nitroprusside (C) is the treatment of choice for malignant hypertension, which is not consistent with the history. Causes of hypertension in children include: essential hypertension, renal (renal artery stenosis, chronic kidney disease, Wilms' tumour), cardiac (coarctation of the aorta), endocrine (Cushing's syndrome, phaeochromocytoma, neuroblastoma) and metabolic (hyperaldosteronism, congenital adrenal hyperplasia). Therefore, simple discharge home is not acceptable without considered exploration (E).

A 15-year-old boy was diagnosed with Down's syndrome at birth. He is short for his age, had cardiac surgery as a baby, has treatment for hypothyroidism and now attends mainstream school with some support. His parents are enquiring now about what complications he faces. Which of these is not a recognized complication of Down's syndrome? A. Retinoblastoma B. Atrioventricular septal defect (AVSD) C. Type 1 diabetes D. Leukaemia E. Alzheimer's disease

A. Retinoblastoma Down's syndrome affects many systems. Cardiac: ventricular septal defect, AVSD (B) and Tetralogy of Fallot. Endocrine: hypothyroidism, Addisons disease, type 1 diabetes (C). Ocular: cataracts, but not retinoblastoma (A). Malignancy: leukaemia (D). Gastrointestinal: duodenal atresia, Hirschprung's disease. Musculoskeletal: atlanto-axial instability. They are also at risk of Alzheimer's dementia (E) in later life.

A term baby is awaiting his discharge check when you are called to see him at 10 hours of age. His mother reports that he has turned a dusky colour and is not as alert as he has been. On examination he has central cyanosis, pulse 150bpm regular, and both brachial and femoral pulses are palpable. He has normal heart sounds with no murmur. His oxygen saturations are 65 per cent in air. What is the most likely underlying diagnosis? A. Transposition of the great vessels B. Ventricular septal defect (VSD) C. Tetralogy of Fallot D. Aortic stenosis E. Coartation of the aorta

A. Transposition of the great vessels This child has a cyanotic defect affecting the oxygenation of the blood. Transposition of the great vessels (whereby the aorta is attached to the right ventricle and the pulmonary artery attached to the left ventricle) (A) and the Tetralogy of Fallot (multiple defects including VSD, right ventricular outflow tract obstruction, right ventricular hypertrophy and overriding aorta) (C) are both cyanotic presentations but the Tetralogy of Fallot more commonly presents at around 6 months of age with cyanotic spells. Transposition of the great vessels is more likely to present at birth and is only compatible with life if there is a mixing defect in addition (VSD, atrial septal defect, persistent ductus arteriorus). In this child the ductus arteriosus helps to shunt blood to the lungs until it starts to close physiologically. He then is at risk of worsening cyanosis. He needs a prostaglandin infusion to keep the duct open and surgical intervention. The other answer options (B) (VSD is a communication between the left and right ventricles allowing left-to-right shunting), (D) (narrowing of the outflow tract of the left ventricle) and (E) (narrowing of a section of the aorta) are acyanotic and therefore do not account for this presentation.

A 14-year-old girl comes to see you as she has not had her periods yet. You note that her breasts are stage II and her nipples are set lateral to the mid-clavicular line. She has no pubic hair. Her weight is on the 50th centile but height is on the 9th centile. Her parents are both of average height. What is the most likely diagnosis? (menstrual cycle delay) A. Turner's syndrome B. Polycystic ovary syndrome C. Anorexia D. Constitutional delay E. Underlying undiagnosed chronic illness

A. Turner's syndrome The answer is Turner syndrome (A) (45,X), which may go undiagnosed in many women until they try to start a family. It is associated with lymphoedema of the hands and feet as a neonate, webbing of the neck, short stature, wide spaced nipples, congenital heart defects (coarctation of the aorta) delayed or absent puberty and infertility. Polycystic ovarian syndrome (B) would be unlikely in a girl who has not started puberty yet. She is of average weight so this is not anorexia (C). Constitutional delay (D) is much more common in males and is the isolated finding of delay in skeletal growth, i.e. height. A chronic illness (E) significant enough to delay puberty would also be associated with failure to thrive.

A 15-year-old boy comes to see you, complaining of recurrent abdominal and back passage pain relieved by passage of diarrhoea. He is also complaining of low back and knee pain and last week there was blood mixed into his stool. He has been losing weight recently. On examination he is slim and looks pale. His abdomen is soft but tender in the left iliac fossa with no masses. What is the most likely diagnosis? A. Ulcerative colitis B. Crohn's disease C. Coeliac disease D. Gastroenteritis E. NEC

A. Ulcerative colitis Ulcerative colitis (A) can affect any age but is rare in early childhood and increases in incidence with age. It is a recurrent inflammatory ulcerating disease of the mucous membrane layer of the rectum which spreads proximally to involve the colon. It classically presents with abdominal pain with blood or mucus mixed into diarrhoea. It is associated with erythema nodosum, pyoderma gangrenosum, arthritis and spondylitis, which may explain his back and knee pain. Crohn's disease (B) can be difficult to distinguish from ulcerative colitis but it is not commonly associated with spondylitis suggested by the back pain. The main distinction is on histology with Crohn's disease showing non-caseating granuloma and full thickness lesions. Coeliac disease (C) usually presents with malnutrition and abdominal bloating. Diarrhoea may be a presenting feature but blood would be unusual and would be digested, i.e. dark stools. This is a more chronic story than you would expect for gastroenteritis (D) and the arthritis does not tie in. NEC (E) is predominately an illness of the premature infant presenting with abdominal distension and bile stained aspirates.

Which child should be moved to the resuscitation area for urgent management in accident and emergency? A. A miserable 2 year old with a fever and vomiting temperature of 38.5°C, heart rate of 150, respiratory rate 42, capillary refill time 2-3 seconds who is alert and clinging on to his father and has just been given paracetamol and started on a fluid challenge with oral rehydration salts 5 minutes ago by the triage nurse B. A quiet 4 year old brought in with an asthma attack who is sitting upright with a respiratory rate of 50, heart rate of 162, capillary refill time of 3 seconds, subcostal recessions and poor air entry on chest auscultation following a salbutamol nebulizer C. An 8 year old, known diabetic, brought in vomiting with her glucose reader saying HI. She is able to tell you her history and has a heart rate of 120, respiratory rate of 25, capillary refill time of <2 seconds D. An alert 3 year old who has had a cough and cold for the past 3 days which is keeping him up at night and mum noticed a rash on his neck and face which did not disappear when she pressed a glass tumbler against it. His temperature is 37.8°C, heart rate is 110, respiratory rate is 30, capillary refill time is <2 seconds E. A 15 year old, known to social services for a family history of domestic abuse, brought in to accident and emergency by her best friend after she admitted to taking 20 paracetamol tablets 4 hours ago. She is alert but does not make eye contact, her heart rate is 98, respiratory rate is 20, capillary refill <2 seconds

B Child (B) is the sickest here and should be moved to the resuscitation area with a dedicated nurse and doctor to manage her. She is having a severe asthma attack: she is tachycardic and tachypnoeic with circulatory compromise, a prolonged capillary refill time and she has not responded to initial management. Child (A) is unwell with tachycardia, tachypnoea and a slightly prolonged capillary refill time but is pyrexic and has just been given treatment. He should be reviewed in 30 minutes to see that his parameters are improved. Child (C) is at risk of diabetic ketoacidosis and should be assessed quickly with a blood gas but she remains alert and orientated with no evidence of circulatory compromise so she is stable enough to remain in general paediatric accident and emergency, but it important to be aware that these patients can get very sick. Patient (D) has a non-blanching rash in the superior vena caval distribution and has normal observations. This is unlikely to be meningococcal sepsis and more likely to be a petechial rash related to the pressure of coughing; however any non-blanching rash should be reviewed by a senior clinician and should be isolated in a side room in accident and emergency. Patient (E) also has the potential to become quite sick but she is currently stable and may need privacy for talks with the mental health team once she is medically cleared; she would benefit from a side room if it is free.

Will be friendly to all individuals. Drinks from a cup if held to his lips. (Developmental milestones) A 6weeks B 6months C 30months D 9months E 12months F 18months G 2years H 3years I atbirth J 5years

B 6months

A 15-year-old girl presents with a 10-week history of diarrhoea, abdominal pain, bloating and weight loss. She also complains of an itchy skin rash at the back of her elbow. Blood tests reveal a positive result for IgA anti-endomysial antibody. A appendicitis B coeliac disease C ulcerative colitis D intussusception E Peutz-Jeghers syndrome F Crohn's disease G anorexia nervosa H constipation I peptic ulcer J Wilms' tumour

B coeliac disease Coeliac disease is an inflammatory intestinal disorder due to sensitivity to gluten. Dermatitis herpetiformis is an autoimmune blistering skin rash often found on the extensor surfaces of joints and is the classic skin manifestation of coeliac disease. IgA anti-endomysial antibody or IgA anti-tissue transglutaminase antibodies (tTGAs) are the usual tests for coeliac disease. If positive, patients are sent for biopsy confirmation.

A young mother brings her 4-year-old daughter to see you. You note that she has a widespread purple blotchy rash throughout her body. She is pyrexial and turns her head away from the light. Pressing a glass against the rash does not cause it to fade. A atopic eczema B meningococcal septicaemia C alopecia D systemic lupus erythematosus E Henoch-Schönlein purpura F contact dermatitis G erythema multiforme H pompholyx I psoriasis J incontinentia pigmenti

B meningococcal septicaemia Meningococcal septicaemia. The 'glass test' does not cause the rash to fade in meningococcal septicaemia. Swift action is required as patients deteriorate rapidly. In the community, GPs must give intramuscular antibiotics before transport to hospital.

You see a baby for the first baby check at 6 weeks. Mum reports no problems and he is feeding well. On examination you are unable to palpate the testicles on ether side and do not feel any lumps in the groin area. He has a normal penis with no hypospadias and the anus is patent. He is otherwise a normal baby on examination. What is the most important diagnosis to rule out? (Neonatal examination) A. Klienfelter's syndrome B. Congenital adrenal hyperplasia C. Undescended testicles D. Virilized female infant E. Testicular cancer

B. Congenital adrenal hyperplasia The correct answer is congenital adrenal hyperplasia (CAH) (B), which is most commonly due to 21-hydroxylase deficiency. These infants may present with ambiguous genitalia or bilateral undescended testicles and are at risk of a salt-losing adrenal crisis (vomiting, weight loss, floppy unwell infant), typically around 1-3 weeks of age. It is therefore important to measure urea and electrolytes in addition to chromosomal analysis and a pelvic ultrasound to look for the location of the sex organs. This baby could be a virilized female (D) infant secondary to CAH but the important diagnosis is the underlying cause for the virilization. Undescended testicles (C) are important to identify as they should be followed up to ensure they do descend by 2 years of age. If they remain out of the scrotum they are at increased risk of developing testicular cancer (E) but this would not be a diagnosis found at this stage. Klinefelter's syndrome (A) has a karyotype 47XXY and typically presents as tall stature, delayed puberty and possibly mild learning difficulties but not undescended testicles.

10-year-old boy presents with recurrent mouth ulcers, abdominal pain, distension and frequent episodes of diarrhoea with mucus. He has been losing weight. On examination he is slim and plotting his growth shows a fall in weight from the 50th centile to below the 9th. His abdomen is soft with generalized discomfort on deep palpation but no masses are present. What is the most likely diagnosis? A. Ulcerative colitis B. Crohn's disease C. Coeliac disease D. Gastroenteritis E. NEC

B. Crohn's disease Inflammatory bowel disease (IBD) can be difficult to separate clinically into Crohn's and ulcerative colitis. A younger child with IBD is more likely to have Crohn's disease (B), although its incidence increases with age. He has mouth ulcers which would not go with ulcerative colitis (A). Ulcerative colitis is a rectal disorder with proximal spread. Coeliac disease (C) would normally present in young children as it is associated with the weaning process and ingestion of wheat, although the diagnosis should be considered in all children with growth failure. This is a more chronic story than would be expected for gastroenteritis (D) which should not produce failure to thrive. NEC (E) is predominately an illness of the premature infant presenting with abdominal distension and bile stained aspirates.

A 4-year-old child has been losing weight recently and has been vomiting for the past 24 hours, unable to eat anything. His mother has brought him into accident and emergency out of concern as he seems confused. The triage nurse has taken him to the resuscitation room and asked for your help. On examination he is drowsy, has a heart rate of 150, respiratory rate of 60 and a central capillary refill of 5 seconds. He has subcostal recessions and good air entry bilaterally with no added sounds. He moans when you examine his abdomen but there are no masses. You put in a canula and take bloods. The venous blood gas shows: pH 7.12 PCO2 2.3 kPa PO2 6.7 kPa HCO3-15.3 mmol/L BE -8.6 Glucose 32.4 mmol/L What is the most likely diagnosis and what is the first management step? A. Diabetic ketoacidosis, start an insulin infusion B. Diabetic ketoacidosis, give a fluid bolus C. Pneumonia, start IV co-amoxiclav D. Ruptured appendix, give a fluid bolus and book the emergency operating theatre E. Gastroenteritis with severe dehydration, give a fluid bolus

B. Diabetic ketoacidosis, give a fluid bolus This child has new-onset, type 1 diabetes; he has been losing weight and the blood gas shows a very high glucose and a metabolic acidosis (remember that in paediatrics, arterial gases are rarely used, and that a venous PO2 will be low; this gas does not show respiratory failure.) The chest is clear on examination and the gas does not show a respiratory acidosis; therefore this is not pneumonia (C). The first management for him is rehydration (B). Only after an hour of rehydration should he be started on intravenous insulin. It is important to correct his dehydration and hyperglycaemia slowly over 2 days due to the risk of brainstem demylination with rapid shifts in salts in the cerebrospinal fluid; therefore (A) is incorrect. Appendicitis (D) or gastroenteritis (E) would be reasonable if the glucose were normal. In the context of a tachycardia with prolonged capillary refill, a patient should be given a fluid bolus and reassessed. Patients should be stabilized before going to the operating theatre.

A 6-year-old boy with a history of anaphylaxis to peanuts is brought in by ambulance unconscious. He was attending a children's birthday party. His mother says there was a bowl full of candy and he may have eaten a Snickers bar but she is not sure and she did not have his EpiPen with her. His face and lips are swollen and erythematous, he is still breathing but weakly and there is wheeze. His pulse is tachycardic and thready. Which type of shock is this? A. Hypovolaemic B. Distributive C. Septic D. Cardiac E. Obstructive

B. Distributive Shock is inadequate perfusion of tissues which is insufficient to meet cellular metabolic needs. This child presents with anaphylaxis, but did not have early intramuscular adrenaline to prevent the capillary leak of fluid into his airway tissues. Due to oedema his airway is closing off. The rapid shifts in fluid to the interstitium results in intravascular hypovolaemia and shock. This is distributive shock (B). Hypovolaemic shock (A) is due to haemorrhage or dehydration, and the patient would be pale, cool and poorly perfused. In septic shock (C) you would expect a fever and history suggesting infection. In cardiac shock (D) the heart is unable to meet the circulatory demands of the body, resulting in shock. There is chest pain or other cardiac symptoms and this is rare in childhood outside of congenital heart or Kawasaki's disease. Lastly, obstructive shock (E) is due to blockage of blood flow from the heart, either due to cardiac tamponade, fluid in the pericardial sac compressing the heart or tension pneumothorax.

A 20-month-old boy has been referred due to delayed walking. On further questioning you establish he has no difficulty feeding, had head control at 3 months of age, and sat up by 8 months. He has been crawling for the last 8 months, but he does not pull to stand or walk with support. He has no dysmorphic features. There is no known family history of muscle problems. His mother has no myotonia. His mother is very concerned and asks you what is wrong. What is the most likely diagnosis? A. Myotonic dystrophy B. Duchenne's muscular dystrophy C. Down's syndrome D. Myasthenia gravis E. Becker's muscular dystrophy

B. Duchenne's muscular dystrophy Delayed walking is a common paediatric presentation. Of the causes listed, Duchenne's muscular dystrophy (B) is the most common, with Becker's muscular dystrophy (E) being less severe but a similar presentation. Both are due to defects in the dystrophin gene and are inherited in an X-linked manner. Myotonic dystrophy (autosomal dominant) (A) and Down's syndrome (C) may present with hypotonia from birth, both centrally and peripherally, though children with Down's syndrome may have other features including dysmorphic facies (low set ears, epicanthic folds, protruding tongue, flattened nasal bridge), single palmar crease, sandal gap toes. Myasthenia gravis (D) can present with delayed walking, but fatigability is a key sign that differentiates it from the others.

A 5-day-old baby who is formula fed is on the neonatal unit being treated for sepsis secondary to an Escherichia coli urinary tract infection. He has been on antibiotics for 5 days. He is still unwell and vomiting. The parents are consanguineous and this is their first child. He has had repeat blood and urine cultures taken. Urine reducing substances are positive. What is the most likely underlying diagnosis? A. Fructose intolerance B. Galactosaemia C. Phenylketonuria D. Lactose intolerance E. Glycogen storage disease

B. Galactosaemia Galactosaemia (B) is due to a deficiency in galactose-1-phosphate uridyl transferase. It results in illness with lactose-containing milks, with vomiting, cataracts and recurrent episodes of Escherichia coli sepsis. Fructose intolerance (A) can also produce a metabolic acidosis and vomiting, but is distinguished from galactosaemia by the E. coli sepsis. Phenylketonuria (C) presents with developmental delay when the child is older and a musty smelling urine, not acute illness and sepsis. Lactose intolerance (D) is unlikely as lactase levels are usually high at birth and may present with diarrhoea and poor weight gain. Glycogen storage diseases (E) are a heterogeneous group of disorders that present with liver, muscle and cardiac defects.

A 7-year-old boy presented to accident and emergency with diarrhoea and vomiting for the past week. He had no history of foreign travel and but had been to a zoo recently on a school trip. He was discharged home, after providing a stool sample, with rehydration advice as he was less than 5 per cent dehydrated and tolerating oral fluids. The stool had grown 'Escherichia coli 0157' which was phoned from the microbiology laboratory to the on-call doctor 48 hours later. What is the most serious complication? (post-infection complications) A. Acute kidney injury B. Haemolytic uraemic syndrome C. Severe hypernatraemic dehydration D. Henoch-Schönlein purpura E. Post-gastroenteritis syndrome

B. Haemolytic uraemic syndrome Haemolytic uraemic syndrome (B) is a serious complication of particularly E. coli and Shigella gastroenteritis. It is caused by a verocytotoxin produced by these bacteria which initiates an inflammatory reaction in vascular endothelium and neutrophil activation. It is recognized by a triad of microangiopathic haemolytic anaemia, acute kidney injury and thrombocytopenia. The morbidity and mortality results from the renal impairment which may require dialysis or progression to chronic kidney disease (A). Children with severe gastroenteritis of any cause may suffer from dehydration and if water loss is greater than salt loss, this may cause hypernatraemia (C). Children may also suffer with food intolerances (i.e. lactose intolerance) after an episode of gastroenteritis due to damage to the mucosal enzyme activity (E). Henoch-Schönlein purpura often follows a viral infection and is characterized by arthralgia, purpuric rash on the extensor surfaces of the lower limbs, abdominal pain and renal involvement (D).

You are on elective in Uganda and spending the day on the paediatric ward. You are told that it is the rainy season and malaria is now becoming increasingly problematic. Almost all the children on the ward are suffering with the effects of malaria. The first child is a 5-year-old boy with a cyclical fever, abdominal pain and a 4cm splenomegaly. He has 2 per cent parasitaemia on blood film. You are asked how you would treat this child. What is the best initial management step? A. IM quinine B. IV fluids and IV quinine C. IV fluids and prophylactic splenectomy D. Emergency splenectomy E. Oral atovaquone

B. IV fluids and IV quinine This child has moderate parasitaemia. This child requires antimalarial treatment, IV in the first instance (B) and when he is improving this could be changed to oral (E). With splenomegaly secondary to acute haemolysis he should not have a splenectomy (C) or (D) unless there is evidence of a ruptured spleen; signs include hypovolaemic shock and peritonism. The first choice of antimalarial depends on the local resistance of the mosquitoes. This can be checked on national websites if the patient has just returned from travelling in endemic areas. It is also important to know what prophylaxis, if any, was taken and duration of the course.

A baby is born and you are asked to do the baby check at 6 hours post-natal age. You go to see the baby and mum states that he has not yet had a feed. You advise they stay in hospital until the feeding is established. This is the first child of non-consanguineous parents. On day 4 when you review the baby he has still not had an adequate intake, has lost over 10 per cent in birth weight and is markedly hypotonic. Your consultant asks you to request genetic testing for Prader-Will syndrome. What is the inheritance of Prader-Willi syndrome? A. X-linked B. Imprinting C. Monosomy D. Microdeletion E. Trisomy

B. Imprinting Prader-Willi syndrome is inherited by the phenomenon of genetic imprinting (B). It is due to the fact that for some chromosomes, you need the paternal or maternal chromosome to be present for normal functioning. To inherit Prader-Willi syndrome, there is loss of part of the paternal chromosome 15. X-linked conditions include Duchenne's muscular dystrophy, Fragile X syndrome (A) monosomy is seen in Turner's syndrome with only one X chromosome in girls (C), microdeletions cause DiGeorge's syndrome and Williams' syndrome (D) and trisomy 21, 13 and 18 are seen in Down's, Patau's and Edwards' syndromes, respectively (E).

A 13 month old is referred up to her local district general accident and emergency by a GP who is concerned she has intussusception following an 18-hour history of fever, vomiting and intermittent colicky screaming. A kind radiologist agreed to do an urgent ultrasound which shows an area of invaginated bowel in the right side of the colon. What is the most appropriate management? A. Ask the radiologist to attempt a reduction by rectal air insufflation and if this fails make nil by mouth (NBM) and transfer to a local paediatric surgical unit B. Make NBM and start intravenous fluids while waiting for transfer to a paediatric surgical unit C. Move to theatre for an attempt of rectal air insufflation reduction and if this fails move to surgery in the local hospital as the patient will be too unstable for transfer D. Make NBM, start IV fluids and admit for observation E. Make NBM and start intravenous fluids, and book him onto the emergency theatre list as he is too unstable for transfer to a local paediatric surgical unit

B. Make NBM and start intravenous fluids while waiting for transfer to a paediatric surgical unit Up to 75 per cent of cases may be reduced by air insufflation rectally, but if this fails the child will need to be taken directly to theatre as there is a risk of perforation with the procedure. As it is unlikely that a district general hospital will be able to take such a young child to theatre, all procedures should be carried out in a paediatric surgical centre; therefore (A), (C), (D) and (E) are all incorrect and the only option is to transfer (B).

A 2-year-old child is referred to you by the GP because he has not started walking. His mother says that he can stand but cries to be picked up or sits down shortly. His older sister was walking by 14 months. You note that he is talking well with short two to three word phrases. He is able to build a tower of six blocks. What is your management plan? (development milestones 1) A. Advise mum to work harder at giving him independence and follow up in 4 months. B. Request blood tests including a creatinine kinase C. Refer for physiotherapy D. Reassure and discharge as his development is normal E. Refer to orthopaedics

B. Request blood tests including a creatinine kinase The best answer here is requesting blood tests including a creatinine kinase (CK) (B). His development is not normal (D). He has delayed gross motor skills but seems otherwise normal. Therefore, an important diagnosis to make would be muscular dystrophy and the mother should not be told to try harder (A). This presentation is more severe than is often seen; typically children are a bit clumsy and walk with a waddling gait. If the blood test shows a raised CK then he will need a muscle biopsy to confirm the diagnosis. Until you have established the diagnosis, it would be inappropriate to refer to physiotherapy (C). Developmental hip dysplasia could present with failure to walk and would require orthopaedic referral (E) but a hip ultrasound to confirm the diagnosis would be the appropriate first step in management.

A 2-week-old baby is brought to accident and emergency by his parents because he has been intermittently inconsolable for the past 12 hours. He does not want to breastfeed and has vomited. The parents think his tummy is upset as he keeps drawing up his legs. He was born at term by normal vaginal delivery with no problems. On examination the abdomen is distended and tense. He is crying and there is a firm swelling in the right groin area. You can hear active bowel sounds. What is the most likely diagnosis? A. Appendicitis B. Right inguinal hernia C. Gastroenteritis D. NEC E. Sepsis

B. Right inguinal hernia On examination you can feel a swelling in the groin which is associated with a tense abdomen. In a vomiting baby this is an incarcerated inguinal hernia (B) and needs to be seen urgently by the surgeons to go to theatre. Appendicitis (A) is common in childhood, most commonly seen between the ages of 11 and 20 years old. However, it is very unusual in the infant as the neck of the appendix is wide and unlikely to obstruct at this age. Gastroenteritis (C) can occur in young infants, especially if parents are not using sterilized (boiled and cooled) water to make up formula. It normally presents with a short history of vomiting and diarrhoea in an unwell infant, and there may be a contact history such as another family member with diarrhea or vomiting. NEC (D) is predominately an illness of the premature infant presenting with abdominal distension and bile stained nasogastric aspirates. When it occurs in term infants there is usually a history of birth asphyxia or severe growth restriction. In the absence of localizing signs to the abdomen in a child this young you would treat with IV antibiotics to cover neonatal sepsis (E) but this does not explain the swelling in the groin. It is worth noting that inguinal hernias all require surgical review whereas umbilical hernias will self- resolve and need no surgical intervention.

A 5-year-old girl is brought to accident and emergency with a 24-hour history of vomiting and diarrhoea and now her eyes and skin have gone very yellow. She has been taking oral rehydration salts and is still passing urine. She is normally healthy and there is no family history of jaundice. On examination her heart rate is 130 and respiratory rate is 26. She is alert, warm and well perfused. The chest is clear, heart sounds are normal and the abdomen is soft with a 2cm liver edge. What should the management be? A. Reassure and discharge home, to return if not keeping fluids down B. Take bloods to test for liver function, hepatitis, and urea and electrolytes; inform the Health Protection Agency and discharge home with follow-up to review results C. Take bloods to test for liver function, hepatitis screen and urea and electrolytes and admit for IV fluids D. Take bloods to test for liver function, hepatitis screen and urea and electrolytes and admit for observation with continued oral rehydration salts E. Take bloods to check liver function and urea and electrolytes. If they are normal, discharge home with reassurance but to return if not keeping fluids down

B. Take bloods to test for liver function, hepatitis, and urea and electrolytes; inform the Health Protection Agency and discharge home with follow-up to review results The mostly likely cause of this child's illness is hepatitis A, which should be reported to the Health Protection Agency (B). If at all possible she should not be admitted to reduce the spread to other patients (D). She is drinking and well hydrated. She does not need IV fluids (C). (A) would be reasonable as she is clinically well but the jaundice and palpable liver should be investigated. (E) is wrong as it does not have a hepatitis screen or health protection reporting.

A baby is born by emergency caesarean section due to fetal tachycardia. His delivery was uneventful and you are asked to see him 5 hours later on the post-natal ward. He has just taken his first feed and has been coughing and spluttering since. He had an episode with blue lips transiently and this has now improved; his oxygen saturations are 97 per cent in air and he is apyrexial. On examination, you note other features including vertebral and limb abnormalities, imperforate anus, pansystolic murmur at the lower left sternal edge and renal anomalies noted on antenatal scans. What is the most likely cause for his coughing episode? A. Cleft palate B. Tracheoesophageal fistula C. Choanal atresia D. Incoordinated swallowing reflex E. Pneumonia

B. Tracheoesophageal fistula All the above may result in coughing in this baby. With the constellation of other signs suggestive of VACTERL (vertebral, anal imperforation, cardiac, tracheo-oesophageal fistula, renal and limb anomalies) it makes (B) the correct answer. Babies with a cleft palate (A) also have a similar history but you should expect to see or feel the cleft palate and other dysmorphic features perhaps consistent with DiGeorge's or Down's syndrome. Choanal atresia (C) is associated with CHARGE (coloboma, heart defects, atresia choanae, retardation of growth and development, genitourinary abnormalities and ear anomalies) syndrome. An incoordinated swallowing reflex (D) may exist if there was concern about the neurological or neuromuscular status, but there is no evidence of this from the history. Pneumonia (E) is likely as a consequence of tracheo-oesophageal fistula (B) and may exist independently but the child's afebrile state makes this less likely.

A 13 month old is brought in having had a blue floppy episode at home lasting 1 minute. While you are taking a history from the mother, you notice the baby has gone blue again and seems to be unconscious in her arms. You call for help and place the baby on the examination table. There is no obvious work of breathing. The nurses bring the crash trolley and give you a bag valve mask, which they are connecting to the oxygen. You give two inflation breaths but do not see the chest rise. You reposition the air way and this time the breaths go in. You feel for a pulse and there is none. When asked to do CPR the nurse asks for direction on how many breaths and compressions you both need to do. A. Two inflation breaths per 30 compressions B. Two inflation breaths per 15 compressions C. Continuous inflation breaths about 10-12 per minute and compressions 100-120 per minute D. One inflation breath per five compressions E. Two inflation breaths per five compressions

B. Two inflation breaths per 15 compressions As of November 2010 the paediatric advanced life support guidelines for CPR recommend two inflation breaths per 15 chest compressions (B). This is different from the adult guidelines of two breaths to 30 compressions (A). Answer (C) is correct once the child is intubated but in this scenario, the child is not. Answer (D) is the old neonatal guidelines and is no longer used. Answer (E) is not used at all.

You see a boy in outpatients whose parents are concerned he is not talking yet. You do a developmental assessment and find he is walking well and able to build a tower of three blocks. He will scribble but does not copy your circle. He is able to identify his nose, mouth, eyes and ears as well as point to mummy and daddy. You do not hear him say anything but his parents say he will say a few single words at home such as mummy, daddy, cup and cat. He is a happy, alert child. Parents report him to be starting to feed himself with a spoon and they have just started potty training but he is still in nappies. What is the child's most likely age? (development milestones 2) A 12 months B 15 months C 18 months D 2 years E 2.5 years

C 18 months Developmental assessment should involve assessing the child in all four areas: gross motor; fine motor and vision; speech, language and hearing; and social, emotional and behaviour. This child is scoring 18 months old (C) in all four areas. He is walking steadily (gross motor), scribbling (fine motor), identifies four body parts (speech and language) and reportedly has several single words - he should have at least 10 words and you have not elicited this from him, but he is not worryingly delayed. This would warrant some follow-up. Likely, his older sister is saying everything for him so he does not have to talk yet. He is feeding himself with a spoon and starting to potty train (social and behavioural). At 12 months (A) he would be walking unsteadily and have 2-3 words; he does more than this. At 15 months (B) he would be scribbling and likely walking well but not managing to feed himself with a spoon. At 2 years (D) you would expect him to build a tower of six blocks and be putting words together. He may also be getting close to potty trained, although modern nappies keep children dry so they are not learning to potty train as early. At 2.5 years (E) he should build a tower of eight blocks.

A 1-month-old baby attends accident and emergency with a 2-day history of fever to 38.8°C measured at the GP surgery. He has been vomiting, with no diarrhoea, rash, cough or coryza. A clean catch urine has leukocytes +++ and ketones, no nitrites, blood or protein. An urgent microscopy shows >200 cells/μL white cells. What is the most appropriate course of action? (febrile child) A. Discharge home with 3 days of trimethoprim B. Admit for a course of IV antibiotics to cover a urinary tract infection (UTI) C. Admit for a lumbar puncture, blood cultures and chest x-ray, IV antibiotics D. Organize an urgent DMSA scan E. Discharge home with reassurance and advice to return if fever persists

C. Admit for a lumbar puncture, blood cultures and chest x-ray, IV antibiotics This child has a non-specific presentation. The differential diagnosis of vomiting in children is broad including infection, surgical, reflux and raised intracranial pressure. This child is febrile which increases the suspicion of infection; however, in children under 3 months a febrile presentation requires a thorough search for the cause. Therefore this child needs a septic screen which includes, blood culture, urine cultures, cerebrospinal fluid cultures, chest x-ray and C-reactive protein - (C) and not (E). Although the urine has leukocytes, these are not sensitive on their own or in infants. This child should not be discharged until a focus has been found for the infection (A). If the urine culture grows >105 colony-forming units per millilitre of a single organism, the diagnosis of a UTI can be made and then (B) would be correct. These children will require imaging of the renal tract with ultrasound, DMSA and micturating cystourethrogram depending on age (see NICE guidelines on UTI in children), though a DMSA scan is done at least 6 weeks after the infection (D) as it can be misleading in the acute situation of infection.

A 2-week-old baby was referred to the prolonged jaundice clinic by the community midwife. The pregnancy was unremarkable, and she was born at term with no antenatal abnormalities on ultrasound (US) scans or blood serology. She is now 17 days old and has been jaundiced since day 5 of life and never required phototherapy. She is breastfed and feeds 3-hourly for 20-25 minutes. She is afebrile and not lethargic. Her mother reports that the stools are pale and she has dark coloured urine. The bilirubin is 300 μmol/L, and conjugated bilirubin 100 μmol/L. What is the most important diagnosis to exclude? A. Breast milk jaundice B. ABO incompatibility C. Biliary atresia D. Neonatal hepatitis E. Hypothyroidism

C. Biliary atresia Jaundice is important in neonates, particularly the timing of onset and duration. Jaundice in the first 24 hours may be due to haemolysis either immune (ABO incompatibility or rhesus disease) or non-immune (G6PD, spherocytosis) and is unconjugated; therefore (B) is incorrect. Jaundice between 24 hours and 2 weeks can be physiological as the phenomenon of breast milk jaundice (A) but should not persist. Hypothyroidism is very important to exclude in the neonate due to the potential learning difficulties that can ensue, but it is unlikely to present with acholic stools. Neonatal hepatitis (D) can be secondary to infection (congenital) or metabolic diseases (alpha-1-santitrypsin, cystic fibrosis), but with the history of pale stools and dark urine a diagnosis of biliary atresia (C) must be fully investigated due to the risk of liver cirrhosis.

A 15-month-old girl has come to see you with her father. The family are worried that she has had diarrhoea for more than a month, occasional vomiting and is losing weight. She used to be a happy interactive baby but now seems lethargic and miserable most of the time. She has no significant past medical history, the rest of the family are well and there is no history of travel. Her mother has well- controlled type 1 diabetes. The child's weight at 6 months in the personal child health record ('red book') was on the 50th centile but she is now just below the 9th. What is the most likely diagnosis? A. Crohn's disease B. Ulcerative colitis C. Coeliac disease D. Irritable bowel syndrome E. Giardiasis

C. Coeliac disease The most likely diagnosis is coeliac disease (C), an autoimmune sensitivity to dietary gluten which results in villous atrophy of the small intestine and corresponding malabsorption and malnutrition. Children usually present after weaning and exposure to wheat, with diarrhoea, abdominal distension, failure to thrive and wasting, especially of the buttocks. There is often a family history of autoimmune disease such as thyroid disease or diabetes. Crohn's disease (A) and ulcerative colitis (B) are rare in paediatrics outside of teenagers. Irritable bowel syndrome (D) would be unusual in such a young child but is associated with abdominal pain and bloating with altered bowel habit, including diarrhoea or constipation. It is primarily a diagnosis of exclusion and while weight loss can be seen if the child eats less to avoid symptoms, it is a less prominent presenting feature. Lastly, giardiasis (E) could easily present like this in a young child; however you would expect a travel history to an endemic area or other family members to be unwell having drunk contaminated water. It would be worthwhile sending a stool sample for cysts and parasites.

A 16-year-old boy is brought to the GP by his parents. They are concerned he is the shortest boy in his class. He is otherwise well. His height and weight are on the 9th centile. His father plots on the 75th centile and his mother on the 50th centile for adult height. On examination, his testicular volume is 8mL, he has some fine pubic and axillary hair. The rest of the physical examination is normal. On further questioning you elicit from his father that he was a late bloomer and did not reach his full height until he was at university. What is the most likely cause of the boy's short stature? (growth 2) A. The 9th centile is a normal height and weight so there is nothing wrong with him B. Growth hormone deficiency C. Constitutional delay of growth and puberty D. Underlying chronic illness should be sought E. Anorexia

C. Constitutional delay of growth and puberty This is constitutional delay of growth and puberty (C). It classically runs in families in the male line and presents with delayed puberty and growth spurt. He has started puberty and it seems to be progressing in the normal pattern so he should be reassured that he will get his growth spurt as he finishes puberty. While 9 per cent of the population will have his height and weight (A) and be normal, the mid-parental height suggests he should be taller than he is (the mean parental height plus 7cm for boys or minus 7cm for girls predicts the adult height of the child ± two standard deviations). As he has started puberty and is otherwise well this is unlikely to be growth hormone deficiency (B) or underlying chronic illness (D). In anorexia (E) you would expect the weight to be less than the height centile and to be more extremely low.

A 12-year-old boy who was born with multicystic dysplastic kidneys. He had a renal transplant when he was 7 years old due to chronic kidney disease stage V after having peritoneal dialysis for 1 year. Which of the following would you not expect him to be taking? (renal transplant) A. Septrin B. Tacrolimus C. Diclofenac D. Growth hormone E. Erythropoietin

C. Diclofenac After renal transplantation there are many medications that are required. These children are susceptible to opportunistic infections and septrin (A) is used as prophylaxis to Pneumocystis carinii (Pneumocystis jiroveci), they are also particularly at risk of cytomegalovirus, varicella zoster virus and fungal infection. Immunosuppression is necessary to avoid rejection of the transplanted organ, including tacrolimus (B), ciclosporin and mycophenolate mofetil. Children with chronic renal disease may have impaired growth which can respond to growth hormone (D). They are often anaemic due to defective renal erythropoietin production and supplementation is beneficial (E). Diclofenac (C) should not be used in renal impairment as it and other non-steroidal anti-inflammatory drugs are nephrotoxic.

You are asked to see a 3-day-old baby on the post-natal ward. The baby was born at term and is the first child of consanguineous parents. The baby is drowsy and vomiting, with no fever, rash or diarrhoea. On examination, the baby is noted to have ambiguous genitalia. You do some blood tests: white cell count 5 × 109/L, C-reactive protein 2 mg/L, Na+ 125 mmol/L, K+ 8 mmol/L, glucose 1.7 mmol/L. 17-OH level progesterone is low. You make a diagnosis of congenital adrenal hyperplasia. What is the best initial management plan? A. IV hydrocortisone B. IV dextrose C. IV dextrose and IV hydrocortisone D. IV 0.9 per cent saline E. IV 3 per cent saline and IV hydrocortisone

C. IV dextrose and IV hydrocortisone Congenital adrenal hyperplasia is due to a deficiency of the enzymes that metabolize sex hormones (testosterones) to cortisol. Hence the accumulation of sex hormones can result in masculinized female genitalia or abnormal male genitalia. The lack of cortisol presents as a salt-losing crisis, as in this child, with low glucose and vomiting. Replacement of glucose and steroids is required in the first instance (C); they are both of high priority, so not (A) or (B). This will help to correct the electrolyte imbalances. Sodium disturbance should generally not be corrected too quickly; therefore (E) is incorrect. Normal saline (0.9 per cent) alone will not help, although may be required if there is hypotension.

You see an 18-year-old boy who is the first child of his African parents and was born in Kenya before moving to the UK 1 year ago. He has white skin and pink irises. He was diagnosed with oculocutaneous albinism at birth. He has difficulty with his sight but has recently developed a skin lesion on his face. His mother has brought him to his GP as it has recently started to increase in size. On examination you note is an elevated, 3 cm diameter lump on the left of his nose. It has irregular edges, is firm and immobile and pigmented in areas. What is the likely diagnosis? A. Benign naevus B. Scar from a healing wound C. Malignant tumour D. Abscess E. Wart

C. Malignant tumour This boy has oculocutaneous albinism, a metabolic condition affecting the production of skin pigment melanin. He is therefore at higher risk of skin malignancy (C) due to the lack of melanin. There is no history of trauma (B) or infection or fever (D) so both are less likely. Benign naevus or mole (A) would be static in size, smooth and regular edged in comparison to a malignant tumour. Warts caused by human papilloma virus (E) may be in the differential diagnosis but are rarely located on the face. With the medical background of albinism, (C) is more likely.

You see an 8-year-old boy in accident and emergency who fell off his bike 3 days ago and scraped his left calf. The cuts are now angry, red and painful. You note he is a big boy and plot his growth: his weight is on the 99th centile and height is on the 75th centile. You note mild gynaecomastia and stretch marks on his abdomen which are normal skin colour. His past medical history is unremarkable except for mild asthma. What is the most likely cause of his large size? (growth 1) A. Cushing's syndrome secondary to a pituitary adenoma B. Cushing's syndrome secondary to becotide inhaler use C. Obesity D. His size is within the normal range and is a variant of normal E. Liver failure

C. Obesity This child is obese (C). He should be advised to continue exercise, such as bike riding, and make healthy choices for his diet, such as eating fruit rather than potato chips for snacks. Children should not be put on diets under normal circumstances as a normal healthy diet with regular exercise will improve his weight as he grows. In the context of a wound infection and abdominal striae, many worry that obesity may be Cushing's syndrome but this would typically present with growth failure and his height is above average (A), which is typical of obesity (C). Cushing syndrome is not caused by steroid inhalers (B) but may be seen in children with severe asthma on oral steroids. While both his height and weight plot on the lines of the centile chart, only 1 per cent of the population of 8 year olds have his weight normally and his height does not match his weight centile, suggesting that he is overweight (D). The mild gynaecomastia is unlikely to be made of breast tissue and is more likely to be fat pads as he is active riding bikes. It is unlikely that he is unwell enough to be in liver failure despite possibly having gynaecomastia (E).

An older mother books in to see you after attending the health visitor for a weight check at 2 months for her first child. She and her husband have had a hard time coming to terms with their daughter's diagnosis of Down's syndrome. She is relieved that the appointment with the cardiologists went well and the heart is normal. However they have a lot of trouble getting her to take the whole bottle, she was slow to regain her birth weight and looking at the plotted weight yesterday she is not growing along her birth centile and the mother is worried she is not doing a good enough job. She is not vomiting except for small possets after feeds, is passing urine and opening her bowels. The red book growth chart shows the weight to be falling off centiles. What is the most appropriate management? (Down's syndrome) A. Contact the cardiologists in light of the poor feeding and slow weight gain for a second opinion as baby's with Down's syndrome are at high risk of heart problems and they may have missed it B. Refer to the dietician for nutritional support C. Replace the growth chart in their red book with a Down's syndrome growth chart, reassure mum by re-plotting her growth and explain she is normal but arrange to review again D. Tell the mother to try a different milk and come back in 2 weeks E. Advise the mother to change to a faster flow teat for their bottles so that she takes her feed faster

C. Replace the growth chart in their red book with a Down's syndrome growth chart, reassure mum by re-plotting her growth and explain she is normal but arrange to review again Children with Down's syndrome do have different growth patterns to other children and should be given special growth charts in their red books, so the correct answer is (C). If the cardiologist has carried out an echocardiogram (A) and discharged the patient, unless there are clinical signs and symptoms to warrant re-referral, this would be a waste of resources and needless worry for the parents. A dietician referral (B) will be needed if she is failing to thrive and continues to have difficulty feeding, but this should be done with investigations for a cause. Parents do shop around to find a milk their child prefers but in the absence of an intolerance the over the counter milks (D) are all equally good and this is unlikely to help. A faster teat (E) may make things worse as Down's syndrome is associated with low tone and swallowing difficulties, so she may choke if the flow is too fast

A preterm baby is born at 25 + 6 weeks gestation. He is delivered by caesarean section due to maternal pre-eclampsia. He is intubated at birth and given surfactant via the endotracheal tube. He is ventilated and commenced on IV dextrose. After 4 hours of age he has increased work of breathing, with intercostal and subcostal recession and a respiratory rate of 60/min. A chest x-ray shows a ground glass pattern in both lung fields. He has no audible murmur. He is afebrile. You diagnose respiratory distress syndrome. What is the aetiological factor responsible for respiratory distress syndrome? A. Pneumonitis B. Lung hypoplasia C. Surfactant deficiency D. Immature lung parenchyma E. Infection with group B Streptococcus

C. Surfactant deficiency Respiratory distress syndrome (C) is secondary to surfactant deficiency due to immaturity of type 2 pneumocytes in the alveoli of the developing lung. Preterm babies are at risk by virtue of their gestational age. They have respiratory distress, and require artificial surfactant and ventilatory support as their lungs are prone to atelectasis. The lung parenchyma is structurally immature but functional (D). Congenital pneumonia (E) may present with increased work of breathing and focal signs on chest x-ray. Lung hypoplasia (B) occurs due to renal problems or diaphragmatic hernia, where the underlying lung is unable to develop and therefore is impaired functionally. (A) occurs as a part of meconium aspiration syndrome, and is due to the irritation to the lung caused by the chemicals in meconium.

A 13-year-boy is brought to see you as he has recently been complaining of abdominal pain and is increasingly tired. On examination you note some early clubbing and erythematous palms. His conjunctivae look pale. He has one or two spider naevi on his chest. His abdomen is soft with mild tenderness in the epigastrium and right upper quadrant. The liver is palpable at 1 cm and you feel the splenic tip. He has normal bowel sounds and no bruits. On slit lamp examination of his eyes, an amber ring is noted around the cornea. What is the most likely diagnosis? A. Abdominal tuberculosis B. Cystic fibrosis C. Wilson's disease D. Acute hepatitis A E. Glandular fever

C. Wilson's disease He is presenting with signs of chronic liver failure, clubbing and palmar erythema. While he does have spider naevi you need at least five to be significant. Slit lamp examination of his eyes revealed Kayser-Fleischer rings which are a sign of Wilson's disease (C). This is an autosomal recessive defect in copper metabolism that results in copper deposition in the tissues and leads to liver failure with cirrhosis, neurological sequelae, renal involvement and cardiac complications. In addition patients develop haemolytic anaemia due to copper deposits in the red cell membrane. Thus, as he has a palpable liver and spleen, he is anaemic and complaining of upper abdominal pain with stigmata of liver failure, the most likely diagnosis is Wilson's disease (C). Abdominal tuberculosis (A) is part of the differential diagnosis, but is unlikely to cause such significant signs of hepatic dysfunction, and would not explain the eye findings. Cystic fibrosis (B) may cause clubbing, but you have been given no pulmonary information, making this diagnosis unlikely. Acute hepatitis A (D) would not produce signs of chronic liver failure and normally presents with vomiting and possible jaundice. There is often a palpable liver but splenic enlargement would be unusual. Similarly, glandular fever (usually caused by infection with the Epstein-Barr virus (EBV)), would not produce signs of chronic liver failure on examination, but the child may give a history of lethargy and have a palpable liver and spleen. Again, the eye findings would not be explained by EBV infection.

A 10-month-old boy is referred to you by a GP. He is suffering from intermittent colicky pain. There are episodes between these attacks of abdominal pain where the child is completely quiet without distress. His nappy shows blood resembling redcurrant jelly mixed with stools. A appendicitis B coeliac disease C ulcerative colitis D intussusception E Peutz-Jeghers syndrome F Crohn's disease G anorexia nervosa H constipation I peptic ulcer J Wilms' tumour

D intussusception The key to this vignette is the history of intermittent abdominal pain and the classic description of redcurrant jelly stools. In most children the intussusception is ileocaecal, with a male preponderance and a peak age of occurrence between 5 and 10 months. Early diagnosis is vital to avoid open surgery.

An 11-year-old girl presents to the out of hours GP while on holiday in England with abdominal pain. She tells you she has polycystic kidney disease which was diagnosed early in life. She has bilateral palpable kidneys and hepatosplenomegaly, with visible distended veins on the abdomen and ascites. Abdominal ultrasound shows liver fibrosis. What is the inheritance of this condition? (polycystic kidney disease) A. Autosomal dominant B. X-linked C. Sporadic mutation D. Autosomal recessive E. Microdeletion

D. Autosomal recessive This girl has polycystic kidney disease, of which there are two types. The autosomal recessive type (D) presents in childhood with bilateral renal masses, respiratory distress due to pulmonary hypoplasia and congenital hepatic fibrosis with pulmonary hypertension. It is due to tubular dilatation of the distal collecting system. Renal function is impaired and progressively deteriorates, requiring renal replacement therapy (dialysis or transplant). The autosomal dominant (A) type may present in older children or adults. The cysts are grossly dilated nephrons which compress normal renal tissue. It affects renal, liver and cerebral vasculature.

A 10-year-old child is brought in by ambulance with seizure activity. His mother reports it starting 30 minutes ago in his right arm and quickly became generalized tonic clonic jerking. She gave him his buccal midazolam after the first 5 minutes and called an ambulance when he did not respond after another 5 minutes. The ambulance crew gave him rectal diazepam on arrival at 15 minutes into the seizure. He is receiving high flow oxygen via a face mask and continues to convulse. The mother tells you that he was weaned from his long-term seizure medication, phenytoin, 2 weeks ago and that he has had a cold for the past 2 days. What is the next step in management? A. Gain intravenous or intraosseous access and administer lorazepam B. Gain intravenous or intraosseous access and administer ceftriaxone C. Repeat the rectal diazepam D. Gain intravenous or intraosseous access and start a phenytoin infusion E. Gain intravenous or intraosseous access and start a phenobarbital infusion

D. Gain intravenous or intraosseous access and start a phenytoin infusion The child has now had two doses of barbiturate and management should proceed to the next step in management of status epilepticus (seizure lasting greater than 30 minutes): IV phenytoin (D). Due to the risk of respiratory depression further doses of barbituates ((A) or (C)) should be avoided. It should be noted that many text books still recommend the use of per rectum peraldehyde if two doses of barbituates fail; however this is now not routinely used in management of status epilepticus due to significant rates of drug errors such as IV rather than rectal administration. If his epilepsy was currently managed by phenytoin he should be given intravenous phenobarbital (E) but he was recently taken off of it. Intravenous ceftriaxone (B) may be indicated in a situation of status epilepticus associated with fever as encephalitis may present with seizures, but it is not the next step in management.

A 3-day-old baby is seen by the midwife for a routine post-natal review. She notices that he is very floppy and his mother has raised concerns about his poor feeding. He has a protruding tongue, epicanthic folds, low set ears and sandal gap toes. She explains to the parents she thinks he may have Down's syndrome and refers him to the paediatrician. What is the diagnostic test for Down's syndrome? A. Serum alpha fetoprotein, beta human chorionic gonadotrophin, oestriol, inhibin B. Gene mutation analysis C. Clinical diagnosis D. Karyotype E. FISH

D. Karyotype Down's syndrome is due to trisomy 21 (three copies of chromosome 21) and therefore karyotyping will identify this extra chromosome (D). Women can be screened in pregnancy for the condition using various serum markers in combination with US scan (A). There is no gene mutation to identify (B); this would be more suitable in the diagnosis of cystic fibrosis. Although identification of a syndrome does involve elucidating clinical features, this needs to be confirmed with a diagnostic test if there is one available and karyoptying is the most definitive investigation in confirming a suspected diagnosis of Down's syndrome (C). Fluorescence in situ hybridization (FISH) (E) is used for the identification of microdeletion syndromes such as DiGeorge's or cri du chat syndrome.

A 3-year-old girl presents to accident and emergency with a 6-day history of fever and she is over 38°C when measured by her mother with a tympanic thermometer. She has become very miserable for the last few days. She has developed a rash on her trunk, which is blanching, erythematous and confluent. On examination, you also note bilateral non-purulent conjunctivitis, cervical lymphadenopathy, and a red tongue with lip cracking. Her extremities are also erythematous but not peeling. WCC 14×109/L, C-reactive protein 200 mg/L, and erythrocyte sedimentation rate 60 mm/hour. Blood culture is pending. What is the diagnosis? A. Staphylococcal scalded skin B. Toxic shock syndrome C. Scarlet fever D. Kawasaki's disease E. Measles

D. Kawasaki's disease This girl has all the features of Kawasaki's disease (D). This is an inflammatory disease of unknown aetiology. It shares many features with all the other diagnoses listed, but may also have cardiac involvement in the form of coronary artery aneurysms. (A) causes a peeling of skin with a fever. (B) presents with a red macular rash, fever and usually an additional diarrhoeal illness. (C) presents with a sandpaper rash and erythematous mucous membranes. (E) is now an uncommon disease due to vaccination, though with reducing uptake of the MMR vaccine, cases are increasing, causing a febrile child with an erythematous, macular rash.

An 8 year old known asthmatic is brought into accident and emergency by ambulance as a 'blue call'. He has been unwell with an upper respiratory tract infection for the past 2 days. For the past 24 hours his parents have given him 10 puffs of salbutamol every 4 hours, his last dose being 90 minutes ago. The ambulance staff have given him a nebulizer but he remains agitated with a heart rate of 155, respiratory rate of 44 and sub/intercostal recessions and on auscultation there is little air movement heard bilaterally. Saturations in air are 85 per cent. He is started on 'back to back' nebulizers with high flow oxygen. How severe is his asthma exacerbation and what other bedside test would support this? A. Moderate, venous blood pH 4.4, gas PCO2 = 3.1 kPa B. Severe, peak flow <33 per cent expected C. Severe, venous blood pH 4.4, gas PCO2 = 3.1 kPa D. Life-threatening, peak flow <33 per cent expected E. Life-threatening, venous blood pH 4.4, gas PCO2 = 3.1 kPa

D. Life-threatening, peak flow <33 per cent expected This child has signs of a life-threatening asthma exacerbation: agitation, cyanosis, silent chest. Other signs would be fatigue, drowsiness or poor respiratory effort and a peak flow of less than 33 per cent expected (D). Therefore, (B) is incorrect as severe would be a peak flow between 33 and 55 per cent expected. In addition, signs of severe exacerbation include being too breathless to talk or feed, tachycardia and tachypnoea. A blood gas is not part of the guidelines for assessing asthma severity but in this child a worrying gas reading would show a respiratory acidosis, i.e. a rising CO2 suggests that he is unable to maintain the respiratory effort and is therefore failing to clear CO2 leading to a rise in the pH. For this reason (A), (C) and (E) are all incorrect as they are signs of a moderate exacerbation showing hypocapnia associated with hyperventilation, which is typical of asthma.

A 10-day-old baby boy was brought to accident and emergency with a distended abdomen. On questioning, he was born at term with no antenatal concerns. Until 2 days ago he had been feeding well and not vomiting, he had been wetting nappies, but mother has not witnessed a good urinary stream. On examining the child, you find a mass, dull to percussion, arising out of the pelvis, and he has had no wet nappies for the last day. You suspect he may have posterior urethral valves. Which one test will help to diagnose this underlying condition? (bladder outflow obstruction) A. DMSA scan B. Renal biopsy C. Computed tomography (CT) abdomen D. Micturating cystourethrogram E. Renal ultrasound

D. Micturating cystourethrogram This presentation of an enlarged, palpable bladder and anuria for 24 hours is suggestive of bladder outflow obstruction. This, in conjunction with a poor urinary stream, would make the diagnosis of posterior urethral valves likely. This is diagnosed using micturating cystourethrogram (D). DMSA (A) and renal biopsy (B) would provide information about the kidney. CT abdomen (C) would image the enlarged, fluid-filled bladder but possibly not the urethral valves. Renal ultrasound (E) will show the enlarged bladder and possibly show evidence of hydroureter and hydronephrosis with backflow from the bladder.

A 5-year-old boy presents to his GP with a 3-day history of puffy eyes. He has been unwell with a coryzal illness for the last week. His mother states he has had no new medications and no hayfever, allergies or asthma. On further examination he has generalized oedema and scrotal oedema. He is tachycardiac and has cool peripheries, no skin rashes or erythema. What is the most likely diagnosis? (Generalised oedema) A. Periorbital cellulitis B. Allergic reaction C. Nephrotic syndrome D. Nephrotic syndrome with hypovolaemia E. C1 esterase deficiency

D. Nephrotic syndrome with hypovolaemia Nephrotic syndrome (triad of proteinuria, hypoalbuminaemia and generalized oedema) presents with puffy eyes, which could also be a feature of any of the above diagnoses. However, the examination findings of generalized oedema make periorbital cellulitis (A) (in the absence of a fever) and allergic reaction (in the absence of skin rash, wheals, flares) (B) unlikely. The diagnosis of nephrotic syndrome (C) precipitated by an intercurrent infection in this case is correct, but with tachycardia and cool peripheries this suggests this child additionally has hypovolaemia, due to increased interstitial fluid collection due to reduced oncotic pressure (D). Rarely, C1 esterase deficiency (E) can present with generalized oedema following an intercurrent illness.

A 12-year-old boy presents to his GP with left-sided unilateral breast development stage III. He is very upset as he is being bullied at school. His mother is worried as her friend's sister has just been diagnosed with breast cancer and wants to know if he could have breast cancer? What is the management? (puberty) A. Refer for a breast ultrasound B. Test sex hormone levels C. Test alpha fetoprotein D. Reassure and explain this is a normal part of puberty; it will resolve but the other breast may enlarge transiently as well E. Do a fine needle aspirate on his left breast

D. Reassure and explain this is a normal part of puberty; it will resolve but the other breast may enlarge transiently as well This is a common presentation for adolescent males and the correct answer is to reassure (D). Normal puberty for boys starts between 9 and 13 years old with increasing testicular volume above 4mL. This is followed by penis enlargement, pubic hair growth and lastly the growth spurt. It is not uncommon for boys to develop transient gynaecomastia during puberty. He does not need investigation (A, B, C or E) or intervention, unless the problem does not resolve.

A mother comes to see you with her 2-year-old daughter, Stacey, out of frustration that her daughter is so ill behaved. She does not know how to make her listen and is worried that she is going to get hurt. Yesterday she ran ahead and did not stop when her mother called to her. She ran into the street and was hit by a cyclist, but fortunately he was OK and Stacey had only had a few cuts and scrapes and seems alright! On questioning you hear other stories of a naughty child. She is active and eats well, feeding herself a lot now, but her mother does say she gets frequent coughs and colds. Her mother says that Stacey only says about 5-10 words and only she can understand what Stacey says. What is the best next management? (children development) A. Ask the health visitor to visit mum for parenting advice and support B. Order blood tests for full blood count to check for leukaemia as she has recurrent coughs and colds C. Give Stacey a tetanus shot to cover her after her fall the day before D. Refer for a hearing test E. Tell Stacey that she needs to listen to her mother and not have any more accidents

D. Refer for a hearing test Stacey seems to have some speech delay. She is feeding herself and running which is appropriate for a 2 year old but she is only talking at about a year-and-a-half's age level and she is difficult to understand. This, along with a story of poor attentiveness in a child, raises concerns of hearing problems. The most common cause would be glue ear associated with recurrent coughs and colds and she should have a hearing test (D). She does not need any blood tests (B) as all children at this age have recurrent coughs and colds, the average being eight illnesses a year. Only recurrent severe infections warrant investigation for immune deficiency. The routine childhood vaccinations cover her for tetanus (C); unless her records say she has not had them she does not need a booster, and she will have the pre-school booster and one when she is a teenager. Answer (E) will not change Stacey's behaviour; young children respond to positive and negative reinforcement techniques to modify behaviour.

A 6-year-old girl presents to hospital with a large right-sided abdominal mass. It does not cross the midline. On further questioning she has had macroscopic haematuria and weight loss of 4kg over the last 4 months. She has reduced appetite and lethargy. Her blood pressure is 125/73 mmHg, heart rate 120bpm. Which of the following is not a complication of this malignancy? (Haematuria) A. Malnutrition B. Hypertension C. Renal impairment D. Urinary catecholamines E. Metastatic spread

D. Urinary catecholamines Wilms' tumour can result in hypertension (B) and renal impairment (C), dependent on the functioning of the contralateral kidney. General complications of malignant disease include metastases (E) and malnutrition (A) due to poor appetite, vomiting and increased metabolic demands. Sympathetic nervous system stimulation does not occur with Wilms' tumours but with neuroblastoma, an important differential diagnosis, due to catecholamine production from the tumour which originates from the adrenal medulla. Catecholamines can be detected in urine samples.

A mother brings her 7-year-old daughter to see you. There is a bright red papular rash around her wrist. When you examine it the child tells you that it started when she put on her new watch. A atopic eczema B meningococcal septicaemia C alopecia D systemic lupus erythematosus E Henoch-Schönleinpurpura F contact dermatitis G erythema multiforme H pompholyx I psoriasis J incontinentia pigmenti

F contact dermatitis Contact dermatitis. The watch strap may contain nickel which often precipitates a contact dermatitis. Patients may present with such a condition near the umbilicus (due to wearing a belt containing nickel). Sufferers of contact dermatitis need to be aware of the offending agent to avoid it. Sometimes this can be accomplished by performing patch testing.

A father brings his 7-year-old son to clinic complaining that he has dark red- purple spots on his legs and buttocks. The child complains of sore knees and a mild headache. His father reports that he recently had a sore throat. Urine dipstick is positive for haematuria and proteinuria. A atopice czema B meningococcal septicaemia C alopecia D systemic lupus erythematosus E Henoch-Schönlein purpura F contact dermatitis G erythema multiforme H pompholyx I psoriasis J incontinentia pigmenti

E Henoch-Schönlein purpura Henoch-Schönlein purpura. This is an IgA-mediated autoimmune vasculitis of childhood. This is often preceded by respiratory illness such as group A streptococci or Epstein-Barr virus. Skin purpura, joint pain, abdominal pain and other symptoms in this vignette are typical of the kind of presentation that can occur. Intussusception is sometimes a complication of HSP. Corticosteroid mediation is sometimes provided but normally the condition self-resolves.

A diabetic mother gives birth to her first child at 31 weeks gestation. A few hours later the baby becomes tachypnoeic, and you notice nasal flaring, intercostal recession, expiratory grunt and cyanosis on examination. The lecithin/ sphyngomyelin ratio was noted to be < 2:1 A foreign body B asthma C pulmonary haemorrhage D pneumothorax E respiratory distress syndrome F congenital heart disease G meconium aspiration syndrome H pneumonia I croup J respiratory syncitial virus

E respiratory distress syndrome Respiratory distress syndrome starts within 4 hours of birth and is due to lack of surfactant production and structural immaturity of the lungs. Risk factors include hypothermia, asphyxia, acidosis and lecithin/ sphyngomyelin ratio < 2:1. Often this syndrome can be prevented if mothers are given glucocorticoids prior to delivery which speeds up the surfactant production.

A newborn baby is born to non-consanguineous parents. She is noted to have puffy feet on her 1st day check. She weighs 2.0kg with widely spaced nipples and absent femoral pulses. You have asked your registrar to review her as you think she may have Turner's syndrome. She agrees and asks you to send blood tests for karyotyping. Which is the chromosomal diagnosis of Turner's syndrome? A. 47XXY B. 45YO C. 46XY D. 46XX E. 45XO

E. 45XO Turner's syndrome is due to the absence of two sex chromosomes (pair number 23). Therefore the child is a girl (due to the absence of the Y chromosome) and has only 45 chromosomes. The physical characteristics may be noted at birth, particularly lymphoedema and cardiac anomalies, or noted later in life, such as short stature, delayed puberty, thyroid disorders, coarcation of aorta, aortic stenosis, horseshoe kidney and coeliac disease. (C) and (D) are normal gentoypes for males and females respectively. (A) is the karyotype of Klinefelter's syndrome which presents in boys with tall stature, delayed puberty and gynaecomastia.

Which of the following is not a cause of PR bleeding? A. Constipation with an anal fissure B. Intussusception C. Meckel's diverticulum D. Bacterial gastroenteritis E. Abdominal migraine

E. Abdominal migraine Often the first presentation of migraine in children is abdominal pain, in the context of a family history of migraine. This diagnosis would not be supported if the child has per rectum bleeding (E). The most common cause of PR bleeding in children is constipation with an anal fissure and this would produce bright red blood on the toilet tissue or streaked on the surface of the stool (A). Intussusception has a late and uncommon sign of 'redcurrant jelly' stool (B) which is due to blood and mucus from the distal end of the invaginated segment of bowel becoming necrotic. At this stage there is a risk of perforation, especially if insufflation is attempted. The diagnosis should ideally be made long before these signs evolve. Two per cent of the population have a Meckel's diverticulum, an ileal remnant of the vitellointestinal duct which contains ectopic gastric mucosa and can lead to ulceration, perforation and a presentation with severe rectal haemorrhage of dark red blood (C). Bacterial gastroenteritis with Shigella or Salmonella may produce blood and/or pus mixed into the stool (D).

A 10-year-old boy is brought to the GP with tall stature. He is taller than his peers at school. His arm span is greater than his height, he has long, thin fingers, scoliosis and pectus excavatum. He is also concerned that he gets short of breath at school during PE lessons. You refer him for an echocardiogram and chest x-ray. You make a clinical diagnosis of Marfan's syndrome. What is the inheritance of Marfan's syndrome? A. X-linked recessive B. Autosomal recessive C. Sporadic D. X-linked dominant E. Autosomal dominant

E. Autosomal dominant Marfan's syndrome is an autosomal dominant (E) condition affecting the fibrillin gene. It is a connective tissue disorder that affects the musculoskeletal, ocular and cardiac systems. X-linked conditions affecting the musculoskeletal system include Duchenne's and Becker's muscular dystrophy, which can present with muscle weakness or delayed motor milestones (A). Autosomal recessive conditions affecting the musculoskeletal system include homocysteinuria, which has similar skeletal characteristics but also thromboembolic tendency (B). Klinefelter's syndrome is sporadic and can present with tall stature (C). Hypophosphataemic rickets is an X-linked dominant condition, which can present with genu varum and short stature (D).

James is an 8-year-old boy who recently attended accident and emergency with a swollen left ankle. He had an x-ray and was discharged home and told there was no fracture. He has now developed a dark purple rash on his legs, which does not disappear with a glass pressed on. He was brought back to the department today vomiting, with abdominal pain. His observations and urine dipstick are all within normal limits. What is the most likely diagnosis? (purpura) A. Diabetic ketoacidosis B. Viral gastroenteritis C. Meningococcal sepsis D. Idiopathic thrombocytopenic purpura E. Henoch-Schönlein purpura

E. Henoch-Schönlein purpura Henoch-Schönlein purpura (E) is a diagnosis based on a set of symptoms including arthralgia, rash (urticarial to purpuric), abdominal pain and renal involvement (hypertension, nephrotic syndrome). It is a common paediatric condition affecting 3-10 year olds and usually follows a viral illness. It is a vasculitis process which can cause severe gastrointestinal and renal complications but is usually benign and self-limiting. Any child with abdominal pain and vomiting should have their glucose checked as they may be presenting for the first time with diabetic ketoacidosis (A). Viral infections can precipitate a reactive arthritis, though this follows the primary infection (B). Meningococcal sepsis (C) is always a concern in children with a purpuric rash and this must be considered and excluded and, in this case, is less likely with the normal heart rate, blood pressure and respiratory rate (observations within normal limits). Idiopathic thrombocytopenic purpura (D) may present with the rash, but the other symptoms are unusual with this condition and the platelet count would be low, whereas in Henoch-Schönlein purpura the platelet count is normal.

A 3-year-old boy presents with a right swollen eyelid. He has had a cold for the last week but his eyelid started swelling yesterday. He has had no injury or broken skin around the eye. On examination, his right eye is swollen and red, there is no discharge, he is now unable to open his right eye and he has proptosis. You are concerned about the complications of this infection. Within the last hour he has become more drowsy and started to vomit. His observations are all normal. What is the concerning complication in this case? A. Visual loss B. Abscess C. Septicaemia D. Orbital cellulitis E. Meningitis

E. Meningitis All of these can be complications of periorbital cellulitis. This child, with reduced conscious level and vomiting, would raise concern about meningitis (E) as the infection spreads to the cerebrospinal fluid around the optic nerve. All of these complications are worrying and require further investigation. Septicaemia (C) may also cause these symptoms if the child is in shock with poor perfusion of the brain. (A), (B) and (D) need to be excluded with CT head and ophthalmology review.

A preterm baby is now 25 + 7 weeks corrected gestation. He is on the neonatal unit being cared for while his mother recovers on ITU after he was born secondary to an eclamptic seizure. He has been receiving formula milk as the parents have not consented to donor breast milk. He has been having bilious aspirates from his nasogastric tube and today his abdomen in very distended and tense. He has had one episode of bloody stools. You are going to treat him for nectrotizing enterocolitis (NEC). What is the best initial management plan? A. Conservative management, observe and reassess B. Nil by mouth (NBM), IV antibiotics and emergency exploratory laparotomy C. IV fluids, emergency laparotomy and bowel resection D. IV fluids and IV antibiotics E. NBM, IV fluids, abdominal X-ray and surgical review

E. NBM, IV fluids, abdominal X-ray and surgical review NEC is a complicated disease occurring in preterm and also term babies. It has many risk factors, in this case formula feeding, prematurity and potential ischaemia at birth. The suspicion for NEC must remain high and therefore in this child where there are clear signs of bowel pathology, must be treated as such. He should therefore be managed actively, not conservatively (A). As with any suspected surgical problem, patients should be made NBM and given IV fluids, hence (D) is incorrect. He needs to be reviewed by a surgeon and imaging may assist this assessment (E). He cannot be operated on unless he is first seen by a surgeon and initial investigations are carried out; therefore (B) and (C) are incorrect. However he may require laparotomy and bowel resection.

An 18-month-old child is brought into accident and emergency with a 2-day history of vomiting, abdominal pain and fever. Which of the following is an unlikely cause of this clinical picture? A. Lower lobe pneumonia with pain referred to the abdomen B. Mesenteric adenitis C. Diabetic ketoacidosis D. Pyelonephritis E. NEC

E. NEC The answer is NEC (E) which classically affects preterm babies in the neonatal period and would not be a cause of abdominal pathology in an 18-month-old child. Pneumonia in the lower lobes (A) may present as pain in the upper abdomen and fever. Any upper or lower respiratory tract infection in young children can be associated with vomiting, mostly because of inflamed upper airways triggering the gag reflex and increased work of breathing putting pressure on the stomach just below the diaphragm. Mesenteric adenitis (B) is the most likely answer as this is classically how mesenteric adenitis presents, although it is a diagnosis of exclusion, having ruled out other pathology. There is often a preceding viral illness (typically either upper respiratory or gastroenteritis). Enlarged mesenteric glands can be seen on abdominal ultrasound examination. Mesenteric adenitis may be mistaken for appendicitis. While it is unusual for such a young child to develop diabetes (C), even babies can. Infection is often a trigger for new onset or an episode of ketoacidosis in a known diabetic, and this may present with vomiting. Pyelonephritis (D) should always be ruled out in a child with fever and vomiting unless a clear alternative source of infection is identified.

A pregnant woman seeks advice from you regarding her condition and its impact on the pregnancy and risk to the baby. She has phenylketonuria (PKU) and has been on a phenylalanine-free diet for life. She was told that it was very important during her pregnancy to be compliant with this diet. She would like to know how the baby will be tested for the condition as she is aware that is an inherited condition. What is the initial investigation you will advise? A. Serum tyrosine levels B. Genetic screening C. Serum phenylalanine levels D. Urine phenylketones E. Newborn blood spot screening

E. Newborn blood spot screening PKU is an autosomal recessive metabolic condition resulting in a defect in enzyme phenylalanine hydroxylase, which converts phenylalanine to tyrosine. Due to the accumulation of phenylalanine and conversion to phenylketones, unrecognized and untreated PKU can result in seizures and musty smelling urine and eventually microcephaly and learning difficulties. PKU is screened on the newborn blood spot screening test (E). It is a rare but treatable condition with a good prognosis if management is started from birth (phenylalanine-free diet for life). (A), (C) and (D) can be measured but not usually as the diagnostic test, but they can be useful for monitoring of treatment. (B) is not performed routinely for PKU.

A 10-year-old boy is brought to the paediatric outpatient department for a review of his height. He was found to be on the 0.4th centile and his mid-parental height is the 98th centile. He also has widely spaced nipples, wide carrying angle, hypogonadism, pulmonary stenosis and developmental delay. What is the most likely diagnosis? A. Angelman's syndrome B. Williams's syndrome C. Turner's syndrome D. Prader-Willi syndrome E. Noonan's syndrome

E. Noonan's syndrome Noonan's syndrome has a similar phenotype to Turner's syndrome but can present in both girls and boys (unlike Turner's syndrome which only occurs in girls (C)). These features are not typical of the other syndromes: (A) developmental delay and happy demeanour, (B) typical facial features, aortic stenosis, developmental delay, (D) poor feeding and weight gain in the neonatal period, followed by overeating in later life and obesity.

You are called to see a baby who has just been born at 39 weeks' gestation, as the midwife thinks he is small and should be admitted to the neonatal unit for his care. You review the baby. His weight is 1.8kg, below the 0.4th centile and his head circumference is 35 cm - 50th centile. He has no dysmorphic features. Which is the most likely cause of this IUGR? A. Chromosomal anomaly B. Maternal smoking C. Congenital infection D. Maternal alcohol use E. Placental insufficiency

E. Placental insufficiency Placental insufficiency is the most likely cause of IUGR. This child has asymmetrical IUGR, suggesting that the insult occurred late in pregnancy with head growth sparing. Causes include maternal diabetes or pre- eclampsia, both resulting in placental insufficiency due to their effects on placental microvasculature. Chromosomal anomaly (A), maternal smoking (B), congenital infection (C) and maternal alcohol use (D) would more likely cause low birth weight and head circumference changes, i.e. symmetrical IUGR is present from the first trimester.

A 5-year-old girl was brought to hospital at midnight by her mother with 5 per cent partial thickness burns to her chest and abdomen. Her mother states that she pulled on the kettle at 2 pm and the boiling water scalded her. On examination she is tachycardic, and drowsy with cool peripheries. Her initial blood tests: sodium 150 mmol/L, potassium 7.8 mmol/L, urea 10.2 mmol/L, creatinine 104 μmol/L, haemoglobin 14 g/dL. What is the most likely aetiological factor to account for these results? (renal impairment) A. Post-renal cause of acute kidney injury B. Poisoning C. Renal cause of acute kidney injury D. Dehydration E. Pre-renal cause of acute kidney injury

E. Pre-renal cause of acute kidney injury This girl has presented with a scald injury. The accident happened 10 hours before presentation which may arouse suspicion of non-accidental injury, but at present the more concerning feature is the acute kidney injury in a child with clinical shock. Burns damage the protective skin barrier, which allows excessive fluid and electrolyte loss. She may have been losing fluid through the damaged skin during the day, causing worsening dehydration (D); however she is now so fluid depleted that she has pre-renal acute kidney injury and hypovolaemia (E). This may subsequently result in acute tubular necrosis and thus a renal cause of acute kidney injury (C), but the primary cause is (E). The high sodium and haemoglobin would also support the diagnosis. Causes of post-renal acute kidney injury (A) are obstructive, such as ureteric stones, bladder outflow obstruction and neuropathic bladder. Poisoning (B) should be considered in children with drowsiness, especially in the context of non-accidental injury, and clues in the history such as access to illicit substances or medicines.

A mother brings her 4-week-old baby to see you for the third time. He was born at term by normal vaginal delivery with no complications. You started him on anti- reflux medicine last week but it has not helped. He is now vomiting his whole feeds and is becoming lethargic and passing less urine and stool. His mother says he is hungry even after he vomits. The practice nurse has weighed him and he has lost 200g since last week. His mother was breastfeeding him while waiting to be seen and as you go to examine him, the baby has a large milky vomit, which cascades over the clinic floor. What is the most likely diagnosis? A. Gastroenteritis B. Volvulus C. Necrotizing enterocolitis (NEC) D. Intussusception E. Pyloric stenosis

E. Pyloric stenosis The most likely explanation is pyloric stenosis (E), which classically presents at 1 month with projectile vomiting and is more common in males. This is caused by hypertrophy of the pyloric muscle at the gastric outlet leading to delayed stomach emptying and gradually increased vomiting. These children are often hungry after the vomit. A volvulus (B) and intussusception (D) would have significant irritability, abdominal distension and tenderness associated with them, in addition to vomiting. NEC (C) is predominately an illness of the premature infant presenting with abdominal distension and bile stained gastric aspirates. When it occurs in term infants there is usually a history of birth asphyxia or severe growth restriction. Gastroenteritis (A) can occur in young infants especially if parents are not using sterilized (boiled and cooled) water to make up formula. It normally presents with a short history of vomiting and diarrhoea in an unwell infant.

A 14-year-old girl presents with a four-month history of recurrent abdominal pain in the evenings. You notice a mass on palpation of the abdomen which indents on palpation. Blood tests reveal no abnormalities. You note in the history that the patient's intake of roughage is poor. A appendicitis B coeliac disease C ulcerative colitis D intussusception E Peutz-Jeghers syndrome F Crohn's disease G anorexia nervosa H constipation I peptic ulcer J Wilms' tumour

H constipation Constipation is associated with a diet poor in roughage (fruit and vegetables). An abdominal x-ray would typically show faecal loading.

13-year-old girl presents with 3-day history of clear vesicles affecting the sides of her fingers and toes bilaterally. There is a strong family history of asthma. A atopic eczema B meningococcal septicaemia C alopecia D systemic lupus erythematosus E Henoch-Schönleinpurpura F contact dermatitis G erythema multiforme H pompholyx I psoriasis J incontinentia pigmenti

H pompholyx This usually occurs in children over 10 years of age, often occurs with sudden onset and is of unknown aetiology. There is usually a history of previous episodes and there is usually a family history of atopic disease (asthma in this case). The condition is self-limiting but it is intensely itchy. Symptomatic relief may be required.

A mother brings in her 4-year-old child with a prominent barking cough. He is systemically well but recently suffered from a cold. A foreign body B asthma C pulmonary haemorrhage D pneumothorax E respiratory distress syndrome F congenital heart disease G meconium aspiration syndrome H pneumonia I croup J respiratory syncitial virus

I croup Croup is the common name for acute laryngo-tracheobronchitis. It is commonly caused by infection with one of the parainfluenza viruses. The history of a barking cough in a patient that is systemically well is strongly suggestive of croup. Severe croup is less common but may present stridor and acute upper airway obstruction. This is a medical emergency and requires input from an anaesthetist and ENT surgeon.

What size of boys' testes (in ml) is consistent with onset of puberty? Select one: a. 4 b. 8 c. 5 d. 6

a. 4 One of the first signs of puberty for boys is testicular enlargement. This is can be measured with an orchidometer. Teste volume >4ml is very reassuring when there are concerns regarding the possibility of pubertal delay.

The Reference Nutrient Intake for a vitamin or mineral is likely to meet the requirements for what percentage of the population? Select one: a. 97% b. 50% c. 84% d. 16% e. 3%

a. 97% The Reference Nutrient Intake is set at 2 standard deviations above the estimated average requirement and therefore is likely to meet the needs of approximately 97% of the population.

What is the average age (in years) that a boy in the UK experiences their pubertal growth spurt? Select one: a. 14 b. 11 c. 12 d. 13 e. 10

a. 14 The average age for a boy to experience his pubertal growth spurt is 14 years. Boys experience their peak height velocity towards the end of puberty whilst girls tend to grow faster at the beginning of puberty. Growth stops soon after the pubertal growth spurt. The fact that boys have a few years of additional growth before this growth spurt is one of the reasons why, on average, men are taller than women.

Which of the following statements best describes the reason for measuring the occipitofrontal (OFC) circumference in infants? Select one: a. An increasing head circumference may be the first or only sign of a problem b. Regular measurement can be used to detect abnormal brain/skull growth c. It is a good indicator of the general health of an infant d. An increasing OFC can be an important finding in children who have subdural haemorrhages from non-accidental injury e. It is helpful in the assessment of a child's nutritional status

b. Regular measurement can be used to detect abnormal brain/skull growth Whilst all of the above statements are true, the principal aim of checking OFC regularly is to detect abnormal brain/skull growth.

What is the average age (in years) that a girl enters puberty in the UK? Select one: a. 9 b. 12 c. 11 d. 10

c. 11 The average age for a girl to enter puberty in the UK is 11 years. There is a wide range of what is considered normal for starting puberty - anything between the age of 8-13 years. There is also a secular trend whereby girls are entering puberty earlier than previous generations. Girls' puberty follows three distinct phases: adrenarche (greasy skin, spots, hair); thelarche (breast development); then menarche (periods usually start 2-3 years after thelarche). On average, boys in the UK enter puberty at 11.5 years.

Personal and social milestones

• sleeps and feeds - newborn • will smile back at parent or an examiner - 1-2 months • smiles spontaneously - 1.5-3 months • finger feeds - 4-8 months • chewing - 5-7 months • shy when with strangers, 5-10 months • drinks from cup, 6-16 months • takes off clothes - 14-20 months • dry during day - 18-38 months • separates easily from mother - 20-50 months • uses knife and fork - 32-50 months


Set pelajaran terkait

MCN - Unit 1 - Chapter 2: Family-Centered Community-Based Care

View Set

Final Exam Review Questions Fall 2023

View Set

Chapter 35 Geriatric Emergencies

View Set

Insurance Ch 7 Long Term Care Insurance

View Set

Multiplying Numbers in Scientific Notation, Dividing Numbers in Scientific Notation

View Set